^Ti \. r\%. .r,0^ % «>.% IMAGE EVALUATION TEST TARGET (MT-3) Y /. < ^^ ^1 L<*^ 1.0 1.1 iiM i'l^ !!!!!M |||||W 2.0 1.8 1.25 1.4 1.6 ^ 6" - ► % '".% "^^ "t- -1 /^ m Photographic Sciences Corporation 4 iV V % V <^ 1. •s^ \ \ <^ O^ 23 WEST MAIN STREET WEBSTER, NY, 14580 (716) 872-4503 CIHM/ICMH Microfiche Series. CIHM/ICMH Collection de microfiches. Canadian Institute for Historical Microreproductions Inststut canadien de microreproductions historiques 1980 Technical and Bibliographic Notes/Notes techniques et bibliographiques The Institute has attempted to obtain the best original copy available for filming. Features of this copy which may be bibliographically unique, which may alter any of the images in the reproduction, or which may significantly change the usual method of filming, are checked below. L'Institut a microfilm^ le meilleur exemplaire qu'il lui a 6t6 possible de se procurer. Les details de cet exemplaire qui sont peut-dtre uniques du point de vue bibliographique, qui peuvent modifier une image reproduite, ou qui peuvent exiger une modification dans la mdthode normale de filmage sont indiquds ci-dessous. n Coloured covers/ Couverture de couleur □ Coloured pages/ Pages de couleur □ Covers damaged/ Couverture endommagde □ Covers restored and/or laminated/ Couverture restaurde et/ou pellicul6e □ Pages damaged/ Pages endommagdes D Pages restored and/or laminated/ Pages restaur6es et/ou pellicul^es □ Cover title missing/ Le titre de couverture manque □ Pages discoloured, stained or foxed/ Pages ddcolor^es, tachetdes ou piqu^es D D Coloured maps/ Cartes gdographiques en couleur Coloured ink (i.e. other than blue or black)/ Encre de couleur (i.e. autre que bleue ou noire) I tT Pages detached/ I V I Pages d6tach6es A Showthrough/ V-J Transparence D Coloured plates and/or illustrations/ Planches et/ou illustrations en couleur □ Quality of print varies/ Quality in^gale de I'impression D d n Bound with other material/ Reli§ avec d'autres documents Tight binding may cause shadows or distortion /J along interior margin/ La reliu-p serree peut causer de I'ombre ou de la distortion lu long de la marge int6rieure Blank leaves added during restoration may appear within the text. Whenever possible, these have been omitted from filming/ II se peut que certaines pages blanches ajout^es lors d'une restauration apparaissent dans le texte, mais, lorsque cela 6tait possible, ces pages n'ont pas 6t6 film^es. D D n Includes supplementary material/ Comprend du materiel supplementaire Only edition available/ Seule Edition disponible Pages wholly or partially obscured by errata slips, tissues, etc., have been refilmed to ensure the best possible image/ Les pages totarement ou partiellement obscurcies par un feuillet d'errata, une pelure, etc., ont 6t6 film6es d nouveau de fagon d obtenir la meilleure image possible. D Additional comments:/ Commentaires suppl6mentaires: m This item is filmed at the reduction ratio checked below/ Ce document est filmd au taux de reduction indiqud ci-dessous. 10X 14X 18X 22X 26X 30X 7 12X 16X 20X 24X 28X 32X The copy filmed here has been reproduced thanks to the generosity of: National Library of Canada L'exemplaire film<^ fut reproduit grdce d la g6ndrosit6 de: Bibliothdque nationale du Canada The images appearing here are the best quality possible considering the condition and legibility of the original copy and in keeping with the filming contract specifications. Les images suivantes ont 6t6 reproduites avec le plus grand soin, compte tenu de la condition et de la nettetd de l'exemplaire filmd, et en conformity avec les conditions du contrat de filmage. Original copies in printed paper covers are filmed beginning with the front cover and ending on the last page with a printed or illustrated impres- sion, or the back cover when appropriate. All other original copies are filmed beginning on the first page with a printed or illustrated impres- sion, and ending on the last page with a printed or illustrated impression. Les exemplaires originaux dont la couverture en papier est imprim6e sont film^s en commengant par lo premier plat et en terminant soit par la dernidre page qui comporte une empreinte d'impression ou d'illustration, soit par le second plat, selon le cas. Tous les autres exemplaires originaux sont film6s en commen9ant par la premidre page qui comporte une empreinte d'impression ou d'illustration et en terminant par la dernidre page qui comporte une telle empreinte. The last recorded frame on each microfiche shall contain the symbol —^ (meaning "CON- TiNUED"), or the symbol V (meaning "END"), whichever applies. Un des symboles suivants apparaitra sur la dernidre image de cheque microfiche, selon le cas: le symbole — ♦- signifie "A SUIVRE", le symbole V signifie "FIN". Maps, plates, charts, etc., may be filmed at different reduction ratios. Those too large to be entirely included in one exposure are filmed beginning in the upper left hand corner, left to right and top to bottom, as many frames as required. The following diagrams illustrate the method: Les cartes, planches, tableaux, etc., peuvent dtre filmds d des taux de reduction diffdrents. Lorsque le document est trop grand pour gtre reproduit en un seul cliche, il est film6 d partir de Tangle supdrieur gauche, de gauche d droite, et de haut en bas, en prenant le nombre d'images n6cessaire. Les diagrammes suivants illustrent la mdthode. 1 2 3 1 2 3 4 5 6 HIGH SCHOOL CHEMISTRY, CONSISTINtt OF DtKRCTlONS FOR PERFORMING A SERIES OF EXPERIMENTS, WITH TRST QUESTIONS ON THB EXPERIMENTS, AND SIMPLE PROBLEMS FOR Ils^VESTIGATIOIsr. BY A. P. KNIGHT, M, A., M. D., PRINCFPAL, COT.LEGIATE INSTITUTK, K.INGSTON, CANAIV ^tithoii^cb bo the glcpartmcnt x){ Clbucatron for (Dntarm TORONTO; THE COPP, CLARK COMPANY, LIMITED 9 FRONT STREET WEST. 1887. ) ^ f Entered according to Act of tho ParliRinent of Canada, in the year one thousand eifjfht hundred and eighty-seven, hy Tub Coi-p, Clark Companv, Limited, Toronto, Ontario, in the (MBoe of the Minister of Agrieulture. TO M. R. M AND TO N. F. DUPUIS, M. A., F.B.S., EDIN. Profcsnov of Mathematicn, Queen's College, Kingston, Canada, I DEDICATE THIS BOOK, In grateful acknow ledgment that what is of most worth in my life and work has come to me through theirs. INTRODUCTION. A knowledge of chemistry is acquired — 1. By experiment ; 2. By accurate obsei-vation of the phenomena revealed by experiment, and by systematically recording these observations as they occur ; 3. By reasoning based upon our observations ; 4. By investigation ; that is, by originating new pro- cesses or employing those with which the student is already familiar, and applying them to the solution of new problems ; 5. By a knowledge of chemical theory, which is itself based upon a knowledge of the facts of chemistry, and becomes in its turn an aid in making further progress in the science. All these means of acquiring a knowledge of chemistry have been steadily kept in view in the preparation of this manual. I have long held the opinion that a text- book for beginners in chemistry should consist mainly of directions for performing a series of experiments. These experiments should be so graded and arranged that all the prominent facts and principles of the science could be re-discovered, as it were, by any intelligent pupil who performed the experiments and reasoned upon the phenomena they presented. n I NT HOD UOT I ON. How far the teachers of chemistry have departed from these principles in the preparation of text-books can be seerA by opening the first hook on tlie subject which you may chance to lay your hand on in any book store. Here is an extract from one of these books by a very distinguished chemist : — "ExFKUiMRNT. — Let U8 buni our taper in a clean glass hottlo with a narrow neck ; after it has burnt for a few minutes we notice that tlic fiamo grows less and less, and in a short time the taper goes out. Tliis IB the first thing we have to observe. We next have to discover why the taper goes out. For this purpose let us see whether the air in the bottle is now the same as it was before the candle was burnt. How can we tell this? Let us pour some clear lime-water first into a bottle filled with air in which no candle has burnt, and then into the one in which our taper burnt. You see the diflference at once ! In the first bottle the lime-water remains clear, in the second it becomes milky. Hence wo see that the air has been changed in some way by the burning of the taper. This milkinesa is nothing else than chalk, and chalk is made up of lime and carbonic acid. Carbonic acid is, like common air, a colourless invisible gas which wo cannot see, l)ut which we find turns the lime-water milky, and puts out a burning taper. Part of the wax has been changed by burning into this carbonic rcid gas ; that is, the carbon or charcoal of the burnt wax is to be found again in this invisible gas. Some of this carbon you may notice going away unburnt as smoke or soot ; and if you quickly press a sheet of white paper on to the flame so as not to burn the paper, you will see that it becomes stained with a black ring of soot or carbon." Now, the observations, (a) that the flame grew less and went out, (d) that the lime-water became milky, (c) that the carbon of the wax passed off unburnt as soot, (d) that in doing so, it blackened a sheet of white paper, should all be made by the pupil, without assistance from either text-book or teacher. If this be not done, the pupil becomes " the mere recipient of another's observa- tions." In the same way the conclusions, (a) that the air was changed in some w^ay by the burning candle, and (d) that the something produced by the burning lNTll.Oi)U(;TlON. Vll candle " is, like common air, a colorless, invisible gas," should also be reached by the pupil without assistance of riny kind. Only when every pupil in a class fails to make an important observation, or to draw a legitimate con- clusion, should the teacher step in and give his class assistance. This remark, of course, does not apply to much valuable information, the result of long and diffi- cult experiments by great chemists. It applies only to the numerous experiments which may be easily per- formed in any fairly well equipped school laboratory. Any assistance which the pupil requires can easily be conveyed by a series of well-considered questions on each experiment performed. This is the object in the large number of questions found throughout this book. Observations and conclusions have been de- signedly rsmitted. These the teacher will readily find in the thousand and one works on chemistry which have come from the printing press during the past fev/ years. Here and there I have inserted a few simple problems, which are intended to stimulate in the pupil a desire for original research, as well as to test his power of applying the knowledge he has already acquired to the solution of new problems. I agree with the Editor of the Chemi- cal News that even beginners should aim at doing origi- nal work. "Once more the laboratories and lecture-rooms of our colleges are thrown open, ami once more professors, demonstrators, and students are preparing to resume their duties. Once more, also, in accordance wi h our custom, we take it upon ourselves to address a few hints to those who are beginning or continuing the study of chemistry. We haoe alivay-i culvisfd the student to (jnalify liimnelf for orhi'inal renearch, believing that such will prove the best coitrso of training, whether for pure chemistry or for its industrial application," — Chem, News, Sept. 18th, JSS5. 1" Mil INTKODUt'TION. The book is, however, purely a pupil's text-book. Its use in any school implies that the teacher possesses a thorough knowledge of elementary chemistry. It im plies, also, that the pupils have access to the best works of reference on the subject, so that in case of doubt they may appeal to the highest authority. The book is intended to cover two years of school work. In the first half, the aim has been to furnish young students with a stock of facts upon which to found, subsequently, a fuller knowledge of the principles of the science. It is scarcely necessary to say that the calculation of chemical constants is very difficult work for the junior pupils of our secondary schools. Such pupils should not, in my opinion be allowed to read beyond Chapter XXVI 1 1, during their first year in chemistry ; and they might perhaps with profit omit Chapters IX. and XIV. Beginning at Chapter XXIX., the plan of the book was somewhat changed. In that and in a few succeeding chapters an attempt was made to acquaint senior students with the principal theories of chemistry — the knowledge previously acquired being used as a foundation. Then follows a brief study of some of the more important metals, theory and practice being kept prominent, but questions regarding mere observation of facts being almost entirely omitted. A brief outline of Mendelcjeff's classification of the ele- ments comes in naturally towards the end of the book, after the student has acquired some knowledge of the principal elements and their characteristic compounds A few words may not be out of place as to my own practice in tcachlngl^chemistry. All the simpler experi- ments — probably two-thirds in number — are performed INTKODUCTION. l.\ by the })upils, each working at a separate table. Tlie more difficult ones are made by myself in presence of the class. In both cases the pupils arc required to record neatly, in note books, a description of the opera- tions and of the phenomena revealed ; and, lastly, a con- clusion based upon the facts of the experiment, or of it and others which preceded it. A little more than half of the time devoted to the subject is allotted to experi- mentation and to recording results ; the rest of the time is spent in systematic drill in class, in the shape of oral questions on the observations made and the conclusions reached. Prof. Huxley, the late Prof. Miller, and other great teachers, all advocate the method of teaching science that has been adopted in this book. Not only so, but the same method is advocated by writers who are con- sidered high authorities on the principles and practice of teaching. "It la becoming more generally accepted every clay by good teachers not o-.ily of chemistry, but of physics, that the best teaching is given in the laboratory rather than in the lecture room. It is not merely by seeing experiments tried, but by trying them, that the properties of objects, their structures and organization are best learned. But here it must be borne in mind that the discipline you want to give must be definite and exact ; it is not seeing and handling only, but careful measurement if it be mechanics, careful observation if it be botany or physiology, and wliatever it be, careful notes and recordation of the results of each experiment as it is made." — Fitch's Lectures on Teaching. "In manhood, when there are no longer teachers at hand, the observations and inferences required for daily guidance, must be made unhelped ; and success in life depends upon the accuracy and complete- ness with which they are made ... To tell a pupil this and to show him the other, is not to teach him how to observe, but to make him a mere recipient of another's observations ; a proceeding which weakens rather than strengthens his powers of self-instruction, and deprives him of the pleasures resulting from successful activity." — Education, by Herbert Spencer. X INTKODUCTION. If My owti teaching for the past ten years has been along the Hnes indicated, but it was not until October, 1885, that I found time to enlarge my lesson notes and place them in the hands of the Copp, Clark Company for publication. At that time I intended that the book should be through the press before the beginning of the New Year (1886), but for one reason or another its issue has been delayed until now. The publishers are indebted to Messrs. J. & H. Bergc, dealers in chemical apparatus, 95 John St., New York, for the use of a few of the illustrations used in this book. I have to express my indebtedness to Professor Goodwin, of Queen's College, and to Professor Waddell, of the Royal Military College, for valuable suggestions in preparing this manual, and for kindness in reading most of the proof-sheets ; also, to Mr. W. Lochhead, B. A., of Cornell University, for assistance in preparing my manuscripts for the prejs. This gentleman is the joint author of the articles on sodium, iron and lead ; and the sole author of the chapter on the elements of the silver family. The works of the following named authors have been freely consulted in the preparation of this hand-book : Roscoe and Schorlemmer, Remsen. Muir, Wurtz and Reynolds. Ki.vGSTON, 1887. . TABLE OF CONTENTS. CHAPTER I. Solution of Solids, Liquids and Oases— Filtration— Theorj' of St»lution. — pp. 1-5. CHAPTER II. Physical Change. — Chemical Change— Forces Employed in Chemistry, —pp. 5-10. CHAPTER III. Mechanical Mixture- — Chemical Compounds— Decomposition— Com- bination of Elements — Elements and Compounds. — pp. 10-14. CHAPTER IV. Air. — What Composed of— Burning— Matter Indestructible — Law of Constants — Weight and Volume.— pp. 14-2 L CHAPTER V. Oxygen. — Preparation and Properties of— An oxide.— pp. 21-25, CHAPTER VI. Nitrogen. — Preparation and Properties of — The Atmosphere— \nalvsia of Air.— pp. 25-28. ^ " ^ CHAPTER VII. Pure Water.— How Obtained — Analysis of Water— Synthesis of Water— Volume of ISteam. — pp. 28 34. CHAPTER VIII. Hydrogen. — Preparation and Properties of— Water from Hydrogen- Hydrogen from Steam— Steam from Hydrogen, —pp. 34-44. CHAPTER IX. Physical Characters of Gases.— Boyle's Law— Charles' Law— Avogadro's liiw— Theories and Facts— Molecules and Atoms- Specific Gravity of Cases— Atomic Weight— How to Find Atomic Weights — iJulong and Petit's Law — Displacement — Chemical Equivalents— Electro-chemical Series.— pp. 44-52. f . Xll TABLE OF CONTENTS. CHAPTER X. Elements and Symbols. — Metala and Non-metals— Chr jiical Nota- tion — Formulae, Empirical and Rational — Atomicity or Valance — Graphic FormulaB — Chemical Nonienclature — Equations — Signs — Molecular Equations. — pp. 52-65. CHAPTER XI. Carbon. — Preparation and Properties of — AUotropic Forms. — pp. 65-68. CHAPTER Xir. Carbon Dioxide- — Preparation and Properties of — Decomposition of Carbon Dioxide. — pp. 68-70. CHAPTER XIII. Carbon Monoxide. — Preparation and Properties of — Law of Multiple Proportion.— pp. 70-72. CHAPTER XIV. Units of Volume. — Chemical Calculations — Exercises. — pp. 72-75. CHAPTER XV. Hydrocarbons. — Methane — Ethylene — Coal Gas — Coal Tar. — pp 75-79. CHAPTER XVI. Combustion. — Incandescent Gases — Structure of Flame — Candle Flame —Kindling Point.— pp. 79-83. CHAPTER XVII. Sulphur. — AUotropic Forms — Sulphur Dioxide — Bleaching Explained — Sulphurous Acid — Sulphur Trioxide — Sulphuric Acid, Prepara- tion and Properties of — Sulphuretted Hydrogen. — pp. 83-90. CHAPTER XVIII. AcldS- — Hydroxides — Salts — Naming Acids and Salts.— pp. 90-95. CHAPTER XIX. ' Nitric Acid. — Preparation and Properties of — Sources and Tests of. — pp. 95—99. CHAPTER XX. Nitrogen and Oxygen. — Compounds of —Nitrous Acid — Nitrogen Monoxide, Dioxide, and Trioxide, l*reparation and Properties of — pp. 99-106. TABLE OF CONTENTS. xiii CHAPTER XKI. Ammonia. — ^ reparation and IVoperties of — Atataouiuni Hyclroxido — Analysis of Ammonia — Ammonic Chloride -Hydroxides Formed from Salts— Sources and Teats of Ammonia, — pp. 105-114. CHAPTER XXII. Hydrochloric Acid- — Chlorides — Action of Acitls on Metals — Analysis of Hydrochloric Acid — Tests for. — pp. 114-119. CHAPTER XXIII. Chlorine- — Preparation and Properties of — Bleaching Powder — Chlor- ine Compounds. — pp. 119-127. CHAPTER XXIV. HologenS' — Iodine — Bromine — Fluorine — Hydrofluoric Acid. — pp. 127-132. CHAPTER XXV. Lim©— Burning Limestone —Calcic Hydroxicje — Artificji^l Limestone — Solution of Limestone — Mortar — Gypsum. — ^pp. 182-i;i7. CHAPTER XXVI. Phosphate of Lime- — Phosphorus, Redand Yellow— Matches — Acids from Phosphorus — Phosphorus and Hydrogen. — pp. 137-143. CHAPTER XXVII. Sand- — Silicates— Clay — Alumnia — Glass — Pottery. — pp. 1 43- 1 47. CHAPTER XXVIII. » Analysis of Air — By Volunjie and Weight — Impurities in Air — Ozone — Organic Matter — Expired Air — Graham's Law — Ventilation — pp. 147-155. CHAPTER XXIX. Water. — Impurities in — Rain Water — Spring Water — River Water — Sea Water — Tests for Impurities — Hardness. — pp. 155-164. CHAPTER XXX. Dalton's Theory. — Division of Molecule — Unit of Volume. — pp. 164 169. CHAPTER XXXL Classification of Elements. — Selenium — Tellurium. — pp. 169-172. CHAPTER XXXII. The Nitrogen Family. — Arsenic Poisoning — Antimony — Bi»muth. — pp. 172 177. XIV TAHI-E OF CONTKNTS. CHAPTER XXX 1 1]/. Boron. — Horadc Acid — Aluminum, -pp. 177-181. CHAPTEK XXXJV. Extraction of Metals.— Metallic Derivatives.— pp. 181-183. CHAPTER XXXV. Calcium Family.— Calcium — Barium-Strontium. —pp. 183-139. CHAPTER XXXVI. Magnesium Family. — Compounds — Zinc — Compounds and Tests. - pp. 189-194. CHAPTER XXXVII. The Alkali Family. — Potassium ami Compounds — Sodium and Com- pounds — Resemblances — Ammonium, ^jp. 194-203. CHAPTER XXXVIII. The Lead Family. — Lead and Compounds — Tin and its Compounds — pp. 203-212. CHAPTER XXXIX. The Platinum Family.— Cold.— pp. 212-21.5. CHAPTER XL. The Iron Family.— iron and its Compounds. — pp. 215-222. CHAPTER XLI. The Manganese Family.— Compounds of Manganese.— pp. 222-220. CHAPTER XLIL The Periodic Law.— ^Tts Use — Laws of Isomorphism. — pp. 226-231. CHAPTER XLIII. The Silver Family.— Silver— Copper-Mercury.— pp. 231-238. CHAi TER XLIV. Chemical Analysis.— Analytical Tables.— pp. 238-245. APPENDIX. Nitric System. — List of Elements — Science Room- -Apparatus and B" -".nts, List of — Books of Reference — Examination Questions. — 5-278. PRACTICAL CHEMISTEY. f 1 TO THE STUDENT. Before beginning your studies, it becomes necessary to give a few general directions regarding chemical manipulations. The following will be found useful at every stage of your studies : — 1. Work carefully and note all phenomena that occur, and record them in your note-book. 2. When heating a liquid in a test-tube take care that the flame does not strike the tube at the upper end of the liquid. 3. Never hold the mouth of a test-tube, while heating it, towards yourself, or towards another. 4 Never heat beakers, evaporating dishes or flasks without placing a piece of wire gauze or a sand- bath between them and the flame. 5. Use sulphur matches ; parlor ones are dangerous. 6. Do not lay down the cork of a reagent bottle while pouring out a solution, but hold it between the first and second fingers. 7. Have a place for every piece of apparatus and every reagent you use, and when you are done using them, clean the apparatus and put it and the re- agents back in their respective places. PRACTICAL CHEMISTRY. CHAPTER I. § 1.— Solution. Let us begin our study of chcMnistry by some experi- ments. JElxperiments. 1. Fill a small beaker half-full of water, and then suspend in the water by a thread a piece of white sugar, or a crystal of sulphate of copper. 2. Place about a tablespoonful of water in a small evaporating dish, stir into it a small quantity of salt or alum, and after it has entirely disappeared, heat the solution over a spirit lamp, as in Fig. I, until all the water is driven off. 3. Repeat the preceding ex- periment, using weighed quan- tities of salt and of water, and then weigh the solution. 4. Take an ounce each of pow- dered alum, washing soda, and copper sulphate, and dissolve each in a fluid ounce of water in three separate beakers. Stir with a glass rod, and observe the extent 2 Fio. 1. Fio. 2. p ■■ PRACTICAL CIIKMIHTHY. to which each one dissolves. Now heat the solutions until they boil, stir apjain, and notice whether there is any variation in solu- bility. Pour each hot solution into a separate plate, and allow them to cool slowly. After some time, examine carefully and com- pare the geometrical forms which the solids have taken with those in Fig. 2. 5. Ascertain whether calcium chloride is soluble in alcohol ; and sulphur, in carbon disul- phide. In order to test the accuracy of the observations which you have made in performing the preceding experiments, try to answer the followine Fiu. 2. QUESTIONS. 1. Wh^re did the solids go to ? 2. Devise experiments to find out liow a solid may be got most quickly into solntion. 3. How may a solid be obtained from the liquid in which it is dis- solved ? 4. How does the weight of a solution compare with the weight of the solid and liquid which form it ? B. Mix thoroughly some powdered salt and sulphate of copper to- gether, and then devise means of separating them, basing your device upon the preceding experiments. 6. Ascertain, by evaporation, whether there is any solid matter dis- solved in a sample of river or spring water. 7. What influence has temperature on the quantity of a solid held in solution ? 8. Rnd out a substance insoluble in water, but soluble in carbon disulphide. 9. How can salt be obtained from sea-water. Define solution. inrrniiiiTirii^nm FILTRATION. Saturation. — When a liquid has dissolved all that it can of a given solid, at a given temperature, the liquid is said to be saturated. § 2.— Filtration. Experiments- 1. Half-fill a beaker with water, and into it drop a piece of quick-lime the size of a bean. Stir the mix- ture, and then filter it. This is done by folding a cir- cular piece of filtering paper — white blotting paper will do well enough — across twice, so as to form a quadrant, and fitting it to a funnel, as in Fig. 3. A glass stirring rod should be held vertically above the funnel, and the solution poured down the rod. Catch the liquid that passes through the filtering paper, and taste it. Evaporate some of it upon a piece of platinum foil, or p^os. upon a sheet of mica. 2. Ascertain whether salt or sugar dissolved in water can be removed bv filtration through paper. QUESTIONS. 1. What kind of matter is usually removed from a liquid, by fil tratiou ? What, by evaporation. 2. Did the quick-lime dissolve in the water, or remain in suspension, or (lid it do both ? Give reasons for your answer. § 3.— Solution of Liquids. Experiments. I. Fill a small test-tube to the depth of 4 centimetres with water, and then add chloroform, or sulphuric ether, to the depth of I centimetre. Shake well, and allow 4 PUAt'TlrAL imK\xa AIR. 16 and tin-foil, when these were burnt in air, shewed us that the ashes formed from these metals weighed more than the metals themselves. Whence came this increase of weight? Evidently these metals, when heated, united with the air, or with something in the air, and this air, or its gaseous constituent has taken the solid form in the ashes. We can easily devise an experiment which will revea; the source of the gain in weight, as well as answer, partially at least, the question : 0/ what is air composed ? Experiments. 1. Wet the inside of a pickle bottle with water, and drop into it some fine iron filings. Then shake the bottle so that the inside may become closely sprinkled with the filings. Place the bottle, mouth downward, over a soup-plate filled with water, and allow the whole to stand for a day or two. Without awaiting the result of the preceding experi- ment, proceed with the following ones. 2. Cover a cork about two inches in diameter with a piece of tin and float on a soup plate full of water. Take a piece of phosphorus about the size of a pea, and place it on the cork. Now set fire to the phosphorus, and then cover it quickly with a beaker or small hell jar, placing it mouth downwards, as in Fig. 7. Allow it to stand thus for 1 5 or 20 minutes. 3. Prepare the gas as before, using a large bell jar, then transfer it by means of a pneumatic trough to four separate beakers, and proceed to perform the following experiments : — (a). Smell the contents of the first beaker. (ft). Plunge a burning taper into the second. Fia. 7. \:t I |l ! V, 16 PRACTICAL CHEMISTRY. Ill I (r). l*our some lime-water into the third, and then shake it. (d). Try to pour the gas in the fourth downward into an " empty " beaker. (e). Try to pour the contents of the fifth upvjard into an " empty " beaker. (/). Test the result of (d) and (e) by plunging lighted tapers into the vessels into which you tried to pour the gas. QUESTIONS. 1. Compare your results in experiment 2 with those of other stu- dents, or repeat the experiment, using more phosphorus than at first. 2. Is air a simple or a compound substance ? Give reasons for your answer. 3. By how much was the original air reduced in volume? Where did it go to ? 4. If air is composed of two things, how can one be distinguished from the other? 5. What color had the substance which was formed by burning the phosphorus ? Where did it go to ? 6. What effect had the gas on the burning taper ? On the lime- water ? Is the gas heavier or lighter than air ? 7. What gas escaped on covering the burning phosphorus with the bell jar ? Why ? The gas that remained in the bell jar is known as nitrogen ; that which united with the phosphorus, mag- nesium, iron filings, and tin foil is called oxygen. The latter is often called a supporter of combustion, and the former, a non-supporter. I § 18.— Burning. Several interesting inquiries are suggested by these experiments. Are all cases of ordinary combustion due to the oxygen in the air ? Will air burn in an atmosphere of coal gas, just as coal gas burns in the air ? Is matter ever lost or destroyed in combustion ? If the combustion of a substance, like magnesium, and the increase in weight which takes place, are due to BURNING OP AIR. 17 union with the oxygen of the air, does the union of the oxygen and magnesium take place in definite, or in variable proportions by weight? These inquiries will be partly answered in the following experiments. Experiments. 1. Light a short piece of candle and float it on a flat cork in a soup plate filled with water. Then cover the floating candle with a jar, as in Fig. 8, pressing the jar down into the water. 2. Take a small lamp-chimney and place it over the lighted can- dle used in the preceding experi- ment. Press the chimney firmly upon the cork so that no air can enter from below. The candle must, of course, be placed upon a table in this case. 3. Cut a piece of cardboard so as to fit the upper two- thirds of the chimney, place it inside, and then repeat the preceding experiment. QUESTIONS. 1. In which of these experiments diil the candle go out ? Why ? 2. What substance was supplied to the burning candle in the third experiment which was not supplied to it in the first or second ? 3. Criticize the following conclusion, supposing it to be based upon the foregoing experiments .—All things, when they burn, unite with the oxygen of the air, and increase in weight. Fig. 8. § 10.— Burning of Air. Experiment. Take a small lamp-chimney and fit into its lower end a good cork carrying two tubes. The upper end of one of these must merely pass through the cork, and its lower 3 eae 18 I'HACTIOAL CHEMISTRY. end must be attached to the gas supply. Tho other tube must move easily through the cork, and must be long enough to project above the top of the chimney ; its lower end must be attached to a tube supplying a steady .stream of air. When the apparatus is all ready, turn on the gas, and after a minute or two, light the gas at the top of the chimney. Then turn down the gas until there is only a small flame ; move up the long tube to the top of the chimney ; turn on the air supply and when the air is alight, withdraw the air tube until the flame of burning air is about the middle of the chimney. Ill li' § 20.— Matter Indestructible. In most cases of ordinary combustion there appears to be an utter destruction of the substances burnt. Wood, coal, oil, wax, and many other things burn and leave little or no trace behind them. Is the matter of which they are composed really destroyed, or does it also unite with the oxygen of the air, take on new forms, and increase in weight just as magnesium did ? We shall put this question to Nature in the shape of an experiment. Experiments. I. Pass the product of the combustion of a candle or of a coal oil lamp through the apparatus Fig. 9, at- taching, if necessary, an aspirator to the U tube D, in order to draw a current of air through it. Weigh the candle before and after the experiment,and weigh the U tube D also, and the bent tube C. The tube D contains pieces of caustic potash, and the middle part of the tube C is immersed in water, in order to keep it cool. Fig. 9. LAW OP CONSTANTS. 19 2. Repeat this experiment first putting cnouj^h lime water in the tube C to merely cover the bend. QUESTIONS. 1 . What change took place in the weight of the can»^Hha>«MHUKiMM 22 rilACTICAL CHEMISTRY. IHii Experiments I. Weigh out I gram of red oxide of mercury and Fia. 10. place in a test-tube fitted with a cork and deh'very tube as in Fig. lO. Heat uniformly at first so as not to break the glass. Oxygen is driven off, and collects in the upright glass vessel standing in the " pneumatic trough." As soon as the disengagement of gas slackens, you should raise the end of the delivery tube from the water, and then extinguish the lamp. The gas that col- lects at first should be thrown away. Raise the tube containing the oxygen out of the water and plunge a glowing splinter into it. 2. Heat some red oxide of lead in the same manner as in the preceding experiment. Test as before. 3. The best method of preparing oxygen in moderately large quantities in a small laboratory is the following : — Into a mortar put 10 grams of potassic chlorate and 2 J grams of manganese dioxide Powder them, and place in a dry Florence flask. Use chemically pure material only, as serious explosions have occurred from organic matter or carbon being mixed with the manganese di- oxide. Heat strongly but carefully, and collect four or five small jars of the gas ; or better still, collect in a gas-holder. Manganese dioxide is mixed with the potassic chlorate to cause the latter to part with its OXYGEN. 23 oxygen at a lower temperature than it otherwise would. The manganese dioxide remains unchanged at the end of the experiment. 4. Into a jar of oxygen plunge a piece of glowing charcoal. After combustion has ceased, pour some water into the jar, shake, and then test with blue litmus.* *To make red and blue solutions, ** steep sonic solid litnii s in water or weak alcohol. Divide the liquid which you pour off from the sediment, into two parts ; to one add a few drops of weak sulphuric at^id ; to the other add a little solution of caustic sotla. You will then have red and blue litmus solutions, and if you aoth wires appecar to yield an equal quantity ? 1 . Wh.at appeared to collect at the top of the tube ? Where did it come from ? SYNTHESIS OF WATER. 31 3. Wha,t became of the water as the gas collected ? 4. What force produced the chemical change ? 2. When all the water has been expelled by the accumulated gas in the preceding experiment, raise the tube keeping it mouth downward, and apply a lighted match to it. 3. Repeat experiment i, using two test-tubes full of acidulated water, inverted over a soup plate. Place the end of a wire under each tube. Each wire must be coated with paraffin or sealing wax where it touches the water, except about I centimetre at the end. When gas has filled one of the test-tubes, stop the current, and examine the gases. Put a glowing splinter of wood into the one with least gas in it, and apply a lighted taper to the full one. By the use of the apparatus, Fig. 15, this experiment may be easily and neatly pcrforrr.ed. This process of decomposing water is called the Elec- trolysis of water. QUESTIONS. 1. Why coat the wires with wax ? 2. What diflference is there in the amount of gas collected in each tube ? 3. Name the gas of which there was the smaller volume. Give a reason for your answer. 4. What effect had the gas in the other tube on the burning splinter ? 5. How did tho two gases differ in behaviour ? Mention three pro- perties which they have in common. The gas of which there was the larger quantity is called Hydrogen. § 31.— Synthesis of Water. We have now proved that two gases, oxygen and hydrogen, may be obtained from water, by passing a current of electricity through it ; our next step is to reverse the process, and prepare water from the union of these same gases. This process of forming water is called the synthesis of water. m^ 32 PRACTIOAF. f'HEMISTRY. Experiments- I. Take a graduated tube called a eudiometer, fill it with mercury, and invert it over mercury in a soup- plate or saucer. Then pass into it a known volume of oxygen and twice the volume of hydrogen, measuring both at the same temperature and pressure. Pass a spark from a Leyden jar, or from a Ruhmkorff's coil, through the mixed gases. Figure i6. Before igniting the gases, press the eudiometer firmly on a rubber pad placed at the bottom of the plate or saucer. After ignition examine the top of the eudiometer with a good lens. Fig. 16. 2. Repeat this experiment using equal volumes of the two gases. Test any gas that remains. 3. Try the experiment again, using twice as much oxygen as hydrogen. Test as before any gas that remains. QUESTIONS. 1. What two phenomena occurred on igniting the mixed gases — one perceived by the hand, the other by the eye ? 2. Did the mercury rise or fall in the tube after the gases were ignited ? Explain. .3. What substance was perceived at the top of the eudiometer after ignition ? In what physical state ? VOLUME OP STEAM. 33 H' 4. What gas remained in excess in the eudiometer after iu'uition in experiments 2 and li'i How did you know ? 5. In what proportion by volume did the gases combine ? § 32.— Volume of Steam. We have now to find out how many volumes of water-gas, or steam, will be produced by the union of two volumes of hydrogen and one of oxygen. Experiment. Fill a eudiometer one-third full of a mixture of hydrogen and oxygen gases — using two volumes of the former to one of the latter. Cover the eudiometer with a larger tube, into the top and bottom of which pass tightly-fitting corks perforated with tubes, admitting steam at the top and giv- ingexit to it at the bottom, Fig. 17. The wires from the battery to the eudio- meter should pass into the jacket through its upper cork. After the steam has been admitted, mark the height of the mercury above that in the trough, and also the volume of the contained gases, then ex- plode them, as in experi- ment § 31. After explosion depress the eudiometer, until the mercury in the tube stands tlie same height above that in the trough as before. Then measure the volume of the water-gas (steam) in the eudiometer, and compare this volume with that of the original mixture. If we represent equal volumes of oxygen and of hydro- gen by equal squares, and then place in these squares the first letter of the name of these elements, we can represent to the eye, by another figure, the volume of B^^ii3 o4 PRACTICAL CIIKMISTHV. watcr-^as or steam formed, and the condensation which occurs after union. Thus : 1 vol. hydro;;cii ^ + ; vol. oxyKcn. = 1 vol. hy(lro(;cn . 2 vuU. itteain. QUESTIONS. 1. Was there expansion or contraction of the original volume? To what extent ? 2. What was the volume of the steam forme«l as compareil with that of the hydrogen ? Of the oxygen ? I'— CHAPTER VIII. § 3.3.— Hydrogen. Having decomposed air and water, we naturally ask : May not oxygen, nitrogen, and hydrogen also be compound substances ? To this question we can only answer that as yet we have been unable to decompose them by any force which we have at our command, and we therefore call them elements. We shall now proceed to study the properties of hydrogen. Experiments. I. Make a mixture of water and strong sulphuric acid in the proportion of 6 to i. Fill a test-tube with this mixture, and invert it over a saucer half full of the same mixture. Below the test-tube, which must always be kept with its mouth below the Fia. 18. level of the acid and water, place some small pieces of pure zinc, as in Fig. i8. lIYDIlOdKN. 36 tcr he a nd ull he be he ce 2. Prepare two or three test-tubes full of lliis j^as ill this manner, and then perform the followin*^ experi- ments with them . — (a). Smell the j^as in one of the tcst- iubes. (b). Raise a second out of the water, and, keep- ing it mouth downward, apply a lighted match to it, QUESTIONS. 1. What became of the zinc? How did you know that the gas waa nut air ? 2. Tell how you know that this gas is not solul)lc to any great extent in water. 3. Why keep the tube with its month downward ? 4. Devise an ex^-urimeut to prove that the hydrogen couhl not have come from the water. 5. What then was the object in usiiii^ water in experiment 1 ? Prove the correctness of your answer by an experiment. 6. What forces came into play in experiment 1, anj. When explaining the facts of solution reference was made to the theory of the molecular constitution of mat- ter. This theory was, in fact, first propounded by Avo- gadro, a celebrated Italian physicist, in order to explain the facts expressed in the two foregoing laws. Avoga- dro's explanation was unheeded at the time, but it was re-announced by Ampere many years afterwards, and is consequently, sometimes called Ampere's law. It may be stated as follows : — " All substances in the gaseous state, and under like con- ditions of temperature and pressure, contain an equal number of molecules in equal volumes." 46 PKACTICAIi CIIEMISTllY. g 43.— Theories and Facts. ** Generalizations from observed facts, so lon^ as they arc uncertain and incomplete, arc called hypotheses and theories ; when tolerably complete and reason- ably certain, they arc called laws. The attention of the student should be constantly directed to the keen discrimination between, y}?r/'^', and \\\q. speculations found- ed upon these facts ; between the actual evidence of our trained senses brought intelligently to bear upon chemi- cal phenomena, and the reasonings and abstract conclu- sions based upon this evidence ; between, in short, that which wc may know and that which we may believe." "If we admit the hypothesis that gases, like other bodies, arc made up of small independent masses called molecules, and that heat causes these molecules to separate from one another, whilst cold or pressure causes them to approach, wc are led to the assumption that in equal volumes of different gases there exist the same number of molecules." On the other hand, " if we admit the law of Avogadro, we see at once why gases are equally expanded by heat, why they are equally contracted by cold or pressure, and why they combine together, according to the discovery of Gay-Lussac, in simple proportions by volume." § 44.— Molecules and Atoms. A molecule is the minutest particle of a compound or of an element capable of independent existence. An atom is the smallest part of an element that can enter into the molecule of a chemical compound. When chemical combination takes place it is supposed that one or more atoms of one element unite with one or more atoms of another clement, and as these atoms have definite weights, it follows that the compound formed must always contain the elements in the proportions of SPECIFIC OIIAVITV OP OASKS. 47 their atomic weights. In other words, the molecular wci^f^ht of a compound is the sum of the atomic wei^^hts of its constituents. § 4r)._Specific Gravity of Gases. From our experiments with water wc were led to believe that one volume of oxygen is 1 6 times as heavy as one of hydrogen. We can test the accuracy of this conclusion by actually weighing equal volumes of these gases. Experiment. Take a globe, capable of holding about half a gallon, and fitted with a good stop- cock. Attach it to a good air pump and exhaust, as far as possible, all air from it. Weigh the empty globe, then fill it with hydrogen gas from the jar. Fig. 30, by turning the stop-cocks both of the jar and of the globe. Adjust the level of the wa- ter inside and outside of the jar and then close the stop- cock. Then weigh again carefully. Exhaust it a se- cond time, then fill with oxygen gas and weigh. Fig. 30. [Consult Gauot'a Physics, edited by Atkinson, 1881, §§ .'J.So-f)]. QUESTIONS. 1 . What was the weight of the empty glol)c ? What was the weight of the globe when filled with hydrogen ? 2. What then must have been the weight of the hydrogen ? INOTE.— The siieoilie gravity of all gases may be determinca iu a similar maimer. •-"•^B" 48 PIlACTICAIi C'lIKMIHTRY. ',i. Siiiiilai ly, wluifc whh tin; wri^lit of tlic (ixy^cii ? 4. How many tiniu8 Ih oxygi;n liuavier tliaii liytlio^m ? How niuiiy times ia one inijleculo of oxygen lieavier tliuii ono of hytlrogiii ? 5. Try lu>w tho weiglit of eaeli coniparea with that of air? Ot liitrogiiu? 0. What connuctiou uxiuts iKtwccii tho weight and the vohiine in which liydrogen and oxygen cond)ine with eatih other to form water ? The § 4G.-— Atomic Weight. Atomic Wci Mercury 0.3.32 Phosphorus 0-1740 Sr. IIkat Platinum ()3l!4 Potassium O'lG.'j.') Silver ... 0-0570 Sulphur .,0-1780 Zinc , V. .... 0O95G § 48.— Dulong and Petit's Law. " The same quantity of heat is needed to heat an atom of all simple bodies to the same extent " [See Ganot's Physics, edited by Atkinson, 1881, §4.')8.] The atomic heat of an element (nearly 6.o), when divided by its specific heat, gives the atomic weight very nearly ; or, the atomic weight of an clement multi- plied by its specific heat gives the constant quantity called atomic heat. The atomic heat is nearly the same for all elements, namely, 6.0. II § 49.— Displacement. TV student must not imagine that these two are the usr nethods of finding atomic weights. The fact is 5 60 PRACTICAL CHEMISTRY. that the complete analysis of a compound, or at least the determination of the proportions by weight in which elements displace each other from chemical compounds, precedes the calculation of atomic weights, and that we generally u'ilize our knowledge of specific weight and of Dulong and Petit's law to correct or corroborate our results. Displacement of one element by another has been illustrated in the preparation of hydrogen, but we must now make a much more accurate experiment than any which we have yet tried in connection with this phenomenon. Experiments. 1. Take about l6o centigrams of acetate of lead, and dissolve in about 40 cubic centimetres of pure distilled water. Clean and dry a por- celain crucible and counter- poise it on a chemical bal- ance. Pour the solution into the crucible. Then immerse in the solution as in P^ig. 31, 1 30 centigrams of pure bright zinc in fine strips, and allow them to remain there until they have all disappeared Pour off the liquid solution, and carefully wash by de- cantation, dry, and weigh the metal that remains in the crucible. Try whether the metal will make a mark on white paper. Compare the mark, if any, with that made by the metal lead. 2. Repeat this experiment, using a solution of nitrate of silver and 12 centigrams of magnesium. Wash, dry, and weigh the residue carefully as before, and then rub ic with a polished knife blade. Compare with silver. rig. 31. [NoTB. — Tliese experiments must be vcrj' carefully iicrforined, especially as rej^ards wusliiug, dryiug and weiijhiug. A thoroughly good 'liemical balance must be used.) CHEMICAL EQUIVALENTS. 51 3.1 QUESTIONS. 1. What change did the ziuc undergo V The magnesium. 2. What weight of lead was obtained ? Uf silver ? 3. Perform one of these experiments a second time very carefully and see if you get a different result. 4. Try to displace cojiper from copper-sulphate by using iron. Then try to displace the iron from the solution by using zinc. [ate -ub 111 § 50.— Chemical Equivalents. The rcsult.s of the above quantitative experiments, and of others which we have performed thus far in our course, .show that : — I/O centigrams of silver nitrate yielded io8 of silver. 207 centigrams of lead were displaced from 325 of acetate of lead by 65 of zinc. ic8 centigrams of silver were displaced from 170 of nitrate of silver by 12 of magnesium. 8 centigrams of oxygen unite with i of hydrogen to form water. These numbers are called chemical equivalents because they represent the proportion by weight in which each of the substances named enters into or leaves chemical compounds. As hydrogen has never been known to unite with a Ic'^ weight of any substance than its own, it has for this reason been taken as a standard by which to measure the equivalences of all other substances. Definition. — Chemical equivalents are numbers representing the proportions by weight in which different substances displace one another in chemical compounds, one pa^t by weight of hydrogen being laken as the unit. § 51.— Equivalents not Atomic Wei;l ^2; EMPIRICAL FOUMUL/E. . 57 In solving this question proceed as follows : Potassium 28-73 -f 39-1 = 73 Hydrogen 73 -f 1 = 73 Oxygen 4702 ^16 = 293 Sulphur 23 52 -^ 32 = 73 Now the smallest of these quotients is 73, and divid- ing each of them by this, we obtain one for K, one for H, one for S, and four for O. The empirical formula is therefore, KHSO4, and the substance is hydro-potassic sulphate. Why do we divide the percentage of each substance by its atomic weight ? Evidently, to get the number of atoms or parts of an atom of each constituent of the compound,, if 100 parts of the compound be taken. To solve all similar problems observe the following rule: 1, Divide the j^ercentage amount of each constituent element hy its own atomic weight. 2. Divide each of the quotients thus obtained hy the lowest of them and the numbers obtained will express the number of atoms of each element in the compound. The problems in the following exercise can all be sol- ved in a similar manner. EXEBCISE. 1. Carbon. 42 '86 ; oxygen, 57*14. Ans. CO, carbon monoxide. 2. Hydrogen, 2-73, chlorine, 97-27. Ans. HCl, hydric chloride. 3. Hyd.rogen, 'SS ; sodium, 10.17 ; aulphur, 2666 ; oxygen, 53*33. Ans. HNaSO^ hydro-sodic sulphate. 4. Sodium, 39*31 ; chlorine, 60*69. Ans. NaCl, sodic chloride. 5. Nitrogen, 82*35 ; hydrogen, 17*65. Ans. NH^, ammonia. 6. Phosphorus, 91 17 ; hydrogen, 8*83. An.s. PH3, pho.sphine. 7. Carbon, 26*67 ; hydrogen, 2*22 ; oxygen, 71*11. Ans. C2H2O4, oxalic acid. 8. Carbon, 75 ; hydrogen, 25. Ans. CH^, marsh g?is. 9. Carbon, 12: calcium, 40; oxygen 48. Ans. CaOOg, calcic car- bonate. i I tf'" 58 PRACTICAL CIIEMISTKY. § 58.— Percentage Composition. Sometimes wc are given the formula of a substance and are asked to calculate the percentage composition. This is easily done. Proceed as follows : — I Find the molecular weight of the compound by- taking the sum of the atomic weights of its constituents. 2. Then apply the " Rule of Three," or the " Unitary Method " as in the following example. Calculate the percentage composition of sulphate of copper, Cu SO 4. Copper C3'5 Sulphur 32 Oxygen (IG x 4) 64 159 5 If, by weight in 150^ parts of sulphate of copper, there are 03^ parts by weight of copper, how many parts by weight of copper will there be in 100 of sulphate. Suljthato 159^ yield 63^ of copper. .-. 1 will yield -^i ^ 159i 63-5 100 and .-. 100 " — — x = 39*81 per cent. 159.6 I ^ The percentage of sulphur and oxygen in this compound may be found in the same way. EXERCISE. What is the percentage composition of each of the following named substances : 1. Arsenious oxide, ASoOj. Aks. 75*75 arsenic ; 24*2r> oxygen. 2. Chloride of gold, AuClg. Ans. .35-09 gold; 64-91 chlorine. 3. Arseniuretted hydrogen, AsH., Ans. 96* 15 arsenic ; 3 '85 hydrogen. 4. Potassium ferrocyanide, K^ FeCoNg. Ans. 42-39 potassium : 15-22 iron ; 17-56 carbon ; 22 83 nitrogen. 5. Magnesium sulphate MgSO^ Ans. 20 magnesium; 26*67 sul- phur ; 53 "33 oxygen. ESBBBH&'N lined jgeu. Irine. lum • sul- RATIONAL rORMULuE. § 59.— Rational Formulse. 59 !l A rational formula besides exprcssiriLj the proportions by weight in which the elements are united, expresses also the way in which the elements are supposed to be grouped within the molecule of a compound. P'or example, HO ) HO r ^^3 '^^ ^^^^ rational formula for sulphuric acid. § 60.— Atomicity or Valence. We have already seen that the elements are divided into two classes — Metals and Non-Metals. There is another classification of the elements that is even mo/e important than the foregoing. The basis of this second classification is the number of atoms of hydrogen or chlorine that will unite with one atom of any of the oti r elements. According to this principle all the elements Tiay be divided into six classes. To the first class wil; belong all those which unite atom for atom with hydrof; n. Such elements bear the name of Monads. To t' j second class will belong all elements one atom of \' nich will unite with two of hydrogen. Such elements are called Dyads. If elements unite with three, four, rive, or six atoms of hydrogen they are termed, Tkiads, TETRADS, PENTADS, and Hexads, respectively. The above statement is true in a general way if the union of hydrogen or chlorine with other elements takes place by volume instead of by atoms. The following table gives this classification in detail so far as the most important elements are concerned. It will be found useful to commit it to memory : — \ lb I ,, • \ i I;! i-ii I ! I J 60 IMIA(!TICAL CHKMISTKY. MoNAUtS. DVAUH, Thiadm. Tktkah.m. Pknt.mm. llKXAb4. < H nromine. Oxygen. lioron. Nitiogon. Suljiliur. Clilorini'. Fluorine. Carlxm. Silicon. IMioaphoruM. &5 Hydrogen. Iodine. • PotaHsiuni. Calcium. IliHmuUi. AluminiiMi. Araenic. Chromlmii Sodium. Copper. Gold. Cobalt. Antimony. 2 Silver. Magneuium. Iron Rismutli. Mercury. Lead. 5^ t 1 Strontium. Zinc. M.inganesc. Niclcel. Platinum. Tin. Severcil of these elements exhibit varyin,. if3, As.O^, A8,0,. § 03.— Equations. A chemical equation consists of signs and formulae, and expresses the fact that certain substances do, of them- selves, or by means of some force applied to them, decompose and ro-arran^^'e their atoms so as to form other substances. For example the chemical equation — H2 + O = H,0 2+16 18 expresses the fact that 2 centigrams of hydrogen unite with 16 centigrams of oxygen and form 18 centigrams of water. Also, that 2 volumes of hydrogen unite with I volume of oxygen and form 2 volumes of steam or v/ater gas. In the same way, the equation CaOOg + 2 HCl = CaClo 100 + 73 i2 + H2O + CO2 111 + 18 + 44 may be thus translated : mix lOO grams (or ounces) of marble with a solution of 73 grams of hydrochloric acid SIONS. G3 iuul they will yield ill ^^raiiis of calcic cliK)ri(le, iS j^rams of water, t nd 44 of carbonic anhyilride. The atoms on one side of an ciiuation nuist all be accounted for on the other. The chemical e(iuation thoroughly understood, enables us to calculate the ainount of material required to produce a ^Mven weight of any substance ; or, the quantity of the substance pro- duced by the decomposition of a known wcij^ht of the material. Its importance, therefore, in working out chemical problems cannot be over-estimated. lite ims 'ith or I) of Lcid § 64.— Signs. The sigjn -h, J^lus^ placed between the formula: of two substances, means that the two substances are mixed together. The sign =, in chemistry, means "yields." Chemical union, as has already been explained, may take i)lace in certain porportions by WcijlU, or when the substances exist in the gaseous condition in certain proportions by Volaine ; and in both cases, the reactions taking i)lace may be represented by an equation. The first equation may be reversed in order, thus : — H2O = O + H2, and now the meaning will be this : — 18 centigrams of the compound substance, water, may be decomposed into two elementary ones, viz., 16 centigrams of oxygen, and 2 centigrams of hydrogen. QUESTIONS. 1. Complete the following cliemical equations and exjilain their meaning. They represent, in part, the reactions that took plact) in all the experiments (lescril)eil in the preceding part of this hook. 'I'hree are completed as examples : the student should try to complete the rest, and in doing so, should apply the i»rinciple of atomicity. {l.)HgO-Hg4-0. (2.) rhjO^ — SPbO + O. ■WWiX^W ;r, '.■::/: 64 PRACTICAL CHEMISTRY. (3. (4. (5. (6. (7. (8. (0. (10. (11. (12. (13. (14. (15. (16. (17. (18. (19. (20. (21. (22. KClOa + MnO., =^ KOI + MnO.j -f 30. (j -f 20 = 3Fe + 4(J = S 4- 20 = 2P 4- 50 = Zn + = • SOj, + II^O ^ P^O, + H,0 = N,03 -f 11,0 = 2K -f O = K2O 4* H2O == Mg + O - MgO + H,,0 = H2SO, -\-Zn^ 2HC1 4- Zn = 3Fe + 4H20--= CuO + 2H = H^O 4- K :=- H2O + Na — ■.- "i.-: v-s- *■ § 05.— Molecular Equations. If it be true Ihat atoms do not exist alone, "perfect consistency \vOuld require that no equation should ever be written in such a nrianner as to represent less than a single molecule of an element in a free state as either entering into 01 issuing from a chemical reaction. Thus instead of 2H + O -■- H2O, we should write 2H2 4- O3 = 2H2O. We do not propose to conform to this theoretical rule for three reasons. First, because many equations, representing chemical reactions, must be multiplied by two, in order to bring them into con- formity with tlie hypothesis conjerning molecular struc- ture ; the equations are thus rendered unduly complex ; secondly, because, in undertak'^g to make chemical equations express the molecular constitution of elements and compounds, as well as the equality of the atomic weights on each side of the sign of equality; there is \-m CARBON. 66 *' perfect onld ever ss than a as either reaction, uld write conform because ons, must into con- lar struc- complex ; chemical f elements lie atomic y, there is imm.nent danger of taking the student away from the sure ground of fact and experimental demonstration into an uncertain region of hypothesis based only on definitions and analogies ; thirdly, because we are ignorant of even the probable molecular symbol of most of the elements. Of all the elementary substances recognized, we have reason to believe that eleven, when in the gaseous state, are made up of molecules each con- taining two atoms ; that two (arsenic and phosphorus) contain four atoms ; and that three (mercury, cadmium, and probably zinc) contain only a single atom to the molecule. Of the molecular structure of the remaining elements, numbering three-fourths of the whole, we at present know nothing." CHAPTER XI. § 6G.— Oarbon.-O, A't W't, 12. We have learnt that neither air nor water is a simple body. We look around us, and seeing many common substances, such as wood, sugar, meat, and bread, we are naturally led to inquire whether these are simple or compound. It getting an answer, we shall be intro- duced to one of the most important of elements, and one which is extensively distributed throughout organic compounds. Experiments. I. Partly fill a narrow test-tube with white paper, dry saw-dust, or pieces of dry wood. Hold the tube in a nearly horizontal position with its lower end in a lamp flame. As the heating goes on, notice whether any odor is evolved. Place separate pieces of blue and red litmus paper within the mouth of the tube. When nearly all action has ceased, turn out and examine what remains in the test-tube. 6 ^mssBsimsm HI 66 rilACTICAL (MIIC.MISTIIY. 2. Place a little of the residue on platinum foil, or on a sheet of mica, and heat over a lamp flame for some time. Observe what is left. 3. Clean as well as vou can the test-tube used in the preceding experiment, and then repeat it, using a piece of woolen cloth, silk cloth, or dry lean meat. 4. Heat on a sheet of mica the residue obtained in experiment 3. 5. Heat some sugar upon a sheet of mica § G 7.— Properties of Carbon. The following experiments illustrate the principal pro- perties of carbon, so far as these can be illustrated by- simple experiments. Experiments- 1. Place a wet niter paper inside a funnel, and then cover the inside of the filter paper with a thick coating of animal charcoal, or bone black. Now filter through the paper a wine-glass full of ale or porter. 2. Place a piece of charcoal in a test-tube and then pour upon it a little strong sulphuric acid. Observe whether the charcoal changes in any way. Try whether an alkali will produce any change in the charcoal. 3. Wet the inside of a large test-tube with liquor ammo- niac. Now drop into the tube some wood charcoal pr<"- viously heated in a covered crucible. Cork the tube and after a few minutes remove the cork and ascertain by smelling it whether there is any amm.onia left in the test- tube. QUESTIONS 1, How is wood charcoal prepared ? Animal charcoal ? Is wood an element? (Jive reasons for your ajiswer. 2. Mention one dilUinjuco between the lifjuid produced in the destruc- tive distillation of wood *ud that obtained iu the destructive distiila tion of lean nieat. < -! ALLOTROIMC FOHMS. 67 3. WrJto out fill account of tlio properties nf e;irl)(in ns illustrated in the foregoing experiniunts. 4. Specify as many of the uses of cliarcoal as you (san think of. 0! lO- by ist- [l an Iruc- liila § 68.— Allotropic Forms. Charcoal is an impure form of carbon ; graphite (plum- bago or black-lead) is another ; but diamond is the ele- ment in an almost pure state and crystalline form. If you can procure a specimen of diamond, you should compare it with charcoal and with graphite, noting dif- ferences in (a) the mark they make on paper, (b) their color, (c) hardness, (d) specific weights, ^ 3) power of conducting heat, and (f) power of conducting electricity. (a) Pit Coal is composed of carbon in large propor- tion, oxygen in smaller quantity, hydrogen in smaller, nitrogen in still less, and a variable proportion of saline and earthy matter. It has been formed by the submer- sion of huge forests under the sea long ages ago, the wood being slowly changed into coal by the combined action of the pressure of water upon it, and of moderate heat from the interior of the earth. There are two prin- cipal varieties of pit coal, anthracite and bituminous. Anthracite coal contains about 90 per cent, of carbon. When bituminous coal is heated in closed iron cylinders free from Lir, a large quantity of gas and tar is formed, containing the oxygen, hydrogen, nitrogen, and some of the carbon of the coal ; the residue is called coke. This is how coal gas is manufactured. This process « if destru^'tive distillation may be applied to wood also, when au inflammable gaseous product will be given otif ; wood-tar, vinegar, and wood-naphtha, are the liquid products ; the black porous mass left behind is called wood charcoal. (b) Larop-black, the basis of printer's ink, is another form of carbon. (c') Animal Oharcoal or ivory black is made by heat- ing tne bones and flesh of animals in iron retorts. I it 68 PRACTICAL CIl KM ISTK Y. The word Allotropism is used to express ''le fact that some elements exist in very unlike states or with very different properties, but all the while oreserve their fundamental chemical identity. PROBLEMS. 1. Ascertain what effect animal charcoal has upon solutions of (a) litnins, f/>^ indigo, ^r^ potassium permanganate. 2. From a sample of coart j brown sugar prepare some that will be pure and white. 3. Devise an experiment to ascertain how long charcoal will retain its decolnrizino; and dcodorizinrj power "whpn in use as a deodorant. 4. Using apparatus similar to that in Fig. 11, devise an experiment to show that when air or oxygen is passed over ^old charcoal, no change takes place, but when, over burniug charcoal a change does take place. To show this change, pass the air or gas from the heated charcoal through clear limn-water. CHAPTER XII. § 69.— Carbon Dioxide. We have been hitherto chiefly engaged in eliminating oxygen, nitrogen, hydrogen, and carbon from compounds which contain them, and in studying their prominent properties. We must now prepare and study some of the compounds formed by the union of these elements. Carbon dioxide ( Carbonic A nhydride, carbonic acid gas ^ choke damp) ; Formula C O^ ; molecular iveight, ^^ ; specific weight, 22. 11.2 litres iveigh 22 grams. Experiments. I. Take the apparatus used in preparing hydrogen (Fig. 19), and in it place some powdered limestone or white marble. Then cover the marble with water, and pour down the thistle-tube some hydrochloric acid. Collect, over the pneumatic trough, two or three beakers full of the gas that comes off, and perform the following experiments : DECOMPOSITION OF CAUBON D]OXIDL\ 60 fact ,vith heir nil be retain it. riment change I place, harcoal nating )Ounds Tiinent )me of ments. id gas, \t. 4-4- ' drogen :one or er, and acid. )eaker3 lllowing a. Remove one of the boakers in the usual way and place it mouth upward on the table. Slip to one side the glass cover and insert a burning taper in the gas. b. Remove the second beaker from the trough. Pouf a \\\X\ litres. What weight of water will bo formed in hurning the gaa in air? What volumo of uir will bo retiuired for tho coinbuHtiou, assiinii!?^ that t)xygeii forma \ of the volume of air ? 0. How many pounds of potassic chlorate must be taken to obtain 144 lbs. of oxygon ? 10. I want 220 grams of oxygeji. If I obtain it from i)otaHsie chlorate, how much of it must I use ? If from water, how much '{ If from mer- curic oxide, how much ? 11. A gas bag is capable of containing 56 litres, how much potassic chlorate must bo taken to procure enough oxygen to iill it at 35^C. and 750 nun. ? 12. 25 litres of oxygen are exploded with 30 of hydrogen. What volume of gas (if any) renuiins V Wluvt volume of steam is produced? And what is its weight ? 13. How much oxygon can bo obtained from 4.35 grams of manganese dioxide by heating it to a redlieat? What volume will it occupy at 30 "C. and 780 mm. ? 14. What volumo will 80 gramg of oxygen occupy at the standard temperature and pressure ? II. 1. How mucli i)otassium will bo required to decompose 1 10 grams of carbon dioxide ? 2. If 10 litres of earlxm dioxide l)e passed ever red hot charcoal, what gas, and how many litres of it, will be formed at 30° ? What weight of it? 3. 20 litres of carbonic oxide are burned in oxygen gas. What gas is produced, wluit volume at 40''C. and weight of it ? 4. How mucli carbon can be obtained from 264 grams of carbon dioxide ? 5. What volume of oxygen at 10°C. is required to burn 66 grams of carbon ? 6. In question 5, what volume of air would be needed for its com- bustion at 0"C. ? 7. What volume do 110 grams of carbon dioxide occupy at 760 m. pressure and 0°C. ? 8. What volume do 1 10 grams of carbonic oxide occupy at standard temperature and pressure ? 9. What weight of carbon dioxide can be obtained from 250 grains of pure limestone by treating with hydric chloride ? 10. What weight of carbonate of lime and hydric chloride must be decomposed to produce 352 grams of carbon dioxide ? 11. Wliat volumo will 08 grams of carbonic oxide occupy at 720 m. pressure and 40" C. ? 12. If 270 gram? of oxalic acid be decomposed by sulphuric acid, find the voluiuo of the gases produced at 750 m. pressure and 20' (!. IMAGE EVALUATION TEST TARGET (MT-3) W^O {■/ >. V '^s ^ ''i^ y. ^ r/j {/. 1.0 I.I 1.25 IIIIIM ||i||M 2.0 1.4 .8 1.6 Vi m :^ /. .^3 <<> O 23 WEST MAIN STREET WEBSTER, NY. 14580 (716) 872-4503 ^tx%> 76 PRACTICAL CHEMISTRY. U^ ; e CHAPTER XV. § 77.— Hydrocarbons. A large number of compounds of carbon and hydrogen are known under the general name of hydrocarbons. So numerous are these compounds, and those which carbon forms with oxygen, nitrogen, sulphur, and phos- phorus, that their mere names would fill a small volume. The study of the carbcHi compounds forms a distinct branch of chemistry under t'le name of organic chemistry. Formerly this name included the study of those com- pounds which, it was supposed, were formed only by the agency of life ; but it was soon found that there was no essential difference between chemical substances whether of animal, vegetable, or mineral origin. The division of chemistry, therefore, into organic and in- organic is a pure matter of convenience. The last named division of the science treats of the composition and properties of air, earth and water. Marsh gas is the first of a series of hydrocarbons known as the marsh gas series. Each member of it differ^ from the following one by CH3. There is also a diffeience of 30° between the boiling points of successive members. All are inflammable. There is also a regular increase or decrease of other physical properties. Such series are called homologous series. The general alge- braic formula for the series is Cn H2n + 2, § 78.— Methane. Methane {Marsh Gas, LigJit carburetted Jiydrogen, " Fire-damp "), CH ^^ ; molecular zveight, 1 6 ; specific zueight, 8. 112 litres zveigh 8 gravis. Experiments. I. Take a hard glass test-tube or Florence flask, and fit with a cork and fine delivery tube. Place in the test- SOURCES. 77 tube 2 grams of acetate of sodium NaCgMaOa, 8 grams of sodium hydroxide and 2 grams of finely powdered quicklime CaO. Heat. After collecting a beaker or two of the gas, light the jet and observe the color of the flame. 2. Fill a small soda water bottle with a mixture of one part of the gas and two parts of oxygen. Ignite the mixture. Express the reaction by an equation. 3. Take a stoppered bottle and fill it with a mixture of equal volumes of marsh gas and chlorine. Expose to sunlight for a day, then test the contents with blue litmus. Note any change in color. QUESTIONS AND PBOBLEMS. 1 . Devise an experiment to ascertain whether marsh gas 4S acid Of basic in reaction. 2. Devise simple experiments to prove that the gas contains carbon and hydrogen as constituent elements . 3. Prove that the gas is lighter than air. Hoav would you distinguish the gas from air ? 4. Find out whether the gas is soluble in water. § 79.— Sources. f sulphur is illustrated in the first expe- riment? Where does the sulphur collect? 2. The residue from the iron pyrites has the composition Fe^Sj. Write the equation expressing the reaction that took place. 3. How long does plastic sulphur retain its plastic property ? 4. Prepare some sulphur crystals by melting 40 or 50 grams of flowers of sulphur in a porcelain or earthenware dish, and then allowing it to cool slowly. As cooling goes on, break the crust which forms, and pour out the liquid sulphur. 5. Try to prepare some more crystals by dissolving sulphur in carbon disulpliide and allowing the solution to evaporate. § 90.— Sulphur Dioxide. Sulphur and oxygen unite to form four compounds, but of these only two are important. Sjilphur dioxide (Sulphurous Anhydride) : For mu lay molecular weighty 6^ ; specific weighty J2. SO 3 > Experiments. 1. Heat a little sulphur on a piece of tin or zinc plate until it catches fire. Note the color of the flame and the smell emitted. 2. Put 20 grams of copper turnings into a Florence flask, fitted up like a hydrogen generating apparatus. Just cover the copper with strong sulphuric acid, and heat the flask very carefully. Collect the gas that escapes, by upward displacement of air. Compare its odor with that of burning sulphur. Collect three jars of the gas for further experiments. Keep them covered. (a). Pour some water into one of the jars and then shake. Is the gas soluble in water? Test the water with blue litmus solution. Taste it. BLEACHING KXPLAINKD. 85 (b). Hang a red rose or other high-coloicd (lower in the second jar. If any change takes place in the flower, remove it and place in pure air. (c). Pour the third jar of sulphur dioxide into "empty" jar with a burning candle at its bottom. m QUESTIONS AND PROBLEMS. 1. Devise a means of collecting sulphur dioxide \vht;u formed by burning sulphur in the air or ir. oxygen 2. How does the weight of this gas compare with that of air 'l 3. Try to liquefy this gas by passing it into a cool test- tube 4. Explain why the fire in a chimney may be extinguished by burning gome sulphur in a stove connected with the chimney. 5. Suspend a moistened wheat straw in a jar of this gas, and observe what Occurs. § 91.- Bleaching Explained. The bleaching power of sulphur dioxide is supposed to be due to its affinity for oxygen. According to this theory the dioxide unites with the oxygen of the water which is used to moisten the article to be bleached ; the hydrogen of the water then combines with the coloring matter and forms colorless compounds. % 'I § 92.— Other Properties. Sulphur dioxide dissolves in water at 0°C to the extent of nearly 8o times its own volume ; miiy be con- densed to a colorless liquid of sp. weight of 221. It is also a good disinfectant in case of contagious diseases like scarlet fever, &c. ; it prevents the decay of animal and vegetable substances that is, it is an antiseptic. § 93*.— Sulphurous Acid. We have already seen that the sulphur dioxide dis^ so'ves readily in water, forming with it a new compound I- is ' If. ^'# \ ill 86 niACmCAL ClIKMISTRY. which has a sour taste and turns bkic litmus red. This new compound is called sulphurous acid. The reaction that takes place may be symbolized as follows : — SO, + 1I2O = USA. Hulphuroiis ucid. An anhydride is an oxide caoable of forming an acid when united with water. § 94.— Sulphur Trioxide, SX),. Unlike the preceding one, this oxide is of no practical importance. It may be prepared by passing sulphur dioxide and oxygen over platinised asbestos in a highly heated porcelain tube. When this is done, there i.ssues from the end of the tube hot dense white fumes of sul- phur trioxide. This substance is a solid, having an in- tense affinity for water and being in consequence very unstable. The reaction is : — SO3 + H.O = H2SO4. Sulphuric acid. § 95.— Sulphuric Acid. This is the strongest of all acids and may appropri- ately be spoken of as the " king" of acids. It is used very extensively, either directly or indirectly, in all arts and trades, and is the most important re-agent which the chemist possesses. Its compounds include many of the most important substances in commerce. Experiments. Sulphurous anhydride, steam, air, and nitric oxide, are passed into an immense chamber. "'The nitric oxide in presence of oxygen, immediately becomes nitrogen per- oxide, and this, when mixed with sulphurous anhydride sui.riiUHio A(;ii). 87 and a large quantity of water, furnishes sulphuric acid and nitric oxide. The sulphuric acid remains dissolved in the water, while the nitric oxide, by absorbing oxy- gen from the air, again becomes nitrogen peroxide ; this combines with fresh sulphurous anhydride, which, when acted on by water, becomes sulphuric acid, the nitric oxide being again liberated, to go through the same series of changes with fresh portions of oxygen and sul- phurous anhydride as long as any remain in presence of each other uncombined. NO3 + SO2 + a;H,0 = NO + (a;-l) HgO + HjSO^." The process of manufacturing sulphuric acid may be illustrated by the use of the apparatus shewn in Fig. 35. Fia. 35. j4 is a, flask containing copper filings and nitric acid for generating nitrogen dioxide and nitrogen trioxide. J? is a flask containing water for generating steam. Cis a flask generating sulphurous anhydride. Z> is a tube through which air is forced into the condenser F. E is an escape pipe for waste gases. It should be connected with a good ventilating shaft. i^'is a glass globe used as a condensing chandler. Sulphuric acid collects at the bottom of the globe. Sulphuric Acid: Formula, H ^SO ^ ; molecular iveip-Jit g8 ; specific gravity of liquid^ i-6\f6. 88 PRACTICAL CHEMIST KY. : ■ ii 1 I, B i|i m tii Experiments. 1. Pour some commercial sulphuric acid into twenty or thirty times its own volume of water in a test-tube. Taste the solution. 2. Half-fill a small test-tube with blue litmus solution and add to it some of the diluted sulphuric acid. Add some of the acid to some red litmus solution also. 3. Place some " bread soda " in a test-tube and pour some of the dilute acid upon it. 4. Observe the color of some chemically pure sul- phuric acid. Compare its weight with that of water, taking equal volumes of each. 5. Make a mixture of equal volumes of water and acid, and immerse in the mixture a glass mercurial ther- mometer, or a small test-tube filled with spirit, to note the change in temperature. 6. Pour some s'-ong acid on a little sugar. What element is shown to exist in the sugar .? 7. Shake up some acid and oil in a test-tube. What action has the acid on the oil ? § 96.— SolubiUty of Salts. The sulphates are all soluble in water, except those of lead, barium, a:id strontium. Tiie sulphate of calci- um is slightly soluble. 11 § 97.— Test. The presence of any .soluble sulphate may be detected by the addition of a few drops of a solution of barium nitrate or barium chloride. The color of the precipitate formed, accompanied by the fact that it is insoluble in any acid you may chose to add, is an invariable test for the acid or its salts. SULPHURETTED HYDROGEN. § 98.— Sulphuretted Hydrogen, SHa- 89 This compound of sulphur and hydrogen is of great importance in chemical analysis. It is usually prepared by the action of an acid upon a sulphide. Experiments. ' 1. Take a hydrogen generating apparatus, and place in it some powdered sulphide of iron. Cover the sul- phide with water, cork the apparatus tightly, and then add a few drops of sulphuric acid. Collect two or three bottles of the gas over warm water. Don't allow much of the gas to escape into the room. It is poisonous. 2. Bubble some of the gas through a solution of blue litmus. 5. Attach a piece of glass tubing drawn to a fine point to the generating apparatus and try whether the gas will burn. 4. Devise an experiment to ascertain whether the gas is soluble in water. 5. Fill a bottle v^ith chlorine gas and another with sul- phuretted hydrogen. Invert the mouth of the chlorine bottle over that of the other and observe what takes place. Smell the gas that forms in the bottles. Infer the reaction, and write the equation. § 99.— Sulphides. A solution of sulphuretted hydrogen in water is much used in mineral analysis, inasmuch as the insoluble sul- phides which it forms with soluble salts of many of the metals have characteristic colors that enable the analyst to recognize the presence of particular metals. Experiments I. Polish a five or ten cent piece of silver, and then place a drop of sulphuretted hydrogen water upon it. w 90 PRACTICAL CHKMISTIIY. 1'^ 2. PiLpare in separate test-tubes solutions of lead nitrate or acetate, copper sulphate, zin^ sulphate, and ferrous sulphate. Add a few drops of hydrochloric acid to each, and then pass into each solution some gas from the generating apparatus. Take care to wash the deli- very tube before passing it from one test-tube to the other. Tabulate your results, and memorize them. Try to write the equations which symbolize the reactions. Pill § 100.— Test. Experiments. 1. Moisten some paper in a solution of acetate of lead, and then bring it into contact with the gas ? 2. If the gas comes off in quantity, its smell is suffi- ciently characteristic to enable anyone to recognize it. QUESTIONS. 1. Why is silver plate so easily blackened in the air of towns ? 2. Why do silver coins or watches when carried in the pocket along with matches change their color ? 3. How would you find out whether a sample of vinegar contained sulphuric acid or not ? 4. Sewer gas nearly always contains sulpuretted hydrogen. If sewer gas found its way into a house how would you disinfect it ? CHAPTER XVIII. § 101.— Acids. Chemists have found it convenient to classify a large number of chemical compounds as acids, bases, and salts. In order to learn some of the general properties of acids, repeat the first three of the experiments in section ninety-five, using any three or four substances labelled acids, which you can find upon your working table. As you make each experiment, tabulate your results as follows : — BASES. .91 Name of Acid. „, ^ i Action on TAHtc. j 1 Red Litmas. 1 Action oil Blue Litmus. Action on Bread Soda. KKMAKK8. § 102.— Bases. Experiments. I. Repeat the preceding experiments, using solutions of (a) quicklime, (d) slaked lime, (c) magnesium oxide, and (d) sodium hydroxide (caustic soda). Tabulate your results a ; before. § 103.— Hydroxides. Experiments.* I. Take a piece of the metal potassium, about the size of a pea, place it in an iron spoon, and heat it over a spirit lamp until it has ceased to burn. Then add a little water, and test the solution with red and with blue litmus, as before. 2 Repeat this experiment, using the metals, magne- sium and sodium. 3. Take a small jar and pour about J inch of water into it. In case you have no spoon with a long handle, take half of a chalk crayon, and wind a piece of wire round one end of it, and use this instead. On the top of this crayon place a small piece of phos- phorus, ignite it, and lower into the jar as in Fig. 36. Cover the mouth of the jar, and when the burning has ceased, remove the spoon or crayon, and shake up the water with the gas or solid produced. Test as before 'with the two litmus solutions. Pia. 86. 1 1 PH ill I PI •if I '..'I g» I[:.; III! 92 PRACTICAL CHEMISTRY. 4. Repeat this experiment, using separately, fragments of charcoal, and of sulphur. Tabulate your results as in previous experiments. QUESTIONS. 1. Write out au account of what you have learnt about acids and bases, and tell how you would distinguish them from each other. 2. If you know whether an element is a metal or a non-metal, can you foretell whether its oxide, when dissolved in water, will produce an acid or a base ? Explain. A hydroxide is a compound formed by the union of the radical OH with atoms of the elements, or with other radicals. The group of atoms, OH, acting as one atom, and being present in a series of chemical com- pounds is called a compound radical^ and its compounds are called hydroxides. The name given to this radical is hydroxyl. The compound of hydroxyl, OH, with potassium is potassium hydroxide KHO ; with sodium, it is sodium hydroxide NaHO. The stronger bases are known as alkalies. Hydroxides are called hydrates by some chemists. i04.~Salts. Let us now examine some of the compounds called salts. Experiments. I. Take a piece of "caustic soda" (sodic hydrate) NaHO, about the size of a pea, and dissolve it com- pletely in a test-tube of water, then add to it hydrochlo- ric acid, drop by drop, until a piece of blue litmus paper placed in the solution slowl)^ begins to turn red, pour half of this solution into an evaporating dish, place on a sand b'^th and heat until all the water is driven off. Caref .y examine the residue. Taste it. A^our the rest of the solution into a flat dish of any kind, and allow it to remain for a day or two in a warm room. NAMING ACIDS AND SALTS. 93 The residue in both cases is called a salt. 2. Perform similar experiments using potassium hydro- xide KHO (caustic potash), and nitric acid : also, sodium hydroxide and sulphuric acid. Write out, in the following tabulated form, a synopsis of the leading characteristics of acids, bases, and salts, including their taste, action on litmus, action on each other, and a method of obtaining the members of each of these three groups of compounds. 1*1 Acids. ALKALIK8. Salt.:. 1. Taste 2. Action on litmus 3. Action on each other .... 4. How obtained What you will have learned about acids bases and salts from your experiments with these compounds, will not be true as regards every member of each of these groups, but your knowledge will be sufficiently accurate for the present. § 105.— Naming Ac^ds and Salts. In naming acids the terminations -ous and ic, and the prefixes hypo- and per-, are used, e.g.: IICIO is called hypocliloroiia acid. HClO.j is called chlorous acid. HOIO3 is called chloric acid. HCIO4 is called perchloric acid. For the least amount o{ oxygen present in the above compounds, hypo ous is used ; -OUS, for more oxygen ; -ic, for still more of it ; and per ^ic, for the greatest amount. Similarly : — 1^ % ' ' ' fin \\ i Jiii : pyfi - III H PRACTICAL CHEMISTRY. HNO is called hyj)onitioua acid. HNOg is called nitrous acid. HNO3 is called nitric acid. Salts are named chiefly from the acids which form them. If the name of the acid end in -ic, that of the salt ends in -ate. If the name of the acid end in -OUS, that of the salt ends in -ite. The prefix of the name of the acid is retained in naming the salt, e.^. : Acid. Namk. t Salt. ! Name. HCIO hypochlor-ous acid. KCIO potassic HYPO-chlor-iTE. H2SO3 sulphur-ous acid. Na^SOg sodic sulph-iTE. HNO3 nitr-ic acid. AgNOg argentic nitr-AXE. HCIO, perchloric acid. KCIO, potassic PER- chlor-ATE. § i 06.— Formulas of Hydroxides. The principle of atomicity may be employed in formu- lating the theoretical oxides and hydroxides of the metals by using water as a type, and substituting in a single molecule of water one atom of a monad metal for one atom of hydrogen to form a hydroxide, and two atoms of a monad metal for the two atom's of hydrogen, to form an oxide. In two molecules of water, we must substitute one atom of a dyad metal for two atoms of hydrogen to form the hydroxide, and two atoms of the dyad metal for the four atoms of hydrogen to form the oxide. For example : Type. Hydroxide. H.,0 KHO 2 H,0 Ca(H0)2 Oxide. K2O CaO NITRIC ACID. 95 1. Apply this principle and formulate hydroxides and oxides of the folowing metals : Sodium, silver, mercury, magnesium, iron, tin, platinum. 2. Name the compounds thus formed. CHAPTER XIX. § 107.— Nitric Acid. Before studying the compounds of nitrogen and oxy- gen, we shall dwell at some length upon an acid, which besides being of great practical use, is important as form- ing a starting point from which all the oxides of nitro- gen can be derived. This acid is known as nitric acid. Its old name is aqua fortis. Nit ic Acid : Formula^ HNO^ ; molecular weight, 63 ; speci- fic weight of liquidy 1-52; boiling point, 84*5'' Freezing^ — 40°. Experiment. Put into a tubulated glass retort 30 grams of powdered nitrate of potash, KNO3, and an equal weight of strong sulphuric acid, H2SO4. Place the end of the retort in a flask which is made to float on a basin of water as in Fi& 2)7' Apply heat to the retort. Soon a yellowish coloured liquid distils over and is collected in the cool flask. The reaction may be represented as taking place in two successive stages, the first requiring a compara- tively low, the second a high temperature. {a). 2KNO3 + H2S04=HKS()4 + HNO3 + KNO3. i m iii i 11 96 PRACTICAL CHEMISTRY. On increasing the heat more acid comes off, the second reaction being represented as lollows :— (6). HKS04 + KN03 = KoS04 + HN03. Sodic nitrate, NaNOg, may be used instead of potassic nitrate in the preparation of nitric acid ; in fact, sodic nitrate is generally used when this acid is to be manu- factured on a large scale. § 108.— Properties. Having prepared a sufficient quantity of the acid in the manner described, you may proceed to study its properties as follows : — Experiments- 1. Immerse some wool, silk or other organic substan- ces in a little of the acid. 2. Add a few drops of it to a solution of indigo. 3. Place some copper filings in the bottom of a test- tube, and then pour in some of the acid. When all action has ceased, evaporate to dryness the solution formed. § 109.— Oxidizing Agent. Nitric acid is said to be a powerful oxidising agents that is, it readily yields its oxygen to substances which have an affinity for that element This oxygen, at the moment it is liberated from the acid, is said to be in its nascent state. The three following experiments illus- trate this oxidizing power of nitric acid. If you can procure some very strong acid — " fuming nitric acid " as it !s called — you can successfully perform these experi- ments, but great care must be exercised or a dangerous accident may result. OTHER ruOPERTIES. 97 Experiments. 1. Place a small piece of phosphorus in a saucer, then drop on it a little of the acid. 2. On powdered pieces of glowing charcoal, pour a little nitric acid. 3. Place a little carbolic acid in a test-tube, and then pour in a few drops of the fuming acid. §110. — Other Properties. Nitric acid when pure is colorless and has a specific weight of i'$. The commercial acid contains about 30 per cent of water, and has a specific weight of 1*4; it is also yellow in color from containing some of the oxides of nitrogen in solution. Undiluted, it is a strong irritant poison when swallowed. It is mono-basic, that is, it contains only one atom of replaceable hydrogen in its molecule. The distinctive characteristic of a monobasic acid from a practical point of view is that it forms salts which have no acid reaction. §111.— Uses. It is used extensively in dyeing and as a medicine. In chemical analysis it is a solvent of great value. The nitrates are all soluble in water. , § 112.— Sources. Nitric acid is formed in air by lightning. It exists also in nature in the form of nitrates of potash, soda and lime, which are themselves the product of decomposition and oxidation of nitrogenous organic compounds with alkalies. " These nitrates are widely diffused in all sur- face soils, especially in hot countries such as India, where 8 ''M '■ ■' ' i'i: ij ^' :!■ ,1 i.i « ■ n puac;ti(^\l ciikmisthy. oxidation takes place rapid!}-. In the neii^hborhood of Indian villages, soil which contains considerable amounts of potash, thus becomes rich in nitre, or potassium nitrate, originating from the decomposition of the urea of the urine. It is from this source that the largest quantity of nitre imported into England is obtained." §113.— Tests. We are said to tfst for an element or for a compound, when we subject it to an experiment which reveals phe- nomena unlike those produced by any other substance under examination. For example, clear lime water becomes milky when carbon dioxide is passed into it , and this fact constitutes a characteristic test for carbon dioxide. Some one or more of the experiments per- formed with oxygen, nitrogen, hydrogen, carbon, and carbon monoxide will have enabled the student to dis- tinguish these substances from each other, and from all other substances which he may hereafter meet with and which may resemble them, but to distinguish nitric acid from all other substances, we mu^t perform a special experiment. Experiment. I. Dissolve a few crystals of ferrous sulphate, FeS04, in water in a test-tube. Add a few drops of sulphuric acid and allow the whole to cool. Then turn the test- tube sideways and gently pour nitric acid or a nitrate in solution down its side. The phenomenon which results rill always enable us to recognize nitric acid or a nitrate. QUESTIONS AND PROBLEMS. 1. Describe the physical and chemical properties of nitric acid ? 2. Compare the action of nitric acid on wool and silk with its action on goose-quill clippings or on the finger nail ? 3. Read up, in some standard work on organic chemistry, methods of manufacturing oxalic acid, gun-cotton, and nitro-glycerine. 4. Describe experiments to prove that this acid is easily decomposed. OXYOKN AND NITROOKN. 90 5. Nitric acid acts on copju'r ami forms tlie Halt ctiprio nitrate Cii (NOala ; lind out wliuther it acts Hiinilarly on utliur coiumuu mctuis Buoh as lea(f, zinc, iron and mercury. 6. The principle of atomicity may bo employed in writing the formulas of salts from nitric acid, by replacing one atom of the hydrogen of the acid with one atom of a monad metal ; two atoms of the hydrogen of the acid with one atom of a dyad metal, and so on. For example : Acid. SaU. Name of Salt. HNOg AgNOg SUver Nitrate. (a) In the same way symbolize the salts which nitric acid may form with the following metals : Potassium, calcium, copper, lead. {b) Name these salts. 7. Explain the action of nitric acid on a solution of sulphate of indigo. i CHAPTER XX. § lU.— Oxygen and Nitrogen. Oxygen and nitrogen unite indirectly to form five well-known compounds. Their names and formulae may be tabulated as follows : — Formulae. Old Names. Modern Names. NaO Nitrous oxide. Nitric " Nitrogen I monoxide. NOlNaOa)....... dioxide. N2O3 Nitrous anhydride .... (( trioxide. NO2 (N2O4) Nitrogen peroxide. ... <( tetroxide. NoO« Nitric anhydride <( pentoxide. Only the first three will be considered in this book. The third and fourth are, of course, important from a theoretical point of view, but not sufficiently so to come within the scope of the present work. Nitrous Anhydride, NOz ; moleadar ii^eight^ j6 ; spe- cific weight, j8. l\ 100 PRACTICAL CIIKMIHTHV. ti i ' 4-1'} 1 * :;' 1 •'■!■ i i 1 ■! i 1 i ■'« Q; ■J ij 'V, t 1' f{' i Fio. 38. Experiment. Fit a Florence flask with a cork and delivery tube and place on a re- tort stand, as in Fig. 38. To the delivery tube at- tach a U tube immersed in a freezing mixture of salt and snow. Connect the other end of the U tube with a glass tube leading to a vessel A, containing ice-water. Place 10 grams of starch in the flask and cover with nitric acid. Gently heat the generating flask and nitrogen trioxidc will be plentifully produced, part of it being condensed in the U tube, and the remainder passing on into the ice-water. Instead of starch, white arsenic, AsaOs, may be used. The reaction in this case may be thus represented Arhrnio Acid. 2HNO3 + AS2O3 + 2H2O = 2(H3A804) + N2O3. Notice the color of the gas. It is condensed to a liquid by a temperature of — 18° C. Try to collect some of the gas over water. Has it any smell ? §115.— Nitrous Acid, HNOa. In preparing nitrogen trioxide, it is difficult to avoid preparing some nitrous acid at the same time. In fact, the ice-water in the preceding experiment already con- tains some of this acid dissolved in it. Its preparation may therefore be represented as follows : — N2O3 + H2O = 2HNO2. Experiments. I. Pass some nitrogen trioxide into a cold solution of potassic hydrate until it is neutralized. Then evaporate to dryness. Try to symbolize the reaction that occurs. The salt prepared in this experiment is known as potas- sic nitrite. OTllKIl rUOl'EttTllia AND TKHTH. 101 2. Add a few drops of nitrous acid to a solution of potassium pcrmanLjaiiate. Try to account for the phenomena whicli arise. § 116.— Other Properties and Tests. Nitrous acid is an unstable compound decomposing, upon standing, into nitric acid, nitric oxide, and water Its salts are all soluble in water. When nitrites ar^ acidulated with acetic acid they give a white precipitate with nitrate of silver. The next experiment furnishes us with a test for free nitrous acid. Experiment. Boil some starch in water so as to form a paste. Then add some iodide of potassium solution, and allow the whole to cool. The reaction, which occurs on adding free nitrous acid to the mixture forms, when taken in connection with the nitrite of silver test a sure indication of the presence of nitrous acid. QUESTIONS AND PROBLEMS. 1. Write out an account of the physical and cliemical properties, (a) of nitrous anhydride, (b) of nitrous acid. 2. Investigate some of the properties of potassic nitrite by (a) throwing some of it upon red-hot charcoal, (b) by placing a drop of any strong acid upon it. .3. Heat some nitrous acid solution, then test the residue for nitric acid. 4. Procure a nitrite and apply to it the test for nitrous acid. § 117.— Nitrogen Dioxide. In preparing nitrogen dioxide in the following experi- menr, a gas is at first formed with which the student is already acquainted, but he must not be misled by its appearance. Nitric Oxide NO (NO^) ; molecular iveiyhty specific weighty i^. 11.2 litres iveigh 75 grams. 30 1>M : if 102 PUACTICAL ClIEMISTKY. m ! 1 3 Experiment. Place some copper filings in a hydrogen generating apparatus, similar to that in Fig. 39, add some warm water, and then pour down the funnel tube some strong nitric acid. The gas that first forms should be al- lowed to escape. The reaction may be thus represented : — Fig. 39. 3Cu + 8HNO3 = 3Cu(N03)2 + 4H.,0 + 2N0. Collect over water four beakers full of the gas and perform the following experiments : — (a). Allow the contents of the first beaker to escape into the air. (I?). Ignite a piece of phosphorus very slightly and plunge it into the second beaker. (c). Allow another piece of phosphorus to burn strongly, and then place it in the third beaker. fi § 118.— Other Properties and Tests. Nitric oxide condenses to a liquid at — il°C, and a pressure of 104 atmospheres. Unlike nitrous anhy- dride, it does not unite with water to form an acid. One test for this gas is its reaction with air or free oxygen ; another is that with a solution of ferrous sulphate, as seen in the next experiment. Experiment. Pour a solution of ferrous sulphate, FeSOi, into the fourth beaker full of the gas, Then hold the hand over the beaker's mouth and shake vigorously. Note the two phenomena that occur. NITROGEN MONOXIDE. QUESTIONS AND PROBLEMS. 103 *. Name the reddish orange gas that forms at first in the flask in preparing nitric oxide ? Name three gases that are mixed in the flask. 2. Try whether you can prepare this gas by using other metals than copper. 3. What gas is formed when it escapes into air? [Nitrogen peroxide is formed as well as the gas, whose name you are to- give]. 4. Explain the differeuce between the tesulta of experiments (h) and (c). 5. Is nitric oxide soluble to any great extent in water ? Give a rea- son for your answer. 6. Investigate the proportions by volume in which oxygen and nitro- gen are in this gas. To do so, fill with mercury a test-tube bent at an obtuse angle and invert it over another vessel containing mercury. Pass into the test-tube some of the gas. Then take a small piece of potassium or sodium and place it under the mouth of the test-tube. When the sodium rises to the top of the mercury in che tube, jerk the tube gently to one side so that the sodium is thrown on to its bent part ; then heat the test-tube immediately below the sodium until it burns. After the tube has thoroughly cooled, note the change in vohime. 7. The specific weight of nitric oxide is found by actually weighing it to be 15, Should its formula be NO, or N^Oo ? § 119.— Nitrogen Monoxide. " Laughing gas " is an old name for this compound. It derives this name from the fact that many persons afterinhaUnga mixture of the gas and air are compelled to laugh — nolens volens. On inhaling more of the gas, temporary unconsciousness is produced ; it is therefore frequently used as an anaesthetic for minor operations in surgery. Nitrons Oxide : formula^ N^O ; molecular zveight, ^^ ; specific weight, 22. 11.2 litres weigh 22 grams. jcixperiment. Put 25 grams of commercial ammonic nitrate, NH^- NO3, into an oxygen generating apparatus, connected with three bottles, as in Fig. 40. The first bottle should contain a solution of ferrous sulphate, the second, a ^\H u r i Wi « ;.;!!! i ' bT - '3 > 'I 104 PUACTICAL CHEMISTRY. solution of caustic potash, and the third, water. Heat — the nitrate gently and liitrogen monoxide ^ will be given off. Thus prepared, the gas will be found mixed with nitrogen dioxide, and t chlorine gas. The first will be re- moved by passing through the ferrous sulphate solu- tion, and the second bypas- sing through the caustic potash solu- tion. Fio. 4a If the nitrate be chemically pure, the wash bottles may be omitted. In this case the reaction may be thus represented : — NH,N03 = 2H20 + N,0 Collect four jars of the gas over warm water and perform the following experiments : — (a). Plunge a lighted taper into the first jar. (d). Burn a piece of phosphorus, or carbon, or sulphur in the second jar. If burning strongly at first, they continue to burn. Write the equation expressing the reaction. (c). Explode a mixture of the gas with hydrogen. Write the equation. (6/). Place the fourth jar, mouth downward, over co/d. water, and then shake. § 120.— Other Properties. Nitrous oxide is soluble in cold water to the extent of 130 per cent, of its own volume; it may be condensed to a liquid by a cold of o"C, and a pressure of 30 atmospheres. Liquid nf'/ous oxide when mixed with carbon disulphide, CSj, forms a freezing mixture capable of producing a cold of — I40°C. NITROGEN ACIDS. 105 § 121.— Nitrogen Acids. It is interesting to note how hyponitrous, nitrous, and nitric, acids may be supposed to be formed from nitrous oxide, nitrous anhydride, and nitric anhydride respec- tively. Thus : — N2O -t-HjO = 2HN0, Hyponitrous acid. N2O3+H2O - 2HNO2, Nitrous acid. NaOg+HaO = 2HNO3, Nitric acid. The first of these has never been prepared in a free state, but its salts are known. QUESTIONS AND PROBLEMS. 1 . Has nitrous oxide any taste or smell ? How can it l)e distinguished from oxygen ? 2. Devise an experiment to ascertain whether it is heavier or lighter than air ? 3. Pass soma nitric oxide into air, and also into nitrous oxide. Explain the difference in the phenomena observed ? 4. Ascertain the composition of nitrous oxide by volume. To do this, make an experiment similar to that by which you found out the com- position by volume of nitric oxide. f if I CHAPTER XXL § 122.— Nitrogen and Hydrogen. There is but one known compound of nitrogen and hydrogen. This compound is called ammonia. Ammonia: (Spirits of Hartshorn) Formula, NHz; atomic weight, ly ; specific zveight^ S.J. 11. 2 litres weigh 8' J gratns. Experiments- I. Mix in a mortar 30 centigrams of fine iron filings, with 2 centigrams of solid caustic potash. Then trans- fer the mixture to a test-tube fitted with cork and delivery tube, and heat until gas escapes. Collect some of this gas and test for hydrogen. 106 PRACTICAL CriKMISffRY. in ii 2. Repeat the experiment, substituting 2 centigrams of nitre for the 2 of potash. In this case test for nitrogen. 3. Now mix 30 centigrams of iron filings, 2 of caustic potash and 2 of nitre, place in a test-tube and heat as before. Smell the gas that is evolved ; it is ammonia. 4. Make a mixture of nitrogen and hydrogen, taking these gases from gas-holders or bottles in which they have previously been stored. After the mixture has stQod for some time, smell it. Does it smell like ammonia.-* QUESTIONS AND PROBLEMS. 1. Describe briefly the physical properties of ammonia. 2. What is the relation of experiment 3, to experiments 1 and 2 ? 3. Try to account for the fact that ammonia has formed in the third experiment and not in the fourth. § 123.— Nascent State. An element is said to be in its NASCENT state at the moment it is liberated from a compound. In this state its properties are more active than after it has been formed for some time. This greater energy is explained on the theory that elements, at the instant of their elimination from a compound, exist in the form of atoms and not of molecules. Soon after being eliminated, molecules are formed ; and then the element loses its power of readily entering into union with other elements. Apply this theory of the nascent condition of elements to explain the relation to each other of the results of the four experiments in the precedin'g section. § 124.— Properties. Experiments I. Take about 20 grams of dry ammonic chloride and an equal quantity of dry quick-lime ; powder them 6nely PROPERTIES. 107 Fio. 41. in a mortar. Smell the mixture, and then transfer it to a flask with tightly- fitting cork and long tube bent up- wards. Heat gently. Hold a large test-tube over the delivery tube, and fill it with gas by downward dis- placement of air, as in Fig. 41. 2. Pass a lighted taper up into the test-tube full of gas. 3. Pass some of the gas into red- dened litmus. Upon the result of this, devise a means of knowing when a bottle is full of this gas. 4. Moisten a glass rod with hydrochloric acid, and then bring it near the end of the delivery tube. Do the same with other acids. 5. Fill a narrow-necked bottle with the gas, by dis- placement of air as before, and then place the bottle mouth downward in water. Shake the bottle. 6. Pass the gas for some time into a long test-tube of ice cold water. Note any changes in temperature and volume of the water. QUESTIONS AND PROBLEMS. 1. Describe the chemical properties of ammonia. 2. How can you tell when a jar is full of this gas, if you collect by displacement of air ? 3. How do you know that ammonia is a base? What compound did it form with hydrochloric acid ? In what physical state did this compound exist ? 4. What became of the gas in experiment 5 ? How does the solu- bility of ammonia in water compare with that of oxygen ? Of nitro- gen ? Of hydrogen ? 5. Aqua ammonia is simply water with ammonia dissolved in it. Knowing this, try to devise a means of obtaining ammonia from aqua ammonia, and prepare some of the gas from this source. 6. Devise an experiment to ascertain whether ammonia is absorbed by freshly burned charcoal. 7. Powder some coal coarsely in a mortar. Then place in a hard glass tube an I haat. Smell the gas that comes off. Is the liquid that forms acid or alkaline ? i^H- 108 PRACTICAL CHElk ISTRY. ^^BS . 1 1 PR li 5. PS liil ■i'l 8. Ascertain, in a similar manner, whether animal matters, such as horn, dried flesh, glue, cheese, &c., will yield ammonia on being heated in a test-tube. § 125.— Ammonium Hydroxide. Ammonium hydoxide is probably contained in ligjior ammonicB or aqua ammoni(e. Its formula is NH*OH. For a description of the process of manufacturing liquor ammoniae on a large scale, you must consult some treatise on chemistry. All that need be said here is that aqua ammonia is a by product from the manufacture of coal gas. Experiments- 1. Pour a concentrated solution of ammonia into a flask. Through this solution pass a current of oxy- gen from a gas holder and tube, as in Fig. 42. Apply a lighted match to the mouth of the flask. 2. Heat to redness a spiral coil of platinum wire, and then place it quickly in the mouth of a bottle containing aqua ammonia. QUESTIONS. 1. What was the color of the flame produced in experiment 1 ? Try to write the equation expressing the reaction. 2. Try whether a jet of pure dry ammonia will burn. 3. Why does the platinum wire continue to glow in experiment 2 ? Whence comes the heat ? 4. Point out any resemblance you can see between the results of experiments 1 and 2. 5. Try whether you can expel all the gas from liquor mnmomcB by boiling it. Test the water afterwards with some of Ncssler^s solution (see test for ammonia). Fia. 42. § 126.— Analysis of Ammonia. Experiment. Take a eudiometer, fill with mercury and invert over a small trough or saucer, also containing mercury. Heat AMMONIO CHLORIDE. 109 some ammonium hydrate and pass 20 c. cs. of the gas into the eudiometer. Then pass a series of electric sparks from an induction coil through the gas, taking care to insert a Leyden jar in the circuit, so as to increase the heating effect. When the gas no longer expands, pass 29 c. cs. of pure oxygen into the eudiome- ter and explode. Depress the eudiometer so as to bring the mercury to the same level inside the eudiometer as the outside, and then note the volume of the gas remain- ing in it, QUESTIONS. 1. How much did the gas increase in volume by passing electricity through it? 2. How many c. cs. were there altogether in the eudiometer after passing in the oxygen ? How many after the explosion ? 3. Explain tlie cause of the reduction in volume ? 4. What must be the composition of the remaining gases ? What volume of each ? 5. vVhat then are the constituents of ammonia by volume ? 6. How many atoms of each element must there be in the molecule of ammonia ? What therefore must be its formula ? If! f T. § 127 — Ammonic Chloride. You have already learned that ammonia is a base, and in your experiments with acids and bases you formed salts by neutralizing one of these compounds with the other. You might, therefore, fairly infer that a salt was formed when ammonia was brought into contact with hydrochloric or other acid. The salts thus formed are called ammonium salts, because they contain the radical NH4 — ammonium. Experiments. I. Pour about 10 c. c§. of aqua ammon.je ,nt© an evap- orator, and add hydrochloric acid until the solution is neutral. Then gently evaporate to dryness. The sub- stance obtained is called ammonic chloride. 110 r K A( !T I C AL II KM I ST K Y. 2. Place a small piece of the salt obtained in the pre- ceding experiment upon a strip of platinum foil and heat for some time. 3. Prepare a solution of ammonic chloride with the remaining part of the salt ; place in a test-tube, and add caustic soda or caustic potash, and heat. Smell. m if;' QUESTIONS AND PROBLEMS. 1. Prepare ammonic chloride by using equal volumes of the two gases — ammonia and hydrocldoric acid. 2. Prepare other salts of ammonia by using other acids than hydro- chloric. 3. How was ammonia obtained from the ammonic chloride ? Draw a conclusion as to the general manner in which ammonia may be obtained by the decomposition of ammonic chloride. 4. Ascertain whether ammonia can be obtained from other salts of ammonia beside the chloride. To do this, heat solutions of these salts, in a test-tube, with solutions of eitiier caustic soda, caustic potash, or quicklime. At intervals, waft with the hand the vapour coming from the test-tube towards the face, no as to smell it. Warm a glass rod slightly, dip it into hydrochloric acid, and bring it over the mouth of the test-tube. Notice whether any phenomenon results. 5. Try to represent by equations the re-actions taking place in x>re- paring the salts of ammonia. Thus : — NH4HO -h HNOg = 2NH4HO + H2SO4 = 6. Also, the reactions when these salts are decomposed by heating with caustic soda or caustic potash. Thus : — NH.NO^ + KHO = ^1 ' '1-i^l § 128.— Formula of Ammonic Chloride. We have already indicated that the formula of aqua ammonia is NH^OH,, or NII3, R^O. What is the for- mula of ammonic chloride? Is it NH3, HCl, or is it NH4CI? The following experiment may help us to decide. 11 hi FORMULA OF AMMONIC CHLOUIDK. m Experiment. Place about i c. c. of mercury in a wide test-tube, and slightly heat it over a lamp. Drop into it, at inter- vals, two or three pieces of sodium, and heat until they have disappeared. Pour the amalgam thus formed into a strong solution of ammonic chloride, previously pre- pared. Allow the whole to stand for about a minute, then pour off the liquid and wash the residue with cold water. Smell the residue. Apply the flame test for sodium to the liquid which you poured off. After the residue has stood for half an hour or so, note whether any change has taken place in its volume. Test the liquids poured off for a chloride. Evaporate some of the liquids and ascertain what common substance is dissolved in it. 1. QUESTIONS AND PROBLEMS. What is an amalgam ? 2. What change took place on throwing the amalgam into the solu- tion of ammonic chloride ? 3. With what element in the ammonic chlorido did the sodium unite ? How do you know ? 4. When the sodium left the mercury, did the latter form an amal- gam with any other substance ? Give a reason for your answer. 5. What change gradually took place in the volume of the metallic looking mass which formed in the ammonic chloride solution ? 6. If (Hg, Na) "4- NH4CI represents the substances and their rela- tions before chemical action took place, how should the reaction which resulted be represented ? The compound represented by the formula NH4 is often called ammonium. It is sometimes spoken of as a meiallic radical^ because it forms salts exactly analo- gous to those of sodium and potassium. Compare the hydroxides : — KHO = potassium hydroxide NH4HO = ammonium hydroxido. Compare also the salts : — KNOs = potassium nitrate NH4NO0 = ammonium niti-ute. I I m 112 I'RArTlCAL CIII'-MIHTRY. § 129.- -Hydroxides from Salts. You have already learned one method of producing oxides. Hydroxides may be formed from some of these oxides by simply adding water to them. But hydroxides may be formed in another way, which we shall now illustrate. Experiments. 1. Pour into a test-tube i c. c. of solution of ferrous sulphate, FeSO^, and add to it gradually I c. c. of dilute ammonic hydroxide. 2. To I c. c. of solution of zinc sulphate ZnS04, a'd I c. c. of ammonic hydrate. 3. To I c. c. solution of silver nitrate, AgNOg, add i c. c. of ammonic hydrate. 4. Filter the solutions obtained in each case, and dry and examine the precipitate. These precipitates are hydroxides. 5. Carefully evaporate the filtrate in each case, using for this purpose a sand bath. Examine any substance that may remain in the evaporator. 6. Prepare a number of solutions of salts of the me- tals and then add ammonic hydrate or caustic potash solution to each one in turn. Notice carefully the cases in which a precipitate forms. QUESTIONS. 1. How are nyaroxides of the metals obtained? How can you distinguish the hydroxides of one metal from the hydroxides of another ? 2. Wiat use can be made of these and similar experiments for ana- lytical ijurposes ? 3. Explain why a precipitate does not form in every case. 4. Write equations representing the reactions taking place in each experiment. 5. Devise experiments to obtain ammonia from the salts left after evaporating the water in each capn. SOURCES OF AMMONIA. § 130.— Sources of Ammonia. 113 It occurs in the urine and in some other products of animals ; also in air as the result of the decay or decom- position of nitrogenous animal matter ; hence it exists in rain water. Its compounds, ammonic chloride and ammonium carbonate, are found sparingly in nature. m § 131.— other Properties Ammonia is soluble to upwards of 700 times its bulk in water at I5°C; it becomes a liquid at — 40°C, and may even be frozen at — 75°C. § 132.— Uses. Dilute liquor ammoniai is used to neutralize acids spilled upon the clothes or upon the face. Its com- pounds are also used in medicine as stimulants in cases of fainting or of syncope from overdoses of chloroform, ether or laughing gas. They are also used in dyeing. ' • § 133.— Test. When present in minute quantities, as it frequently is in drinking water, ammonia is best detected by what is known as Nessler's test : " To a solution of potassic iodide add solution of mercuric chloride until the pre- cipitate formed just ceases to be re-dissolved, then, add an equal volume solution of caustic potash, and allow the whole to stand until clear. A few drops of this solution will give a yellowish-brown precipitate, with even the slightest trace of ammonia." PROBLEMS 1. Calculate what volume 51 crams of ammonia sag will occupy atl20''C? ^ 9 114 rilAOTlCAL CHKMISTRY. ,. t^ffl 2. 01 litres of Riiini(»nia pis arn dturoniposcMl in a eudioinottir, what vohiuio will itH uoiistitiu'iit gases occujjy ? 3. What weight and volume of aniniuuia gas at GO" F. can be obtained from 214 grams of ammonic chloride ? 4. If 85 grams of aniujonia gas bo decomposed in a eudiometer, and 22'4 litres of oxygen gaa be ad(led to tlio constituent gases, and the mix- ture exploded, what will bo tlio volume of the resulting gases at 0°C ? 6. What weight of quick-lime is reiiuired to decompose 107 grams of ammonic chloride, and what will bo the ■weight of the calcic chloride and water produced ? What volume of ammonia gas will bo evolved at 150° F. CHAPTER XXII. § 134.— Hydrochloric Acid. Before entering upon the study of the next element, viz., chlorine, we shall consider at some length the properties of hydrochloric acid. As the name implies this substance is a compound of hydrogen and chlorine Hydrochloric Acid ('^ Spirit of salt ") : Formula, HCl ; molecular wcighi, j6.^ ; specific weight ; i8.2^. ii,2 litres zveigJi iS'2^ grams. Experiments. I Place some ammonic chloride in a medium-sized test-tube, aud add a few drops of sulphuric acid. Bring a lighted match to the mouth of the test-tube ; also a piece of blue litmus paper ; and lastly, a glass rod dipped in ammonium hydroxide. Smell very carefully. 2. Repeat this experiment, using a large test-tube or flask fitted with a cork or delivery tube, and substituting sodic chloride NaCl, (common salt) for the ammonic chloride. Use twice the weight of sulphuric acid that you do of salt, and apply heat very carefully. Collect some of the gas by passing the delivery tube to the bottom of an "empty" jar. Cover its mouth with a glass plate. Having filled the jar, remove the plate cover, and turn the jar mouth downward over some water coloured blue with litmus. Then shake slightly. Devise a means of finding out when the jar is full. COMMKRCIAL ACID. llf) 3. I^'ill a second jar with this ^ms as before. Place twD or three globules of sodium, the size of a pea, in a deflagrating spoon, the handle of which passes through a cork that exactly fits the jar. Heat the sodium to ignition, and lower the spoon into the second jar of gas. After all action has ceased, withdraw the cork and quickly bring a lighted taper to the mouth of the jar. 11 'i',t , § 135.— Oopimercial Acid. A solution of this gas in water is what is usually sold by druggists under the name of hydrochloric or muriatic acid. How can the gas be obtained from such a solu- tion ? The commercial acid is prepared as a by-product in the manufacture of common "soda" by Lcblanc*s process. QUESTIONS AND PROBLEMS. 1. What other substance was prepared in previous experiments from ammonic chloride ? Contrast the diflPereut properties of the two ditfer- ent gases thus differently prepared. 2. Write a description of the physical properties of hydrochloric acid gas. 3. Explain the effect of dry hydrochloric acid gas on air. 4. What common substance was formed by burning sodium in this gas ? What element was liberated ? How do you know ? § 136.— Chlorides. Procure some chemically pure hydrochloric acid and with it perform the following experiments. They illus- trate the formation of chlorides. Experiments. I. Place a few pieces of zinc in a test-tube and then pour upon them about 2 c. cs. of hydrochloric acid. After all effervescence has ceased, remove the surplus zinc and evaporate the solution to dryness. 116 PRACTICAL CHEMISTRY. Mil! IH liti 2. Place a few fragments of j;old leaf in a test-tube, and pour upon them about i c. c. of hydrochloric acid. Warm slightly. After a minute or two, add a (gw drops of nitric acid. 3. Repeat the preceding experiment using small scraps of platinum instead of gold. 4. Place a little cupric oxide CuO, in a test-tube and pour some hydrochloric acid upon it. When the oxide ceases to dissolve, filter, and evaporate the solution to dryness. 5. Fill a tube with hydrochloric acid gas over mercury, and then pass a piece of quicklime CaO under the mouth of the tube. 6. Take 3 test-tubes : half fill the first with a solution of silver nitrate Ag NO3 ; the second with a solution o( mercuric nitrate Hg (NO3) ? ; and the third with a solu- of acetate of lead Pb (CjHjOa) 3. Into each tube pour a few drops of hydrochloric acid. Try to write the equations. Aqua Regia. — A mixture of three volumes of hydro- chloric acid and one volume of nitric acid is called aqua regia. § 137.— Acids on Metals. The first effect of an acid on a metal is the replace- ment of the hydrogen of the acid by the metal ; hydro- gen is, accordingly, liberated. Hence in preparing hydrogen from zinc and sulphuric acid, we represent the reaction thus : — Zn-f H2S04=ZnSO,-f Ha. In this case no heat is applied ; but when we bring cop- per and sulphuric acid together at ordinary tempera- tures, no change takes place. We would ex-^ect, of course, the reaction to be : Cu -j- HaSOl = CuSO, -f H^. ANALYSIS OP HYDROCHLORIC GAS. 117 and so it would be in all probability it any reaction did take place at ordinary temperatures. No reaction, how- ever does take place unless we apply heat, and thus it happens that the hydrogen which is first liberated, acts upon the sulphuric acid, reducing it to sulphur dioxide and water. Thus : H2SO4 4- H2= 2 H2O -I- SO2. Compare this with the action of nitric acid on copper, Here the reaction varies with the strength of the acid, but when this latter is very strong, the following equa- tions will represent the reaction in the two stages in which it occurs. (1). Cu-h2HN08=Cu(N03)2+H2. (2). 2H]Sr03+H2=:2H20-|-2N02. QUESTIONS AND PROBLEMS. 1. Try to form other chlorides by warming hydrochloric acid with metals, oxides or hydrates which you can liud in the laboratory, and which you have not yet used. 2. Arrange under two headings — soluble and insoluble —the chlorides which you have formed. 3. Compare the action of hydrochloric acid on metals with that of sulphuric acid on metal!>. 4. How is the " solvent " power of hydrochloric acid increased ? 5. Explain the effect of hydrochloric acid gas on quick lime. To observe the eflfect, fill a tube with hydrochloric acid gas over mercury, and then pass a piece of quick lime up under the mouth of the tttb«. Is this gas ftosorbed by charcoal ? Find out by experiment. lof '»'»« § 138.— Analysis of Hydrochloric Gas. Experiment- Take a bent tube like that in Fig. 43. Partly fill the tube, as indicated, ulth hy- drochloric acid, and insert in the ends the terminal wires of a battery. These terminals should be carbon. Bring a lighted match to that end of the tube connected with the zinc of the battery. Moisten Fiu. 43. ft I t ! ■ !! I: mi 118 PllACTICAL CHEMISTHY. a piece of colored calico and place it over the other end of the tube. Color the acid with litmus solution. § 139.— Test for Chlorides. Experiment. 1. Dissolve some common salt in half a test-tube full of pure water, and then add a few drops of nitrate of silver solution. Shake. Now pour half of the solution into a second test-tube ; add a little nitric acid to the one test-tube and ammonium hydrate to the other. Boil the one to which you added nitric acid. 2. Repeat this experiment, using any soluble chloride in place of common salt. The gas may easily be distinguished from others which it resembles, by its behaviour in the experiments hitherto described. QUESTIONS. 1. Suggest any reason why the terminal wires should be tipped with carbon. 2. You ought to be able to recognize one of the gases evolved in the bent tube. Name it and give a reason for your answer. 3. What was the colour of the precipitate thrown down by the nitrate of silver ? How was it affected by nitric acid, and how by ammonium hydrate ? 4. Suppose we have a solution containing 8.ilt8 of any number of metals, and add hydrochloric acid, what three metals will be removed by forming insoluble precipitates with the acid ? § 140.— other Properties. Hydrochloiic acid gas is soluble in water to the extent of 480 times its own volume ; it may be liquefied. EXERCISE. The principle of atomicity may be employed in formulating tlie chlorides of the metals, by taking one ii.olecuIe of hydric chloride as a type, and substituting for its atom of hydrogen one atom of a monad CHLORINE. 119 metal. One atom of a diad metal must ho substituted for two atoms of hydrogen in two molecules of hydric chloride, to form the chloride of a diad metal and so on with triads, tetrads, &c. There are impor- tant exceptions to this application of the principle. Type. Chloride. Name. HCl NaCl Sodic Chloride. 2HC1 CaCla Calcic Chloride. 1. Apply this principle and formulate theoretical chlorides of the following metals : — Arsenic, gold, tin, manganese, iron, potassium, mercury, silver, zinc, calcium. 2. Name the compounds thus formed. 3. State as hriefly as you can the reasons for helieving that hydro- chloric acid gas is composed of equal volumes each of hydrogen and chlorine. 4. What weight of common salt and sulphuric acid must he taken if it be required to eliminate 146 grams of hydric chloride ? 5. Calculate the amount of hydro-sodic sulphate that will he pro- duced in generating 219 grams of hydric-chloride from salt and sul- phuric acid at a moderate temperature. 6. Explain why we helieve that hydrogen and chlorine are united in the proportions hy weight of 1 to 35.5. 7. What volume will 73 grams of hydric chloride occupy at the standard temperature and pressure ? 8. What is the percentage composition of the gas? 9. Calculate the weight and volume of hydric chloride at 30°C. that can he formed hy heating to a moderate temperature 409.3 grams of common salt and 686 grams of sulphuric acid ? CHAPTER XXIII. § 141.— Chlorine. This element is never found uncombined in nature, but is generally obtained from the well-known substance, common jalt. Chlorine : Symbol, CI ; atomic iveigJit, JS-S >' ^noleadar iveight^ Cl^, 7/ ; specific weigJit, JS-5- i^- litres weigh 35-5 g^^^^^^' I 1 II 1 mm BB 1 i 11 1 1 ! W - '(" I i •tiiii It!! nil 120 PIIACTICAL CHEMISIUY. Experiments. 1. Into a test-tube put one part of manganese dioxide, two parts of salt, and three of sulphuric acid. Fit the test-tube with a cork and delivery tube. Heat gently and pas9 the gas that comes off into separate solutions of h'tmus and indigo. Cautiously smell the gas. Note its color. 2. To prepare the gas on a larger scale, take a 4 oz. Florence flask and place in it about 20 grams of manganese dioxide and looc. cs. of strong hydrochloric acid. Use fit- tings similar to those in Fig. 44. Apply a very gentle heat. The delivery tube should pass silmost to the bottom of the jar. Fill several jars, taking care that little or none of the gas escapes into the room. Afterwards pass the gas into a flask perfecily ///// of water ; in about ten minutes place this flask aside for future use. Smell the water. r'la. 44, QUESTIONS AND PROBLEMS. 1, Write out an account of the physical properties of chlorine. '2. In the first experiment what gaseous compound is formed when f.*' \ sulphuric acid acts on the salt ? What becomes of this compound? .3. How does the first method of preparing the gas di£Fer, if at all, from that empkiyed in the second ? 4. What class of oxides will cause hydrochloric acid to yield up its chlorine, and what class will not ? 5. Treat the refuse from preparing oxygen from manganese dioxide and chlorate of potash with sulphuric acid, and observe what gas is evolved. § 142.— Properties of Chlorine. Experiments. I. Take the flask full of chlorine water, prepared in the last experiment, and fit it with a cork and tube. The PROPERTIES OF fHLORINK. 121 i 'ill Fio. 45. outer end of the tube must be drawn to a fine point Insert the cork so that there is not a bubble of air left in the flask. Invert the flask as in Fig. 45, and expose to direct sunHght for a day. Then place the flask on the table, re- move the cork, and quickly bring a glowing splinter to the mouth of the flask. Test the water in the flask with blue litmus solution. Taste it. 2. Lower very slowly a lighted taper into a jar of chlorine. At the same time suspend a piece of blue litmus paper at the mouth of the jar. Smell the gas that is formed during the combustion. 3. Wet a piece of blotting paper with oil of turpentine, C10H16, and then place it in another jar of the gas. 4. Take a few pieces of the metal antimony and pow- der them ; then place on a sheet of paper and shake the powder into a jar of chlorine. 5. Fill a small jar with hydrogen and then bring its 'mouth below the mouth of another jar of chlorine. Keep the jars mouth to mouth, and invert them several times, so as to mix the gases thoroughly. Then separate the jars, carefully corking one, and applying a lighted match to the other. Wrap a towel around the jar which you have corked, so as to exclude the light, and then carry it to where the sun is shining, either in a room or out- side. Place it on the floor or on the ground and quickly unroll the towel so as to send the jar a short distance from you. Chlorine and hydrogen should never be mixed excepting in a c/im light. QUESTIONS AND PROBLEMS. 1. "What element is liberated by the action of chlorine on water? What compound is formed ? 2. Roughly speaking the wax of a taper consists of oxygen anro- ^M- >rtii an hen 41. ibis * Jf all the air be not expelled from the flask an explosion will nsult. When it is desired to stop the formation of the ga.s, enough warm watir should be jjoured into the evaiwrating dish to flh the flask. Tlie lamp should then be witlidrawn. As the solu- tion cools, the warm water is drawn u into the iuside of the flask, and the ovolutiou of gas ceases completely HANI>. \r.\ ro- U8 en lis 3. Tabulate in dpposito columuH tho nwinhhinccs aiul JilFeronces l>e- tween red aiul yellow ])li(>H{)liorus. 4. llow Would you think it jmaflihlo to turn re«l ]>hoHplioru8 back into tho yellow variety V CIIAPTKR XXVII. § 173.— Sand. In treating of the chemistry of the earth, some refer- ence must be made to sand, and to sandstone from which sand has been formed by disintegration. This substance occurs in varying proportions in nearly all good soils, but a soil consisting of pure sand is almost entirely un- productive. Analysis shows that sand is an oxide of the element silicon, the most abundant solid clement in nature. So widely is this element distributed, that it con- stituces from 22 to about 36 per cent, of the solid crust of the earth. It is never found uncombined. and like carbon is known in three modifications — amorphous, graphitoidal, and crystalline. The chemical name for pure sand is silicon dioxide or silica SiO^. The modi- fications of this compound are numerous, as will be seen by a glance at the following list : — " J. Quartz crystals, glassy hcxagonar prisms terminating in hexagonal pyramids. 2. Amethyst^ smoky quartz^ rosy quartz^ and chrysopra^e, colored varieties of quartz. 3. Quartzite^ a sedimentary rock. 4. Sa?id and sandstone^ fine fragments of quartz more or less cemented together. 5. Honesto7ie or novaculitey a fine-grained quartz rock. 6. Chalcedony, a mixture of crystalline and non-crystal- line quartz. 7. Agate^ consisting of layers of crystallized and amor- phous quartz of various colors. 8. Flint and cherty a coarse variety of chalcedony. 9. Opal^ a hyd rated form of silica. 10. Various modifications of the above in which one form is passing into another." < 'MS %-^ it \m fiSB nan ' m: 144 PRACTICAL CHEMISTRY. § 1 74.-Use of Sand Sand, or silica as we shall now call it, is that con- stituent of the soil which give firmness and stiffness to the stalks of grains and grasses, so that they are enabled to maintain the upright position. § 175.— Silicates. Silica is insoluble in water and in all acids except one. Theoretically, it is supposed to unite with two molecules of water to form silicic acid. Thus : — SiO, + 2H2O = H^SiO,. . Silicic acid. No such direct union of water and silica has ever been obtained by experiment, but this acid can be prepared indirectly, and its compounds with metals, that is, its salts, are well known, and as we shall see presently, are important. These salts are called silicates. Silica may therefore be considered the anhydride of silicic acid. What other anhydrides of acids have you studied ? iW-f' § 176.— Oiay. Clay, like sand and limestone, is a very abundant substance. Of what is it composed ? We found that limestone was made up of an oxide of calcium and carbon dioxide ; similarly, it has been found that clay is composed of an oxide of the metal aluminum, AI0O3, united with silica SiOo ; it is, in short, a silicate of alu- mina. Aluminum, the metal of clay, is a white body with a brilliant lustre ; it is 2.67 times heavier than water ; may be rolled cut into sheets, and drawn into fine wire ; and it is not easily oxidised. The great cost of extracting it- is the only reason why it is not exten- sively used in the arts. Every clay bank is a mine of 4iM ALUMINA. 145 aluminum, so that, next to oxygen and silicon, aluminum is the most abundant of elements. The metal gets its name from the fact that it is found in alum. § 177. — Alumina. Alumina is an oxide of aluminum, AI.2O3. A variety of this compound is known in nature as corundum, or emery, when coarse ; but when crystallized it is known to jewellers as sapphire, ruby, oriental emerald, oriental topaz, and oriental amethyst. Experiments. 1. Take a piece of ammonia alum and heat very strongly until all action has ceased. The residue is alu- mina. 2. Dissolve some alum in water ; add ammonia ; fil- ter. Remove the filtrate from the paper, and heat very strongly on a piece of platinum foil. The residue is alu- mina. What compound is precipitated ? § 178.— Soluble Glass. Potassium silicate and sodium silicate are often called soluble glass. They are used in making artificial stone. Soluble glass may be prepared as follows : — Experiment. Powder 4 grams of fine white sand and place in an iron ladle. To this add 8 grams of caustic potash and about 70 grams of water. Boil the whole for three or four hours over a hot fire, supplying fresh water as it becomes necessary. Reaction : — SiO., + 4KHO = ^,SiO, + 2H2O. II Pottissiuiu silicate. fi9ni m m !!^f I'M ' III i i m, : U6 PRACTICAL iHKMISTIlY. § 17 9.- Glass. " The various glasses of commerce are mixtures of a highly silicious silicate of sodium or potassium or of both, with silicates of other metals such as calcium, aluminum and lead." Thus, windo'v glass is a double silicate of sodium and calcium ; Bohemian glass (hard test-tube glass) consists of silicates of potassium and calcium ; flint glass contains silicates of potassium and lead; and bottle glass is a mixture of silicates of alumina, calcium, iron and sodium. § 180.— Pottery. The arts of moulding and pottery depend upon the plastic properties of various kind of clay, when mixed or puddled with v/ater. Experiment. Take a little kaoline or any common clay and place upon a plate. Add a f^w drops of water and knead it into a thick paste. Mould i*; into the shape of a small cup, and try whether this cup will hold water. Dry thoroughly but slowly, and then place carefully in a very hot fire and bake for a few hcurs. *l II ! «II ;;!.. § ISl.-Soil. " All soils have been produced by the disintegration of rocks, generally through the prolonged action of water, air, and frost. The character of a soil largely depends upon the character of the rocks from which it has been derived. Primitive and igneous rocks yield soils rich in potash ; fossiliferous rocks produce soils rich in phosphoric acid. The principal mgredients of soils are sand, clay, carbonate of lime, and humus, or decayed vegetable matter. As each of these prepon- derate tiie soil is sandy, clayey, calcareous, or peaty." ANALYSIS OP AIH. 147 CHAPTER XXVIII. § 182.— Analysis of Air. We have already learned how to make a simple volumetric analysis of air. The result of our experiment in § 28 seemed to indicate that oxygen and nitrogen occurred in air in the proportions, roughly speaking, of one of the former to four of the latter. A more accurate analysis of the proportions by volume in which oxygen and nitrogen exist in air may be made by means of a eudiometer. Experiment. Put a known volume of pure air into the eudiometer, and then a volume of hydrogen sufficient to unite with all the oxygen in the air. Note the total volume. Now pass a spaik of electricity into the eudiometer, taking all the precautions mentioned in § 31. QUESTIONS. 1. What volume of air was passed into the eudiometer? Of hydro- gen ? How much, therefore, of both together ? After explosion, what volume of gas was left ? 2. What was the diminution in volume ? Why must j^ of this dim- inution represent the oxygen present in the air wliich was first passed into the eudiometer ? 3. How then is the volume of the nitrogen in air found ? Twenty-eight volumetric analyses by Bunsen gave the following results : — Oxygen 20.92 per Nitrogen 79.08 cent. § 183.— Compositiou of the Air by Weight. Experiment i, in § 25, illustrates a method of ascer- taining the composition of air by weight. A known ft' I 148 practicaIj chemistry. wcij^ht of air is passed over a known weight of red-hot copper. The gain in weight of the copper represents the weight of oxygen present in the air. Knowing the weight of the oxygen, that of the nitrogen is easily found. The gravimetric analysis of air, made on this principle, by Dumas and Boussingault, shews its com- position to be : — Oxygen 23 per cent. Nitrogen 77 " § 184. -Specific Weight. We have already seen that the composition of air by volume is, nitrogen 79*1 per cent., and oxygen 209 per cent. Knowing this, we can easily calculate its specific weight ; thus : — Nitrogen 79-1 Oxygen 209 a't. w't. < 14 = 1107-4 c 16 =-. .334-4 100 volumes. 1 ' ^lume. 1441-8 14-42 This corresponds very nearly .vith its specific gravity as found by actual experiment, viz., 14*44. The number 14*44 will, of course, represent the weight of ir2 litres of air in grams. § 185. —Air a Mixture. Having analyzed air, the student is now in a position to answer the question : Is air a mixture, or is it a true chemical compound ? To assist in deciding this point, Tl IMPURITIKS IN A IK. 149 the student should devise means of preparing air syn- thetically. In doing thi.} he should observe carefully whether the oxygen and nitrogen on being brought together show any of the characteristics of a chemical change. He should watch particularly for (i) a change of temperature, (2) change of color, (3) change of volume, (4) change of state, or other evidences of chemical action. He should also inquire whether the analysis of air shows oxygen and nitrogen to be united in simple proportions by weight and volume, such as is the case in the union of oxygen and hydrogen to form water, and of carbon and oxygen to form carbon dioxide. Experiment. Expel the air from water and then analyze as in § 28. Compare the proportions by volume in which oxygen and nitrogen occur dissolved in water, with the pro- portions in which they exist in air. Would these pro- portions be the same in both cases if air were a chemical compound ? § 186. —Impurities in Air. Chemists have agreed to consider pure dry air as being made of nothing but oxygen and nitrogen ; but we have already seen that there is present in ordinary air another substance, viz., vapor of water. Our experiments with burning carbon would certainly lead us to suspect the presence of yet another ingredient in air. Moreover, the fact that lime water turned milky when expired air was passed through it, might also suggest the presence of the same ingredient in the air. These conjectures we must now put to the test of experiment. Before trying, however, -to ascertain whether our conjectures are right or wrong, we shall make a preliminary experiment which may aid us in our investigations. il-!' Tmmmmmmmm i 11! iii m. 1 { til : '■■si >i f50 Experiment. PUACTICAL CHEMISTRY. I. Take a bottle such as is represented in Fig. 46. Partly fill with lime water. Then ap- ply the mouth to the shorter tube and suck in air, drawing in fresh air through the longer tube. Do not con- tinue this process for longer than a minute. Now reverse the process and blow air through the longer tube for about the same length of time. 2. Clean the bottle used in this ex- periment, and again partly fill with fresh lime water. Attach an aspira- tor to the end of the shorter tube, and draw air through the longer tube, so that you can count the bubbles as they pass. Continue the process for half an hour or thereabouts. If you have no aspirator, you should use a large bottle filled with water, and Fio. 46. arranged as in § 25, when preparing nitrogen. § 187— Ozone. Symbol O3 ; moleailar iveight, 48. This substance is merely a modified form of oxygen, being one in which there are supposed to be three atoms in the molecule instead of two. It "corrodes india- rubber ; oxidizes silver, mercury, and many other metals, and, in doing so, changes to ordinary oxygen ; possesses great disinfecting powers, and is a powerful bleaching agent. If breathed in small quantities it is said to be beneficial in the treatment of affections of the throat and lungs. City air contains little ozone, but in sea and country air it is present in considerable quantity. ORGANIC MATTER. 151 Experiment. Make a strong solution of potassium permanganate K2Mn208, and place a little of it in a test-tube. Then add a few drops of strong sulphuric acid. Ozone is given off, and may be distinguished by its odor. § 188.— Organic Matter. We have now mentioned all the substances usually enumerated in giving the composition of air, but it is evident that we have not exhausted the list. By more delicate means of analysis than you can be expected to employ as yet, chemists have discovered traces of ammonia and nitric acid in air. and, in the air of large towns more particularly, traces of sulphurous anhydride and sulphuretted hydrogen. Besides all these, there are few people who have not noticed the presence of sub- stances that are peculiar to the air of certain localities. The air is supposed, and certainly not without reason, to be the medium through which the germs of certain infectious diseases are carried from one point to another. Minute particles of dust and smoke pervade the ?ir of most towns. § 189.— Composition of Air. The average composition of air is about as follows — Cubo centimetres. Oxygen, including Ozone 206 1 Nitrogen 7795 Aqueous vapor (invisible vapors clouds and fog) 14"0 Carbon dioxide 4 Traces of ammonia, nitric acid and sulphuret- ted hydrogen .0 Organic matter "0 Dust and smoke "0 Total 10000 I Fwmm 152 t ■II 'I ■Hi PRACTICAL {'IIKMISTRY. § 190.— Analysis of Expired Air. Experiment. Fill a small graduated tube with mercury and invert over another and larger tube also filled with mercury. Pass into the gradu- ated tube, by the use of a curved pipette, as in Fig. 47, a measured volume of expired air. Immediately afterwards pass into the graduated tube a few drops of a strong solution of caustic potash or caustic soda, and note any change in the volume of con- fined air. Pass in also a little strong pyrogallic acid, and again note any change in volume after adjusting the level of the mercury inside and outside the graduated tube. QUESTIONS 1 . What eflfect has pure air on limewater . Expired Fia. 47. air ou lime water ? Kxplaiu the diflference. 2. What effect has caustic soda or caustic potash on expired air ? Ex- press in the fi )rinof an equation. What ingredient did the potashremove? 3. What volume of expired air did you put into the graduated tube? 4. What ingredient did the pyrogallic acid remove ? What volume of it ? What gas must have remained in the tube ? What volume of it ? 5. Compare the proportions by volume in which oxygen and carbon dioxide respectively exist in common air and in expired air. § 191.— Plant Influence on Air. We have learned how that, in respiration and in all ordinary combustion, air is being contaminated with enormous quantities of carbon dioxide. Do other animals besides man consume oxygen in respiration? If so they are but increasing the amount of poisonous gas present in the air, and consequently lessening the time within which all the oxygen available for the sup- port of life will finally disappear. The question therefore naturally arises, zv/ij> docs not air soon become so impure as to be unfit to support human life? This question we shall endeavor to answer in our usual way. DIFFUSION OF GASKS. 153 Experiments. 1. Fill a large glass bottie with spring water, or better still, with drinking water, pre- viously impregnated with car- bon dioxide. Then invert it over a plate containing water as in Fig. 48. Put into the inside of the bottle a sprig of fresh mint or of water cresses, and expose the whole to strong sunlight for two or three hours. Examine carefully and test with a glowing splinter the gas which collects in the upper fw. js. part of the bottle. 2. Repeat the experiment, keeping the plant in a dark room. § 192— Diffusion of Gases, Air, however, whether found on mountain tops or in valleys, whether taken from the green fields of England or from the vast prairies of America, is kept uniform in composition through the influence of a physical property which is characteristic of all gases. Experiments. 1. Fill two separate bottom- less glass bottles with oxygen and nitrogen respectively, and place one above the other, as shown in Fig. 49. Then insert a burning candle through the mouth of the upper bottle, and quickly lower it into and raise it out of the oxygen gas. 2. Repeat the experiment, but after bringing the two bottles together, wait for half an hour before testing with the candle. Fio. 49. "Tfi^mfswm 154 PRACTICAL CHEMISTRY. 3. Fit a new and clean porous cell of a galvanic battery with an India rubber cork and tube, and support in a beaker of water, as in Fig. 50. Then fill a large beaker with hydrogen and bring it mouth downward, over the porous cell. QUESTIONS. 1. What eflfect have plants on car- bon dioxide dissolved in -water? What force produces the change ? Via. 50. 2. Is it fair to assume that plants have the same effect on the gas, when in air ? 3. What do plants do with the carbon of the carbon dioxide ? 4. U''hat other iufiuences besides that of plants help to keep the air uniform in composition over the whole worhl. 5. What visible phenomena occurred in experiment 3, § 192? Did the bubbles contain air or hydrogen ? Give reasons for your answer ? 6. Use other gases than air and hydrogen in this experiment, and note changes in tiie phenomena. § 193.— Graham's Law of DiflPusion. The rates of diffusion of two different gases m contact v/ith each other is inversely proportional to the square root of their atomic weights. § 194.— Ventilation. According to Dr. Parkes the air in our houses becomes unwholesome when carbon dioxide exceeds 6 parts in 10,000. If this be so, it becomes a matter of great importance to know when this amount is exceeded. Dr. Angus Smith, a celebrated authority on sanitary NATURAL WATERS. 165 science, lays down the foltovvinj^ rule : — " I.ct us keep our rooms so that the air gives no precipitate when a ioJ4 oz. bottle of the air gives no precipitate with half an ounce of clear lime water." Devise an experiment to test the purity of the air in your school room, employing iJr. Smith's test. CHAPTER XXIX. § 195.— Impurities in Water. The sOiVent action of water and the consequent im- possibility of obtaining it pure in nature, have already been studied. We propose in this chapter to aid the student in testing for the impurities to be found in ordinary drinking water. Obviously our first step is to classify natural waters in some way, and to specify the impurities usually found in each class of water, and then to leave to the learner the task of discovering, by appro- priate tests, what kind of impurity is present in the waters of the locality in which he lives. § 196.— Natural Waters. Naturally occurring waters may be divided into four classes : — 1. Rain water. — This being the product of natural distillation, is the purest form in which we meet with water in nature. Even this, however, contains as impu- rities, the gases of the air, tiaces of common salt, ammo- niacal salts, and organic matter of various kinds. If collected from the roof of a house it will, of course, con- tain other impurities besides those mentioned. 2. Spring water. -The impurities in this kind of water will vary greatly, and will be determined chiefly 156 PRACTICAL CIIEMIHTHY. by the character of the soil through which the water per- colates before rcachinpj the surface of the earth. It con- tains sodic chloride (salt), calcic sulphate, small quanti- ties of mapjnesia carbonate and sulphate, silicates and a variety of other substances. Carbon dioxide is fre- quently found in spring water. Less frequently this \vater is impregnated with sulphuretted hydrogen. Well water is, of course^ only a variety of spring water. In addition to the impurities which may be found in ordi- nary spring water, city well water is likely to contain ammonia, the nitrates and nitrites of calcium and sodium, and worst of all the drainage from animal refuse. These impurities are considered to be exceedingly deleterious to health, as they frequently give rise to various kinds of disease, such as typhoid fever The well water of towns is more likely to be contaminated with sewage matter than that of country districts. Even in country districts, however, wells are often placed near stables and byres ;and the water used for drinking and culinary purposes is thus frequently rendered most unwholesome. Springs, whose waters contain considerable quantities of salts of various kinds in solution, are called mineral spnngs. Such are Saratoga, in New York State, and Caledonia in Canada. Mineralized waters have in all ages been highly esteemed for their medicinal properties. 3. River water, — The suitability of river water for drinking purposes will depend very much upon the character of the organic matter suspended or dissolved in it, and this organic matter again will depend upon the region drained by the stream. Rivers usually con- tain a much less amount of salts in solution than spring water. If the district drained is low and marshy, and the course of the stream sluggish or nearly stagnant, the water is not likely to be wholesome. But if, on the other hand, the district drained be hilly and the current a rapid one, or if the bed of the stream be rocky or sandy, the water will likely be thoroughly good. Running water is fitter for drinking than stagnant, because its motion exposes a fresh surface to the air, so i| :i|';i TESTS FOR IMIMTKITIKS. 157 pcr- con- lanti- .nd a i fre- ' this Well •. In ordi- )ntain dium, These crious kinds ter of ewage Duntry itablcs dinary ^some. ntities dneral and in all )erties. er for the solved upon y con- spring y, and nt, the )n the urrent ky or gnant, air, so •n *'uit the oxygen is continually absorbed and oxidises the animal and vegetable matter in the water. In this way noxi(nis compounds are often transformed into others which are perfectly harmless. 4. Sea water. — This is, of course, never used for drinking or culinary purposes unless first purified by distillation. It is the product of the washing of the land for long ages, and it contains, as might be expected, enormous quantities of soluble salts of various kinds. In what part of the ocean would you expect to find water of the greatest specific gravity ? Messrs. Thorpe and Moreton analysed sea water from the Irish Channel, with the following result : — Water 966.144 SuJium chloride '26.4:^9 Potassium chloride 0.746 Magnesium '• 3.150 Inomide 0.070 " sulphate 2.066 " nitrate 0.002 Oalcium sulphate 1.331 '• carbonate 0.047 Lithium chloride traces Ammonium chloride " Iron carbonate 0.005 Silica traces 1000 parts. QUESTIONS AND PROBLEMS. 1. Catch some rain water in a large, clean earthenware or glass ves- sel, and tiud out how much gas is dissolved in it. 2. Test some of it for ammonia. 3. Test a fresh sample of it for common salt. 4. Mix together some alcohol and distilled water, and then try to prepare some pure water from the mixture by distillation. 5. Mix other liquids with pure water and then try to separate them by distillation. § 197.— Tests for Impurities. The presence of ammonia, with chlorides and nitrites in drinking water, generally indicates contamination Pr>7^ iw^ 158 PIIAOTICAL OHKMISTRY. with sewage water. To decide, therefore, whether water is potable or not, the student should test for ammonia, then ^or chlorides, and lastly for nitrites. Water might however still be unfit for drinking, though containing none of these impurities. Organic matter of vegetable origin such as is found in the waters of some sluggish streams may render such water unfit for use. Evidently then, our next step should be to test for organic impuri- ties. Two tests are usually given, one when little organic matter is present ; the other, when nuick is pre- sent. § 198. — Organic Impurities. Experiments. 1. Place the water to be tested in a Florence flask, and add to it, first, a few drops of sulphuric acid, and then enough of a solution of permanganate of potash to give to the whole a deep purple tint. Set to one side for an hour or two, in a warm place, and if the solution loses its color, organic impurities are present. 2. Fill a bottle with the water to be tested and cork it very tightly. Set aside in a warm place for a few days and then examine. An offensive odor indicates the presence of niuch organic matter. § 199.— Hardness. Water that contains magnesium and calcium salts, and curdles scap, is said to be hard ; water that does not contain, these salts is soft. Hardness is usually con- sidered as beina;: of two kinds, viz., temporary dixxd perma- nent. The fon er is due to the presence of calcic and magnesic carbonate, the latter to the presence of salts of calcium and magnesium other than the carbonates, such as ihe sulphates and nitrates. CLARK S SOLUTION. 159 H vatcr onia, night ining itable ggish ently ipuri- little s pre- ;k, and d then to give for an loses d cork a few icates salts, Lt does y con- penna- cic and -}{ salts onates, § 200. -Clark's Solution. We usually test the decree of hardness by what is known as Clark's solution. This may be prepared as follows : ** Dissolve lo grams of castile soap in i litre of dilute alcohol, containing about 35 per cent, of pure spirit." The solution deteriorates by standiug. A degree of hardness represents .064 gm. of calcic carbonate or its equivalent in soap destroying power, in 4543.5 c c. of water. In calculating the degree of hardness, we include, of course, both temporary and permanent hardness. § 201.— Temporary Hardness. Experiments. 1. Half fill a large test tube with clear lime water, and then pass into it a stream of carbon dioxide. At first the usual result follows ; but as the stream is continued the milkiness gradually disappears on account of the formation of a supposed bicarbonate of lime H2Ca(C03)2, which, unlike the carbonate, is soluble in water. Water with the bicarbonates of calcium of magnesium and of iron dissolved in it, is said to be temporarily hard. 2. Divide the water obtained in the preceding experi- ment into three parts and perform the following operations : (a) To the first part add some of Clark's soap solution. (b) Boil the second part of the solution for half an hour. Then filter and test the filtrate with Clark's .solu- tion. (c) Add carefully, to the third part, some clean lime water. Filter and test with Clark's solution. § 202. —Permanent Hardness. Experiments. I. Fill a large test tube or small Florence flask about three-fourths full of pure water. Then add a gram or Wf-r V ■ 160 PRACTICAL CHEMISTRY. two of calcic sulphate CaSO;,, and shake vigorously. Filter. The clear water contains calcic sulphate in solu- tion and \^ permanently hard. 2. Divide the water thus obtained into four parts, and perform the same experiments with three parts of it that you did with the water which had bicarbonate of calcium dissolved in it. 3. To the fourth part add a solution of washing soda, Na-jCOs. Note the result carefully. Filter and test the filtrate as in the preceding cases with Clark's solution. QUESTIONS AND PROBLEMS. 1, Devise an expei-iment to distinguish in a rough way, drinking water thiat contains organic matter of vegetable origin ; from drinking M'^ater that contains organic matter of animal origin. 2. Explain how organic matter may be removed from drinking water? .3. Complete the two following equations. When complete they represent tlie reaction of washing soda on two kinds of hard water. HgCalCOa), +Na,C03 = CasO^ 4- NaaCOs - 4. Exphun why some hard waters may be softened by boiling. Men- tion othe'' methods of softening hard water. 5. Ascertain whether rain water that has been collected from the roof of a lioube contains any organic matter. 6. Ascertain whether spring water and river water contain any organic matter. CHAPTER XXIX. § 203.— Definition of Chemistry. We have now reached a point m our studies at which it will be profitable for us to pause, and take a general view of the subject, so far as we have studied it. We have had revealed to us something of the wonderful na- ture of many familiar substances, such as air, water, sul- phur, charcoal, limestone, sand, and clay ; and have observed the marvelous changes which these substances, and others derived from them, undergo when subjected CHEMISTllY AND PHYSICS. 161 to the influence of heat, light, electricity, or the still more subtle force, chemical affinity. At the beginning of our studies the fact was made prominent that one of the first questions which a chemist asks himself regarding any substance is : Of what is it composed? While this is quite true, you have now to learn that he also asks himself : Wliat can this substance do ? This second question is by all odds more difficult than the first ; but, notwithstanding this, the chemist patiently and laboriously endeavors to answer it. In one case he seeks to become acquainted with the facts and prin- ciples concerned with chemical composition^ in the other he endeavors to grasp the principles and mode of chemical action. Chemistry, herefore, may be defined as the science which deals with chemical action and its products, whether this action has taken place in past ages or in our own times. vhich ;neral We ul na- •, sul- have ances, iected § 204. — Chemistry and Physics. The student is now in a position to appreciate the dis- tinction between physics and chemistry, and to under- stand their relation to each other. Physics is that branch of knowledge which deals with motion, light, heat, electricity, sound, magnetism, and all physical changes. Chemistry deals with the composition of sub- stances, the changes in composition which substances undergo, and the conditions and laws which govern these changes. Although physical and chemical changes appear at first sight to be quite unrelated, they are not really so. Motion when stopped turns into heat ; heat generates motion. The locomotive moves through the influence of heat ; but what causes .':he heat ? Evidently the burning of the fuel. In this burning, however, a change in the composition of the fuel is actually taking place all the time, and this chemical change is generating heat, *t 1G2 PRACTICAL CIIEMFSTRY. and the heat is causing- the motion. Wc have frequently seen a chcniical chanl '» \ m n 166 PRACTICAL CHEMISTRY. Lussac's statenient that equal voliimr.s contain equal numbers of atoms must be abandoned. The difficu'ty was removed by Avo^adro, who introduced the idea of two kinds of atoms — * molecules intej^rantes,' or as we should now call them molecules; and 'molecules (ildmcntaires,' or as we should now say atoms. The molecules of elements are decomposed in chemical pro- cesses, said Avo^adro, and the atoms unite to form new con. pounds. The reaction between nitrogen and oxy- gen, inexplicable by Gay-Lussac's law, now becomes Clear , each molrcule of nitrogen and each molecule of oxygen divides into two parts, and these parts unite to form the new molecule of nitric oxide, hence t!iere are twice as many molecules of nitric oxide produced as there were molecules of nitrogen or oxygen originally present." The gain to the chemist in adopting the molecular theory has been immense, It affords the best known explanation of such phenomena as the diffusion of mat- ter, diffusion of motion, diffusion of lieat in gases, evap- oration, condensation, electrolysis and spectroscopy. § 208.— Division of the Molecule. Let us now consider a few chemical reactions with which we have become familiar. Take first the one between oxygen and hydrogen in which steam is formed : 2 vols, of hydrogen luiito with 1 vol. of oxygen to form 2 vols, steam ; therefore, since equal volumes contain equal numbers of molecules, 2 niols. of liydr. gen + 1 mol. of oxygen = 2 mols. steam. .-. 1 mol. hydrogen + 1^ mol. of oxygen = 1 mol. st am. Hence, it becomes evident that the molecule of oxygen in this reaction must be divided into two parts. Again : 2 vols, of hydrogen unite with 2 vols, of chlorine to form 2 vols, of hydrochloric acid, Ji DIVISION OF THK MOLKCULH. 167 and since c(inal volumes of ^ases contain the same num- ber of molecules, it follows that 2 mols. of chlorine + 2 mols. of hydrogen - 2 inolo. of liydrocliloric gas, .■. 1 mol. " 4- 1 mol. " = 1 mol. of " « Now, since each molecule of the hydrochloric acid vnist contain both hydro<^en and chlorine, it is evident that each molecule of hydrogen and each molecule of chlo- rine must have divided into it at least two parts, and from the union of these the new molecule of hydro- chloric acid has resulted. Again : — 2 vols, of nitrogen combine witli 6 vols, of LvtVag^^i^ ^ to form 4 vols, of ammonia (ga,s), .-. 2 mols. of N + 6 mols. of H = 4 mols. of NHg, .-. \ "• of N + 1 mol. of H = 1 mol. of " . Here again it is evident that the nitrogen molecule must have been separated into two parts. And so, when various other reactions between gaseous substances are studied, the same conclusion is found to be unavoidable : the chemist is forced to recognize a smaller portion of matter than the molecule. This smaller portion he calls an atom. The student must here notice that the conclu- sion that there are atoms is reached by reasoning, based upon the results of our experiments. Believing Avogadro's law to be true, and remember- ing that the hydrogen molecule diviv^es into two parts in many chemical reactions, we arrive at the following practical definition of the molecular weight of a gas. " The molecular weight of any gas is the weight of that volume thereof which is equal to the volume oc- cupied by two parts by weight of hydrogen." But it is evident that we have not yet learned all from. Avogadro's law and our experiments that we may learn. In the first place, it is clear that the molecule of an ele- 1 ;, 1 ;i 1 108 PRACTICAL OIIEMISTR/. ment cannot contain less than two atoms, unless indeed the atom and tlie molecule should be identical ; and it is also clear, that the molecule of a compound cannot contain less than one atom of each of its constituent elements. It follows, therefore, that we may find the atomic weight of any element by determining the smallest part by tveight of an eleme?tt m the tnolecule of any of its gaseous compounds. In other words, the atomic weight is determined by ascertaining two distinct classes of data : (i) *'*The specific gravity of a series of gaseous com- pounds of the element in question ; (2) " Careful analysis of these compounds." For example, the following data must be obtained in ascertaining the maximum atomic weight of oxygen : — Compound. \Vt. op 2 Vols, of Ga8. H=1. Analysis of Compound, Water, 17-99 15-96 of oxygen I + 2 hydrogen. Carbonic oxide, 27-96 15-96 " + 11 -97 carbon. Carbon dioxide. 4415 31-92 " + 11-97 " Nitrous cxide, 43-9 15-96 " + 28-02 of nitrogen. Nitric oxide, 30-0 15-96 " + 14*01 of nitrogen. Sulphur dioxide, 64-9 31-92 " + 31-98of suljjhur. ** trioxide. 86-9 47-88 " + 31-98" •• From a study of the above data it becomes clear that 15-96, or in round numbers, 16 is the atomic weight of oxygen. § 209.— Unit of Volxime. In chapter XTV., we explained the various units of volume that have been adopted for the measurement of CLASaiPIC!ATION OP KLRMENTS. 169 ^ascs ; bu inasmuch as 22.327 litres is the vohniic of all gases which \vei<;hs the units expressing th ir relative molecular weights, this volume is perhaps the one which is now in most general use. This volume of hydrogen at o°C, and 760 mm. barometric pressure, weighs 2 grams in the metric system. In our English system of weights and measures the volume of hydrogen is 377 cub. feet, and, at 60° F., and 30 inches barometric pressure, weighs 2 lbs. As already intimated, chemists have agreed to take the least weight of any element found in such a molecular weight, as the weight of one atom of the element in question. The practical advantage of this unit may be seen in the following table : — 00 '19 7 litrc3 of hy(h-ogen weigl I 2 grams II oxygen II 32 .1 II chlorine II 71 II nitrous oxide II 44 II fi Ciirbon dioxide II 44 II ammonia II 17 M II hydrochloric gas II 36.5 II and so on with all gases. CHAPTER XXXI. § 210.— Classification of Elements. We have already referred to the arbitrary division of the elements into metals and non-metals. I'his division is of very little practical importance, especially as it has been found impossible to discover any property that is common to all the substances called metals. It would seem better to divide the elements into two great classes, (i) acid-forming and (2) base-forming. Nearly one- fourth of the elements unite with oxygen and hydrogen tmamm^ IMAGE EVALUATION TEST TARGET (MT-3) I.C I.I s. 113.2 IIIIM [2.2 2.0 1.8 1.25 1.4 1.6 M 6" ► o^. % •^^' ^<3 » o 7 ///. Photographic Sciences Corporation «■ # €^ \ \ ^9) % ^^ 23 WEST MAIN STREET WEBSTER, N.Y 14580 (716) 872-4503 # - 170 PRACTICAL CHEMISTRY. !■ to form acids, and the rest unite with these same elements to form bases. It has been found impossible to define a metal except as an element that replaces the hydrogen of an acid to form a salt, consequently, the division into acid-producing and base-producing practically coincides , with that into non-metals and metals, but the former terms are much more suggestive. More important still is the classification of the elements into a number of natural groups or families. One of these families at least, the student must have already noticed while study- ing the acid-producing elements. Those which we have considered in whole or in part are : — Chlorine Family. Sulphur Family. NiTROGKN Family. Carbon Family. Chlorine. Sulphur. Nitrogen. Carbon. Bromine. Oxygen. (?) Phosphorus. Silicon. Iodine. Selenium. Arsenic. Fluorine Tellurium. ' Antimony. Bismuth. Oxygen is generally classed with the sulphur family on account of the resemblance existing between the oxygen and the sulphur compounds, but oxygen in reality differs widely from the other members of the family. Sele- nium and tellurium are rare elements, and we can only glance at some of their analogous compounds and com- mon properties. § 211.— Selenium Se, 79. This element never occurs native, but is found as sele- nides of lead, silver and copper. Amorphous, crystalline, and vitreous modifications of it are known. In the form cf a powder, it has a reddish colour, but flowers of sele- TELLURIUM. 171 nium, analogous to flowers of sulphur, are scarlet. The formulae of its compounds when tabulated shew their simi- larity to those of sulphur. Formulae. Names. Remarks. HaSe Hydrogen selenide. A poisonous gas. SeOa Selenium dioxide. Ha Se O3 Selenious acid. Ha Se O4 Selenic " These acids form salts, called selenites and selenates respectively. Knowing how the analogous compounds of sulphur are formed, how would you expect to be able to form the above named compounds ? Endeavour to answer this question, and then ask your teacher if you are right. § 212.— Tellurium Te, 125. (?) Tellurium occurs native in very small quantities, and also as tellurides of some of the metals. It is a bluish- white, very brittle solid, with a metallic lustre, and specific gravity of 6.24. Its principal compounds are ■, — Formula. Names. Remarks. HaTe, Hydrogen Telluride, A poisonous gas. TeOa, Tellurium dioxide, A white solid. HaTeOa, Tellurious acid, TeOa, Tellurium trioxide, Orange-yellow solid. HaTeO*, Telluric acid, wtm 'f f r 172 PIIAC'TICAL CHEMISTRY. The acids are formed by oxidizing the metal and unit- ing the oxide with water. Note resemblances to the formulae of compounds of sulphur and selenium. How would you expect to be able to form these compounds ? What relation exists between the atomic weights of these three elements ? Which atomic weight is a mean of the other two ? Do the properties of these elements vary with the variation in their atomic weights ? The advantage of classifying the elements into groups naturally resembling each other, is two-fold. In the first place, it enables us to link together the almost innu- merable facts of chemistry, and thus remember them ; and in the second place, it enables us to grasp more readily the fundamental principles of the science. ^jl!I iiii CHAPTER XXXn. § 213.— The Nitrogen Family. Two members of this family have already been fully considered ; the other ones are arsenic, antimony and bismuth. In studying the sulphur family, we found that tellurium was a solid element possessing a metallic lus- tre, and for this reason sometimes classed as an imper- fect metal. In the nitrogen family we have the same thing. Nitrogen and phosphorus are classed as metals ; arsenic is said to be near the border line between metals and non-metals ; antimony and bismuth are usually called metals. A very superficial examination of the formulae of the commonly occurring compounds of these elements is sufficient to convince us of their natural resemblance to each other. 1 THE NITROGEN FAMILY. 173 Hydrides. Oxides. Oxides. Chlorides. Sulphides. NH3 N2O3 N.O, NCI3 (?) P.S3 PH3 P.O3 P^O^ PCI3 As.Sa A8H3 ASgOg AsgO, ASCI3 Sb.S, SbH3 SbaOs SbaO, SbClg Bi,S, Bi.03 Bi^O, BiCl3 m Arsenic occurs in nature in combination with metals ; for example, with iron, copper, &c. The element pos- sesses a metallic lustre : is not itself poisonous, but all its compounds are. Symbol As, Atomic IV t, 75; Molecular JV'f, As,joo; Specific Wt, (as a solid), ^.6. Experiments. 1. Take some arsenic trioxide AsgOg (known as " ar- senic " in the drug shops) and make a pellet of it with some powdered charcoal and a drop or two of water. Place the pellet in the bottom of a hard glass test-tube, and heat to expel the water. Now make a loosely fit- ting stopper of chalk and insert so that it almost touches the pellet. Heat to redness. Arsenic forms above the chalk. Examine with a magnifying glass. What are its physical characters } 2. Heat any arsenic compound on a piece of char- coal before the reducing flame of a blow-pipe. What is the odor like ? The trioxide is formed by heating the metal arsenic in air or oxygen, the reaction, as might be expected, being similar to what takes place when phosphorus is heated in the air. 1 \ inS I \ 174 PRAOTICATi CHEMISTRY. § 214.— Arsenic Poisoning. The frequency with which criminal poisoning has oc- curred by the use of *' arsenic " has rendered it necessary for the chemist to have some thoroughly reliable test for its detection. Such a test he has in the behaviour of arsine, one of the compounds of the element. Arsine, or arseniuretted hydrogen AsHs, is a compound analo- gous in composition to ammonia, and may be made by adding a compound of arsenic and oxygen to a mixture from which hydrogen is being evolved. Experiment. Fit up a hydrogen generating flask and attach to it a drying tube. Put .some granulated zinc in the flask and pour some chemically pure sulphuric acid down the fun- nel tube. When the air has all been expelled from the apparatus, and the hydrogen lighted at the jet at the end of the drying tube, add slowly a little of a solution of arsenic oxide, AsjOs, in dilute hydrochloric acid. What change takes place in the color of the flame ? Are fumes given off*? Hold over the flame a piece of porcelain or while stoneware. The formation of a black or brown shining spot or " mirror" on the porcelain is a test for the presence of arsenic. Attach a hard-glass tube to the end of the drying tube, and heat it near its middle point with a Bunsen burner. Just in front of the heated part there will form a layer of metallic arsenic. Explain its formation. This method of testing for arsenic is known as Mansh's test. In applying it, great care must be taken that the chemicals used do not contain arsenic, otherwise, of course, the test would be of no value. § 215. — Antimony. Ant.mony is another member of the nitrogen and phosphorus family. It usually occurs in combination ANTIMONURETTKD HYDROGEN. 175 with sulphur as stibnite, Sb2S3. The element is a bluish- white metallic looking substance ; remains unchanged in- dry air at ordinary temperatures, but when heated to redness, it burns with a white light, and forms the tri- ojcide SbaOa. Atitiinony — Symbol y Sb. Atomic Weight, 120. The metal is used principally in making alloys, but its compounds are used in maiiy pharmaceutical pre- parations. § 216. — Antimoniuretted Hydrogen SbHg. This compound is analogous to arsine, and it is made in the same way. Another name for it is stibine. Experiments. 1. Prepare some stibine, using "l-e same apparatus as in preparing arsine, but adding tartar emetic solution instead of " arsenic " to the generating flask. The pro- perties of stibine resemble those of arsine. 2. Introduce a piece of porcelain into the stibine flame and notice the antimony spot or " mirror." How does it differ from the arsenic mirror } Powdered antimony combines directly and energeti- cally with chlorine, bromine, and iodine. The terchlo- ride of antimony can be prepared by dissolving finely powdered antimony in strong hydrochloric acid to which a few drops of nitric acid has been added. When com- pletely dissolved, evaporate to a thick syrup. This is " butter of antimony." The tercJUoride decomposes on being thrown into water, forming hydrochloric acid and antimony teroxide. Try this experiment. § 217.— Bismuth. This element occurs mostly native. It is obtained from its ores, bismuth ochre, BigOs, and bismuth ite, BijSs, by heating them and allowing the molten metal to run off. Hi: ii! 176 PRACTICAL CHEMISTRY. Bismuth — Symbol^ Bi. Atomic Weighty 210. Bismuth is hard, lustrous, and brittle, with a reddish tint. Its alloys expand on solidifying, and are, there- fore, used in nnaking delicate castings, and in electrotyp - ing and stereotyping. " Fusible metals " are alloys of tin, lead and bismuth, and get their name from the fact that they melt at -a vciy low temperature. The chief compounds of bismuth are the following : — FoRMULiE. Name;",. Remarks. Bi^Oj Bismuth dioxide. Bi^O, trioxide, • . . Bi,0, pentoxide. Bi(N03)3+3H,0 nitrate. BiN03(0H)a sub-nitrate. Used in medicine. Bi.Sg trisulphido. The trioxide is the chief ore of the metal. It is used as a pigment. Knowing how the oxides of phosphorus are formed, how would you expect to be able to form the trioxide of bismuth ? Experiments. 1. Dissolve a little bismuth in very dilute nitric acid. On evaporation, nitrate of bismuth forms in colorless deliquescent crystals. 2. Dissolve some of the crystals obtained in the last experiment in a very little water. To the solution thus obtained, add a large quantity of distilled water, and subnitrate of bismuth is precipitated as a white powder. § 218.— Test. The result of the preceding experiment is, as far as it goes, a test for a soluble salt of bismuth. If, to the BORON. 177 filtrate of the last experiment, sulphuretted hydrogen be added, a black precipitate is formed, soluble in hot nitric acid. QUESTIONS. 1 What is the gradation in the specific gravity of the solid members of the nitrogen family ? 2. Point out a similar gradation in their metallic character. 3. In what order does the chemical energy of these five elements stand as compared with the order of their atomic weights ? 4. What physical property do antimony and bismuth communicate to their alloys ? 5. Shew how the physical properties of these elements affect the physical properties of their compounds. CHAPTER XXXIII. §219.- -Boron. The next element which we shall consider is boron. It has been variously classified ; oftenest with silicon and carbon, but sometimes with the nitrogen family ; and still more recently, in accordance with the periodic law, it has been classified with aluminum. Boron is a non-metal. As one of its commonest compounds (borax) is fre- quently used in testing for the metals by means of the olow-pipe, we shall begin our study of the element by investigating some of the properties of this compound. last thus and vder. as it ) the § 220.— Borax. Borax occurs native in California, forming massive beds that are probably the bottoms of dried-up lakes of long ages ago. It is much used in the arts as a flux, because it prevents tht formation of oxides when metals are soldered or welded together at a high temperature. It has the formula Nao B. O7 + lo HaO. 13 H fi' If r-n i ill 178 PRACTICAL CIIKMISTUY. Experiments. 1. Solder A short piece of platinum wire into a small glass tube about two inches long. Make a loop on the end of the wire ; moisten in water, and then dip in powdered borax. Heat the loop and adhering borax in a Bunsen gas flame, or in the blow-pipe flame of an al- cohol lamp, and continue the heat until a clear bead is obtained. Now remove from the flame, and when cool dip into a solution of a cobalt salt. Heat again. What color has been given to the bead ? 2. Repeat the experiment with fresh borax, and when a clear, cool bead has again been obtained, dip it into a solution of a copper salt and heat. What colour does the copper salt give the bead .? 3. Prepare new beads, dipping them each time into a different solution of some metallic salt. Use separately solutions of iron, nickel, manganese, and chromium. Tabulate the results obtained. § 221.— Boracic Acid. The student can scarcely have failed to notice that the weaker acids are sometimes prepared from their salts by decomposition and substitution, the acid radicle of a strong acid displacing the weaker acid radicle of the salt, and giving rise to two new compounds — an acid and a salt. For example, nitric acid was prepared from nitre and sulphuric acid ; hydrochloric acid was pre- pared from common salt and sulphuric acid ; and car- bonic acid H2CO3, from limestone and hydrochloric or sulphuric acid. This same principle is used in preparing boracic acid, or boric acid, as it is sometimes called. Experiment. Make a hot saturated solution of borax in an evapo- rating dish. Then add about half the volume of strong hydrochloric acid. Allow the mixture to cool. Collect and wash the crystals which form. They are boric acid, and when pure have the formula H3BO3. t li iilL 1\ TESTS FOR MORATE8. 179 that salts of a the acid rom Dre- car- c or ring Write the equation expressing the reaction. The acid, whose preparation has just been described, occurs dissolved in the waters of certain lagoons in Tuscany. In the vicinity of the lagoons are volcanic jets of steam, and the heat of these steam jets is used to evaporate the water which contains the acid. If boric acid be heated to redness for a long time, water is driven off and boric trioxide B2O3 is formed : — 2H3BO, =3H20-f B^Oa Boron Symbol^ B. ; Atomic Weight, 11 ; Specific Weight (Crystals), 2.^. The element can be prepared from the trioxide B0O3 by fusing it at a high temperature with the metals potas- sium or sodium. Prepared in this way, the element is a dark-brown, odorless, tasteless powder, of no practical importance whatever. § 222.— Tests for Borates. Experiments. 1. Make a solution of the free acid and immerse in it a strip of turmeric paper ; then immerse the paper in dilute hydrochloric acid. Observe the change. Note how the change that takes place in this case differs from that which occurs when the turmeric paper is dipped in an alkali and then in hydrochloric acid. 2. Make a solution of borax in hydrochloric acid, and *^o the solution add alcohol in excess. Warm. Ignite the solution. The color imparted to the flame is a char- acteristic test. apo- rong llect acid, § 223.— Aluminum. This element has already been dealt with in studying clay. Its classification, like that of boron, is difficult, some writers placing it by itself, and some assigning tin 180 PttAOTICAL IJIIKM18TRY. it to the iron, or manganese family. For the reason already assigned, we shall discuss it with boron. The element is without doubt a metal. Its leading compounds not already referred to are the following : — f YOUMVLJE. Namkh. Remarks. NaeAl,0«(?) Sodium aluminate. A1,(S0J, Aluminium sulphate. :. I i A1,(0H)„ *' hydroxide. Al.,rO,(OH)3 4- HaO Turquoifl, a phosphate. There are other phos- phates. There are besides these compounds many varieties of the almns. Of these the principal are : — K2Al2(SO,), + 24H20; Ag^A^SO,), + 24H2O ; and (NH^)^ Al2(S04)4+24H2O. The first of these, potash alum, and the last, ammonia alum, are the most important. The alums are prepared mostly from shale (a silicate of alumina) and iron pyrites, by burning them in heaps, when aluminic and ferrous sulphates are formed. From this mixture, alumic sulphate is dissolved out. To this, potassic or ammonic sulphate is added, according as it is desired, to make the one or the other kind of alum. The alum Js purified by re-crystallization. The alums are a group of compounds all 3i*milar in properties and in composition ; they are f»lso isomor- phous, and contain much water of crystallKation. Experiment. Heat a small piece of alum on a sheet ff mica, and observe the loss of its water of crystallization. The hydroxide of aluminium is d »veak acid as well as a base. TEST. 181 Experiment. To a solution of alum add, little by little, some caustic soda. Aluminic hydroxide is first precipitated and then redissolved in the sodic hydroxide solution, forming an aluminate of soda. % 224.— Tests. 1. The precipitate formed on adding ammonia to a soluble salt of aluminum is characteristic. Ammonic sulphide gives a similar precipitate. 2. If the compound containing the aluminium be insoluble, it may be tested for by moi.stening the com- pound with cobaltous nitate Co(N03)2, and heating with the blowpipe on charcoal. The color imparted is characteristic. CHAPTER XXXIV. 225.— Extraction of Metals. Before proceeding further with the consideration of the metals, it may be useful to refer briefly to some of the general processes of extracting them from their ores. In most cases they occur combined with other elements. Except the ores of the alkali metals, all ores are insoluble in water. I Copper, gold, silver, platinum, mercury &c. occur native. 2. Sodium, potassium, tin, zinc, iron, manganese, antimony, nickel and some others are obtained by heating l:heir ores with coal or charcoal. This process is called reduction. 3. Lead, copper and bismuth are obtained by their ores being at first partially oxidized, and then subse- quently fused. 4. Aluminium, magnesium and calcium may be obtained by heating with sodium or potassium. ¥ n |i.i I 'r 11 i» i'f I ill! ii is 182 PRACTICAL CHEMISTRY. 5. l^iiriiim, calcium, strontium and lithium may be obtained by the electrolysis of their fused salts. § 226.— Compounds of Metals. The derivatives ofjthe metals are very numerous, and may be conveniently classified as follows :- — 1. Compounds with chlorine, bromine, iodine and fluorine, knov/n as c/i/orides, bromides^ iodides and Jluo- rides respectively. 2. Compounds v/ith oxygen — oxides. These may be subdivided into three classes, according to their chemical properties — (a) base-forming oxides ; (b) indifferent oxides ; and (c) acid-forming. There are very few of the latter. 3. Comoounds with oxygen and hydrogen — Jiy- droxides. 4. Compounds with sulphur, and with hydrogen and sulphur, called siilpJiides and Jiydrosulphides respectively. 5. Compounds with nitric and nitrous acids — nitrates and nitrites, 6. Compounds with chloric and chlorous acids, &c., or the chlorates, chlorites, &c. 7. Compounds with sulphuric and sulphurous acids, or sulphates and sulphites. 8. Compounds with carbonic acids — carbonates. 9. Compounds with phosphoric and phosphorous acids, or t\\Q phosphates dL.r\(\ phosphites. 10. Compounds with silicic acid, or the silicates. 1 1. Compounds with boric acid, or the borates. You are more or less acquainted with the acids, and in acquiring your knowledge of them, frequent reference has been made to the salts which they form with the metals. In the remaining part of this book, prominence will be given to those compounds which illustrate gen- eral principles, or which are of special interest on account of their application to familiar processes. :||! CALCIUM FAMILY. 183 CHAPTER XXXV. § 227.— Calcium Family. This group is frequently spoken of as the metals of the alkaline earths. With them is frequently associated magnesium, especially for analytical purposes. Con- sidered from the point of view of the periodic law, zinc also might be studied with the calcium family. We shall, however, first study calcium barium and strontium as a family, and afterwards treat of magnesium and zinc. , and ence I the lence gen- t on § 228.— Calcium. We have already studied to some extent a few of the compounds of this element. We have seen that it exists in carbonate and phosphate of lime. It is found also in gypsum as a sulphate CaS04, and in fluorspar as a fluo- ride CaFg. The metal may be extracted with difficulty by fusing calcium iodide with metallic sodium in closed iron retorts. Calcium — Symbol, Ca ; Atomic Weighty ^o. The most important compounds of this metal are : Formulae. Chemical Names. Common Names. OaClg Calcic chloride OaO « oxide Quick lime Oa(OH)a (< hydroxide Slaked limo CaOOg (( carbonate Limestone CaSO^ « sulphate Gypsum Ca(010)2 (( hypoclJorite Ca,(PO,), « phosphate In apatite. ^ j iie i ' l i eBBB imr. '" i "■I 'l MU n ' M I: iii fl 184 PRACTICAL CHEMISTRY. Of these compounds we need study only the chloride and the sulphate, as we have already sufficiently consid- ered all the other compounds when treating of phospho- rus and lime. Zhxperiment. Dissolve 1 5 grams of limestone or marble in hydro- chloric acid. When all effervescence has ceased, evaporate to dryness. The residue is calcium chloride. Expose a few pieces of it to the air. What change takes place in it ? To what use have you seen this sub- stance put in the laboratory ? Add some sulphuric acid to it and explain what takes place. Try to symbolize the reaction. Calcium sulphate, CaS04, is found native in the form of selenite or gypsum, CaSO* + 2H2O. This mineral, when powdered and heated, loses its water of crystalli- zation and is known as Plaster of Paris — a substance capable of again uniting with water and forming a firm solid. The process of solidifying with water is known as " setting." The use of plaster of paris is as a finishing coat in plastering the interior of houses, and in making casts. Calcium sulphate, as we have already seen, is soluble in water, and imparts to it the property known as permanent hardness. Experiments. 1. Prepare some calcium sulphate by adding sul- phuric acid to calcic chloride or calcic carbonate. Evaporate the solution to dryness. Dissolve some of the sulphate thus formed in water. Is it difficultly or easily soluble in water ? Add some carbonate of soda to the sulphate solution. What is precipitated ? How can permanent hardness in water be removed ? 2. Heat some powdered gypsum to a red heat in an open vessel. Examine what is left. Try whether it will "set" when mixed with a little water, so as to form a paste See whether gypsum will act in a similar manner. TEST. 186 Gypsum is frequently used as a fertilizer by farmers ; but is not nearly so valuable for this purpose as calcium phosphate, Ca3(P04)2, a salt which was partially dis- cussed in treating of phosphorus. The normal phos- phate, that is, the one in which all the hydrogen of phosphoric acid, H3PO4, is replaced by calcium, is inso- luble, and is, consequently, not easily taken up by plants ; but when treated with a certain proportion of sulphuric acid, a mixture of calcium sulphate and mono-calcic phosphate, Cali4(P04)5;, is formed — a valuable fertilizer known as superphosphate of lime. C%(P04)2 + 2 H,,S04 = 2 CaSO* -j- CaH4(P04)2. § 229.— Test. Experiment. Dip a piece of platinum wire into a soluble salt of calcium, and then place in a non-luminous flame. The color imparted to the flame is distinctive. sul- nate. of ly or soda How n an er it form nilar § 230.— Barium. Heavy Spar, BaS04, is the most abundant ore of this metal. The ore is used for weighting paper and as a paint, but the metal itself is not used in the arts. Its symbol is Ba, and its atomic weight, 137. Its com.- pounds closely resemble those of calciurri, and may be tabulated as follows : — Formula. Chemical Namk. Common Name. BaO Barium monoxide BaOj <( dioxide Ba(OH)j « hydroxide Caustic baryta BaCla (( chloride BaSO^ it sulphate Heavj'^ Spar Ba(N03)2 (( nitrate BaCOy (t carbonate In Weatherite I 186 PRACTICAL CHEMISTRY. ii sl ■ i H • III I ihii Barium monoxide has, in recent years, been put to a peculiar use. When heated in air to a dull red heat, it takes up oxygen, forming the dioxide, and this when heated to a still higher temperature yields up its oxygen and returns to the mon-oxide. In this way oxygen has been extracted from the air. Barium dioxide is interesting for another reason. It is used in making hydrogen dioxide. When treated with sulphuric acid the reaction is : — BaOa + H2SO4 = BaSO^ + H2O2. This same reaction may be obtained with another acid — hydrochloric. Compare this action with that of manganese dioxide on hydrochloric acid. § 231.— Preparation of Compounds. Knowing how calcium hydroxide was formed, how would you expect to be able to form barium hydroxide ? Given barium carbonate, BaCOs, and other necessaries, how would you prepare barium chloride and barium nitrate ? The two latter compounds are used reagents in chemical analysis. as § 232.— Test. The flame test for this element is very characteristic. Experiments. 1. Make a solution of the chloride or nitrate of barium, dip into it your platinum wire loop, and then place in the non-luminous flame of a Bunsen burner or spirit lamp. 2. Take a little of the solution used in the last experiment and add to it a few drops of sulphuric acid. The color of the precipitate that is formed, coupled with the fact that it is insoluble in all acids, is a test for barium. STRONTIUM. 187 § 233.— Strontium. The minerals celestine, SrS04, and strontianite, Sr CO3, are the commonest sources of the element. Its symbol is Sr, and atomic weight, 87.2. Strontium may- be isolated in two ways : by the electrolysis of its ciiioriae, or by heating the chloride with an amalgam of soomm. The strontium amalgam which is thus formed is washed, dried, and then heated in a current of hydro- gen. The metal is yellowish in color, malleable, oxidi- zable in air, and burns when heated, forming an oxide of strontium. Its principal compounds are : FoRMULAK. Names. Remakks. SrCOg Sr(N03), Strontium carbonate. " nitrate A mineral Strontium carbonate can easily be prepared in the laboratory by precipitation. Experiments- 1. Dissolve some strontium nitrate in water and add to this a solution of sodium carbonate. Strontium car- bonate will be precipitated. What other salt will be formed in solution. Write the equation. 2. Try to prepare the carbonate by using other soluble salts of strontium, with solutions of other alka line carbonates. 3. The nitrate can easily be prepared from the car- bonate by treating it with nitric acid. Try to do this as an experiment. Write the equation. The nitrate is much used in making red fire on the stage. ^^ (.1 U li I l.'l r m 11 IIP 188 PRACTICAL CHEMISTRY. Experiments. 1. Mix very care/71 //y equal parts of finely powdered and thoroughly dried strontium nitrate and chlorate of potash, with an equal bulk of powdered shellac. No rubbing must be used, or an explosion will occur. The ingredients must be powdered separately and then mixed on paper. Ignite the mixture. 2. Repeat the foregoing experiment, using barium nitrate in place of strontium nitrate. What change in color is obtained ? § 234.— Test. The flame test is usually sufficient to recognize a salt of strontium. Experiment. Moisten any chemically pure salt of strontium with hydrochloric acid and heat on a platinum loop in the non-luminous flame. QUESTIONS. 1. What relation exists between the atomic weights of calcium ba- rium and strontium ? 2. Mix salts of barium and strontium and apply the flame test to the mixture. Which metal colors the flame first ? . Would you expect this result from a study of their atomic weights ? 3. Would you expect analogous results if you mixed a calcium with a strontium salt ? Try the experiment. 4. What weight of water will be needed to slake 100 grams of quick- lime? 5. Explain how calcic carbonate acts as an antidote in poisoning by mineral acids. 6. How would you prepare baric nitrate from baric chloride. 7. Baric sulphate is often used by painters as a substitute for white lead. Why is it preferable ? 8. What substances are formed when solutions of sodic sulphate and baric nitrate are mixed ? • 9. C'Ompare by means of actual experiment the solubility in water of calcic sulphate, with that of barium sulphate, and with that of stron- tium sulphate. I THE MAGNESIUM FAMILY. 189 CHAPTER XXXVI. § 235.— The Magnesium Family. This family consists of but two elements, magnesium and zinc. In some respects magnesium resembles the calcium family, in othsr respects, it appears related to cadmium. It exists abundantly in nature. Its well known and naturally occuring compounds may be thus tabulated : — white tte and iter of I atron- Formulae. Common Name. Remarks. MgCOa. Magnesite. (MgCa)C03. Dolomite. Mountain limestone. MgSO^+HaO. Kieserite. (MgCa)Si03. Asbestos. Mg2Ha(Si03)„. Meerschaum, The sulphate occurs as Epsom salts in some medi- cinal springs, and the chloride is a constituent of sea water. The metal is usually prepared by treating magnesium chloride with sodium at a high temperature. The sodium displaces the magnesium. Write the equation. Magnesium, Mg ; A' I. IV't., 2^; Alelting Ft., 750'. Magnesium is a silver-white metal with a high metal- lic lustre ; it changes slowly in air forming an oxide. It does not decompose water unless heated to 100°. How does this action of the metal compare with that of sodium and potassium ? The metal is used in pyro- techny, photography, and signaling. -3S" II H'-.i mi. |: wm J ; 1 1 '1 \ r-i^knli ' !'' HH 1 11 I^HH i'/' 1 (iit ' ^ 1 1' 1 ^' : ' T." 1 \ V fii:; ;i i i ^^ :J i % ip: ii'ji' ■ ; 1 u^ 190 PRACTICAL CHEMISTRY. § 236.— Impcrtant Compounds. In addition to the naturally occurring compounds already mentioned, the following are important : — Formulae. Names. Remarks. MgO. Magnesia. Used in medicine. MgCl,. MgSO^ + TH.O. Magnesium chloride. Epsom salts. Used in dressing cotton goods. Used in medicine. MgCOg. Magnesium carbonate. Used in medicine and as a f;ice powder. § 237.— Preparation. The oxide — a fine white variety of it called magnesia usta — is generally prepared by heating precipitated magnesium carbonate. A hydroxide of this is obtained by adding water: — MgO+H20=Mg(OH)2. Compare these reactions with the burning of limestone and the slaking of quicklime. Experiment. Take a piece of magnesium ribbon ; set fire to it and catch the white powder that forms. What is the com- position of the powder .<* Add water to it, and test with red litmus paper. How can you prepare Epsom salts from this white pov/der. Try it. Epsom salts are generally prepared from the mineral kieserite by boiling with water. The insoluble kieserite (MgSOi+HaO) takes up more water and changes to the very soluble Epsom salts. The carbonate may be prepared by precipitation. Experiments. 1. Make a hot solution of Epsom salts and to it add a hot solution of sodic carbonate. Basic carbona.e of magnesia will be precipitated. Filter, wash, and dry. Examine the salt. 2. Repeat this experiment, using cold .solutions instead of hot ones. What different result is obtained ? TESTS. 191 3. Try to prepare the carbonate by passing carbon dioxide into a mixture of magnesium hydroxide and water. 4. Try to prepare magnesium chloride from the oxide and hydrochloric acid. § 238. -Tests. Experiments. 1. To a soluble salt of magnesium add a solution of ammonic carbonate. If the precipitate which forms is soluble in acids and in ammonium chloride solution, magnesium is indicated. 2. A solution of di-sodic phosphate (with ammonia) added to a soluble salt of magnesium, gives a character- istic precipitate after stirring the mixture with a glass rod for a few minutes. The solutions must be dilute, and should be mixed in a test-tube. A compound of magnesium insoluble in water must first be brought into solution, by dissolving some of it in hydrochloric acid. Then proceed as in (i) and (2). § 239.— Zinc. The principal ores of zinc with their approximate formulae and chemical names are best seen in a tabu- lated form : — Formulae. COMMOJN Names. Chemical Kames. ZnCOs. Calamine or zinc spar. Zinc carbonate. ZuaSiO*. WiUemite. " silicate. ZnS. Zinc blende. " sulphide. ZnO. Red zinc ore. ZnO.FE,U;i. Franklinite. 192 PRACTICAL CHEMISTRY. The metal is extracted by reduction. The ore is roasted and ground fine, mixed with coal dust, and heated in earthenware retorts. The metal, being volatile, distils over and is condensed in iron tubes. (Indicate the reaction by an equation.) Commercial zinc con- tains lead, arsenic and other metals as impurities. At different temperatures, it exhibits different properties. Brittle at ordinary, and at high temperatures, it is nevertheless quite malleable at ioo° to 150". It does not tarnish in dry air. As we have seen, it dissolves in the mineral acids, and evolves hydrogen. Its chief use is for galvanising iron, that is, covering it with a thin coating of zinc. It is used extensively also in making batteries. Mixed with copper it forms brass^ and this again when alloyed with nickel forms German silver. § 240.— Important Compounds. FOKMULAE. Names. Remarks. ZnO Zinc white Used as a paint. ZnCla " chloride A caustic in surgery. ZnSO^ + THaO ZnS ' ' sulphate (white vitriol) " sulphide Used in medicine and in dyeing. ZnCOg " carbonate Knowing how magnesium oxide is formed, how would you expect to be able to prepare sittc oxide ? Experiments. I. Place a small quantity of zinc carbonate in a small porcelain crucible and heat in a bunsen flame for 15 minutes. The residue is zinc oxide. Note the change in color when it has cooled. f'/: m TESTS. 193 small for 15 lange 2. Given pieces of zinc, hydrochloric acid, and all necessary apparatus, how could you prepare zinc cA/o- ride ? Describe the salt. 3. Given metallic zinc, sulphuric acid, and necessary apparatus, how could you prepare zinc sulphate ? De- scribe the salt. 4. Pass sulphuretted hydrogen into separate solutions of zinc chloride and zinc sulphate. What solid will be precipitated ? Indicate the reaction by equations. 5. Given zinc sulphate and sodic carbonate, how could you prepare zinc carbonate ? Try to prepare some. All the soluble salts of zinc are poisonous. The chloride — used in soldering — is sometimes the cause of poisoning from being contained in canned goods. The sulphate is the analogue of magnesic sulphate. It is obtained on a large scale by heating the sulphide in contact with air. Write the equation. § 241.--Tests. Compounds Oi zinc insoluble in water must be dis- solved in hydrochloric or sulphuric acid, and then tested for as follows : — Experiments. 1. To any soluble salt of zinc add ammonia. A white precipitate indicates zinc. 2. Ammonic sulphide gives a similar precipitate, soluble in dilute hydrochloric acid. QUESTIONS. 1. How would you distinguish white vitriol from Epsom salts ? 2. When sodium hydroxide solution is added to magnesic sulphate solution a white precipitate falls. What is this precipitate ? Write the equation. 3. In what way does calcic carbonate act as an antidote to poisoning by zinc chloride ? 4. How does the solubility of magnesic hydroxide in water differ from that of calcic hydroxide in water ? 5. Point out resemblances between the compounds of magnesium and zinc (a) in composition ; (6) in physical properties ; (c) chemical behaviour. 14 M II tim. 194 PRACTICAL CIIEMISTIIY. CHAPTER XXXVII. § 242.— The Alkali Family. The two best known members of this family are po- tassium and sodium, and as these metals are obtained from the alkalies, caustig potash and caustic soda, the family is often called the metals of the alkalies. The radicle ammonium NH4, is generally classed with this group, as well as three comparatively rare metals : lith- ium, rubidium, and caesium. Of course ammonium is not to be considered a true metal. All that is meant by placing it in this family is that it behaves like the metals of this group and forms compounds with acids that are exactly analogous to those of sodium and potas- sium. § 243. — Potassium. Potassium compounds are derived indirectly from the soil ; but as yet, no cheap and easy method has been discovered of separating them from other ingredients of the soil. Granitic rocks which contain silicate of potas- sium are the source of the.se compounds, and the disin- tegration of such rocks explains the presence of the derivatives of potassium in nearly all suils. From the soil they pass to pkmts, and from the ashes of these they are removed by a process of washing and evaporation. Experiment. Place some wood ashes in a tin vessel and add five or six times the bulk of hot water. Shake up well, then filter, test with litmus, evaporate to dryness. The resi- due is called potash ; when purified it is known diS pearl- ash. How would you prove that this residue is a car- POTASS H J S( COMPOINnS. 105 honatc ? I Tow would you prove that this salt is Jiygro- scopic\ wviiX deliquescent? From this substance -most of the potassium compounds are prepared. PotassiiDH — K. jg.i ; sp. wt, o.S6^. Melting pt. Potassium is m ide from the carbonate by heating it to a very high temperature with charcoal, in iron bottles. The metal distils over as a green-colored vapor, which is not to be allowed to pass into air, but is passed into receivers containing vapor of naphtha. Try to write the equation. Metallic potassium is used as a reducing agent in preparing some of the rare metals. Experiments. 1. Cut off a piece of the metal with a knife. Squeeze it between the fingers. Note the color of the newly cut surface, and any change which the color undergoes immediately after being cut. Explain such change. 2. Throw a small piece of it on some water placed in the bottom of a large bottle. What phenomena result. Explain them. Note the color of the flame and dis- tinguish it from that of sodium. 3. Try the effect of pieces of potassium upon other liquids. 4. Devise an experiment to shew that potassium decomposes carbon dioxide and forms potassium car- bonate, K2CO3, and carbon. Write the equation. § 244.— Potassium Compounds. The following compounds of potassium are import- ant : — ii III! lOfi PRACTICAL CHEMISTRY. FoKMUI.^K. Names. Kbmakks. K2CO3 Potassium carbonate Used as a medicine. HKCO3 KHO Hylain the peculiar action of lead with acid solvents. 8. What are the different methods of forming salts ? 9. Test whether the lead salts formed are soluble (1) in water, (2) in the acid from which they are formed. 10. Shew the similarity in structure between Pb (NOglo and KNO3. 11. How many kilogr. of litharge can be obtained from .S7.1 kilogr. of lead, and what volume of oxygen would be absorbed in the process ? 12. Explain the constitution of the blue line at the edges of the gums in cases of chronic lead poisoning. 13. Wliy are not plumbers, who handle metallic lead only, subject to lead poisoning ? u^ I; ■, m' h li \i 212 PRACTICAL CHEMISTRY. 14. What is the color of the precipitate when sulphuric acid is added to a lead salt ? 15. What is the best chemical antidote for lead poisoning? Explain the reason why. 16. How can you distinguish a stannoua from a stannic compound ? 1 7. What danger is there in eating canned goods that have been closed with adder ? Explain. 18. What is tin salt ? What is it used for ? 19. What are the members of the tin family ? What relation, if any, exists among their atomic weights ? Point out resemblances between the stannates, carbonates and silicates. 20. What is block Hn? What is tin foil ? 21. An inferior quality of tin ware is made from an alloy of tin and lead. What danger in using such tin ware for cooking utensils, or for cans for preserving fruits ? CHAPTER XXXIX. § 263— Platinum Family. This family consists of four members, platinum, iri- dium, osmium, and gold. The first is by all odds the most important element to the chemist ; " without planti- num," said Liebig, " the composition of most minerals would have remained unknown*" It occurs mostly alloyed with small quantities of the rare metals iridium, palladium and osmium, and is found in the Ural m.oun- tains, in the Rocky mountains, in Australia and in South America. It is extracted from its ores by dis- solving it in aqtia regia. The platinum chloride thus obtained (contains some iridium) is precipitated with ammonic chloride forming a double chloride of platinum and ammonium (N 1^4)3 Pt Cle. This is fused in lime crucibles and yields metallic platinum : — (NH,). Pt Ch = 2NH,a-V Pt + 2Cl^ Experiment. Examine a piece of platinum wire, and describe its physical appearance. Test its solubility in aqua regia. TEST. 213 Platinum is used for making crucibles, and for some parts of delicate philosophical instruments. The wire and foil are in constant use in the chemical laboratory. Small weights are generally made of platinum. Platinum Pt, At, JV't. ips, Sp. IV't. 21.1s. The metal resists the action of each of the mineral acids if taken separately; but it is attacked by the caustic alkalies when highly heated, and by the oxides and sulphides of lead, copper, bismuth and a (ew others. It possesses the wonderful property of condensing gases upon its surface, especially the spongy platinum. The difficulty of extracting and working platinum renders it very expensive. Its salts are numerous but compara- tively unimportant. § 264— Test. Experiment. Dissolve the substance supposed to contain platinum in aqua regia ; expel any surplus acid, and add ammonic chloride. A yellow crystalline precipitate indicates platinum. Spongy platinum may be obtained from this precipi- tate by heating it strongly. Direct a stream of hydro- gen gas against some spongy platinum. ^ § 265.— Gold. Gold occurs very widely distributed and nearly always pure. It is found in sand or inclosed in quartz rock. From the latter it is removed by pulverization and by the addition of mercury which readily forms an amalgam with the gold. From sand it is removed by washing in sluices. Pockets containing mercury are placed at intervals along the sluices, and amalgamation takes place in this case as in that of pulverization. The rie 214 PHACTICAL CHEMISTRY. amalgam is subsequently removed and heated, when the mercury distils away and is used over again, and the gold remains behind. Gold Au, At. Wt. igS.y, Sp. W't. ig.3. In a pure state gold is too soft to wear well, and has, therefore, to be alloyed with copper to harden it. It is the most malleable and ductile of all metals. It can be beaten into leaves jo^o of a millimeter in thickness ; I gram of the metal can be drawn out into wire i^ miles in length. Like platinum it is attacked by the caustic alkalies when hot. It combines directly with chlorine but not directly with oxygen. Gold forms two series of compounds, aurous and auric, resembling plati- num in this respect which forms platinous and platinic compounds. Their formulae, however, are difTerent, being for the oxides, Au^O and AU2O3 and PtO and Pt02; while for the chlorides the formulae are AuCl and AuCla ; and PtCl2 and PtCU. As we have already seen gold dissolves in aqua regia. The compound formed is trichloride of gold. Hi § 266.— Test. The test for gold is the formation of the compound known as the Purple of Cassius. Experiments. 1. To a solution of trichloride of gold add a drop or two of stannous chloride. Note — If the re-agent be in excess, purple of Cassius will not be formed. 2. Repeat this experiment, adding ferrous sulphate. A purple precipitate indicates gold. THE IRON FAMILY. 215 CHAPTER XL. § 267.— The Iron Family. This family includes iron cobalt and nickel, and ac- cording to Mendelejefif forms a part of a larger group of elements which includes the platinum and palladium fam- ilies. The relation of the various families and groups of elements to each other will be made clearer when we come to study the periodic law. On account of its great importance iron will be discussed at considerable length ; cobalt and nickel will not be touched upon. § 268.— Ores and Extraction. Iron is obtained native, but chiefly from the following ores : — Magnetite, Hcematite, Siderite Clay, Iro?t-stone, and Limonite. Iron, Fe At. Wt. s6 ; Sp. Wt. 'j.8. Magnetite FesO^ occurs in beds and veins of gr:at extent and thickness in the Laurentian rocks of Canada, in the counties ot Renfrew, Lanark, Hastings, and Ottawa, and isthechief ore of commercial importance in Canada. In the United States, magnetite occurs in North Eastern New York, New Jersey, and California. Haematite Fe203 occurs in many regions along with magnetite. In Canada, it is found chiefly in Renfrew and the Eastern Townships ; in the United States it occurs in great abundance in Missouri, New York and Michigan. The small island of Elba in Europe has been long famous for its haematite mines. This ore is the one chiefly reduced in the United States. It assumes differ- ent forms under different conditions, as Grape ore, Mica- ceous ore. Amorphous ore and Granular ore. \ > PRACTICAL CHEMISTRY. Olay Iron-stone occurs in the coal measures. It is the chief ore of iron in England and the coal regions of Pennsylvania. This is a spathic ore containing clay or sand, and is obtained in bands or nodules. Siderite Fe CO3 is invariably mixed with calcspar or dolomite. Limonite 2Fe203H- 3H2O. In its purest form limonite is fibrous in structure and sometimes forms concutionary masses, when it is known as hroivn hcematite ; while the less pure and more earthy varieties are known by the names of hog iron ore and iron ochre. The bog ores are found along the north shore of the St. Lawrence from Montreal to Quebec, in the Eastern Townships, and on the north shore of Lake Erie. It is very abundant in Connecticut, New York, Pennsylvania and North Caro- lina. In Europe, Limonite is worked abundantly at Erzberg in Styria, where the celebrated Styrian steel is manufactured. The process of getting the cast iron from the ore is simple, chemically, but difficult to perform in a small laboratory. The stages only will be in 'icated here. The first stage consists in the reduction of the ore to the state of oxide by roasting the broken ore along with refuse coal. Moisture and carbon dioxide are expelled in this way. The second stage consists in the reduction of the oxide to cast iron. The product of the first stage is mixed with limestone and coal in a tall, conical-shaped furnace. The oxide is reduced to the metallic state by the gases arising from this mixture. The lime acts as a flux and forms a fusible slag with the silicates present in the oxide of the first stage. Tiiis slag can be easily run off at the bottom of the furnace, and the reduced iron, which unites with carbon at a high temperature, collects on the hearth of the furnace, whence it can be drawn off and run into moulds, forming the well known pig iron^ or cast iron. ORES AND EXTRACTION. 217 It is »ns of ay or )ar or lonite onary le the )y the ■es are from nd on ant in Caro- tly at iteel is ore is small ore to with Ipelled )f the ige is laped [te by is as a lent in ly run iron, )llects Ivn ofif iron, Wrojight iron is obtained from cast iron by the puddling process, which is simply one of oxidation, by which the silicon, carbon, sulphur and phosphorus are removed. Sceel is obtained from pig or cast iron when the car- bon is reduced in amount to 1.5 or 0.15 °/q. Bessemer Steel is produced by adding pure cast iron containing about 5 % of carbon to molten cast iron from which the carbon and silicon have been burnt out The following experiments will illustrate the chief blow-pipe and other characters of the iron ores. Experiments. 1. Procure a small piece of magnetite, and test its hardness with a knife, its streak on a small piece of un- glazed porcelain, and its influence upon the magnetic needle. 2. Heat strongly a small fragment of magnetite upon charcoal before the blow-pipe. 3. Bring a hot bead of borax in contact with a few grains of finely powdered magnetite and heat in the oxi- dizing flame of the blow-pipe. What color is imparted to the bead ? 4. Heat a small fragment of haematite on charcoal before the blowpipe. 5. In the shallow cavity of a piece of charcoal place a mixture of haematite powder and soda ; heat the mixture carefully in the blow-pipe flame. 6. Heat a small fragment of siderite on charcoal before the blow-pipe. Test the residue with the magnet. 7. Introduce ^ grm. of siderite powder into a test- tube, add hydrochloric acid and apply heat. 8. Heat some limonite powder with hydrochloric acid in a test-tube. ■ ip I 'it h ■81 jH! 'IS' I'iiii !;■:'■ 218 PRACTICAL CHEMISTRY. § 239.— Properties. Pure iron is almost unknown. That used for piano wires is the purest. It is greyish-white in color. Iron when placed in a moist place becomes covered with rust. Experiments. 1. Sprinkle a few drops of water on a bar of polished steel, and then expose it to the air for a day or two. That which forms on the polished surface is oxide and hydroxide of iron. 2. Heat one end of an iron wire in the blowpipe flame. 3. Try to clinch a common cut-iron nail. 4. Heat a common cut-iron nail till it is red hot, cool it slowly, then try to clinch it. Iron in its three conditions of cast iron^ ivrought iron and steel is used for so many purposes that it is impossi- ble to give an account of them here. When heated to a white heat, iron wire softens and partially fuses. Welding is a union of two surfaces of iron in this pasty condition. Sl*1ri- ■ > < I , , i:,ji liil § 270.— Compounds. Iron forms two classes of compounds, ferrous and ferric. The former invariably tend to change into the latter, simple contact with the air being sufficient to bring th is about. The change, however, takes place very readily if an oxidizing substance, such as nitric acid or chlorate of potash, be supplied to ferrous compounds. COMPOUNDS. 219 FoRMULiE. Names . Remarks. FeO Ferrous oxide Unimportant. Fe^O, Ferric Magnetite, loadstone. Fe,0, Ferroso-ferric oxide Fed, Ferrous chloride Fe,Cl„ Ferric FeSO, + 7HaO Ferrous sulphate Fe,(S04), Ferric *♦ FeS Ferrous sulphide FeS^ Iron pyrites Occurs native as FooVa Gold. Fe(0H)3 Ferrous hydroxide Fe^COH), Ferric " Ferrous Chloride, FeCla. This substance is ob- tained when iron is dissolved in hydrochloric acid. Experiment. Introduce i grm. of iron tacks into a test tube ; add lo c.c. hydrochloric acid diluted with an equal bulk of water. Filter and evaporate to about half the bulk, then allow the solution to cool. Write the equation. Note the color of the solution. What is the black substance caught on the filter paper } Ferric Chloride, FeaClc. If we saturate a solution of ferrous chloride with chlorine gas we obtain ferric chloride — a most valuable reagent in the laboratory. It may be more conveniently prepared by adding, success- ively, a little nitric acid to ferrous chloride. The reaction is one of oxidation. 2FeCl2 + 2HCI -j- 0=FeaCl6 -+- 220 PRACTICAL CHEMISTRY. 4 n m ^rfl HjO. Try the experiment. Note how the color of ferric chloride solution differs from that of the ferrous. Ferrous Sulphate, FeSOi + 7H2O, or ^reen vitriol, is used in dyeing and ink-making. It can be prepared by dissolving iion in sulphuric acid, filtering and evap- orating slowly to dryness. Try to prepare it. Symbo- lize the reaction. Potassium Ferrocyanide, K^FeCyo, usually called yellow prussiate of potash, and Potassium Fcrricyanide, KaFeCye, or the red prussiate of potash, are both useful reagents in the laboratory in the detection of iron. Ferrous sulphide, FeS, is formed by the direct union of the elements. ; Experiment. Place a little flowers of sulphur and double the quan- tity of iron filings in a test-tube, and heat until sulphur fumes cease to come off. Iron sulphide is formed. How could you prepare sulphuretted hydrogen from this com- pound .<* Iron pyrites, FeSj, occurs native in golden yellow cubes. It is used extensively in preparing ^^'^(?;/ vitriol. Experiments. 1. Powder some iron pyrites and heat it in a long test-tube. What element is given off .^ 2. Heat strongly some powdered pyrites in an iron spoon. What becomes of the sulphur ? What is left behind ? § 271.— Tests. The blowpipe tests for the common ores have already been given ; but for any iron salt in solution the follow- ing test will be valuable : — color of the vitriol, epared I evap- 5ymbo- called yanide, , useful n. :t union e quan- [ sulphur H ovv lis coia- y el low vitriol. a long an iron is left already foUow- TR8TS. 221 ExperimentB. 1. Make a borax bead on the loop of a phitinum wire> and moisten it in a solution of an iron salt ; heat the bead again, (i) in the oxidizing flame ; (2) in the reduc- ing flame. Note the effect on the color of the bead. Potassium Sulphocyanide (KCyS) is a valuable test reagent. 2. Add potassium sulpho-cyanide to a solution of (i) a ferrous salt, and (2) a ferric salt. Note the effects on each solution. 3. Add to separate solutions of a ferrous salt, (i) ferro- cyanide of potassium ; (2) ferricyanide of potassium and note results. 4. Add to separate solutions of a ferric salt these same reagents and note results. The tests in (3) and (4) are the simplest for young students. QUESTIONS AND PROBLEMS. 1. 2. 3. 4. Compare the stroak of magnetite with that of haematite or limunite. Devise an experiment to prove that limonite contains water ? What happens when powdered limonite is heated r n charcoal ? What are the chief physical eharactei-s of cast iron ? 5. What aic the chief Viirieties of cast iron ? Whereijx consists thuir difference ? 6. How may steel be made brittle ? 7. What effect has stiong nitric acid on iron ? Try the experiment. 8. Will iron rust in water that is free from air ? Answer this question by making uu appropriate experiment. 9 How would you convert iron which is in the ferric condition into the ferrous condition ? 10. How would you prepare ordinary black writing ink ? 11. In the preparation of copperas or green vitrii»l on a large scale, moistened iron pyrites are subjected to " atmospheric oxidation." Try to explain by an equation. 12. Given a solution containing silver, lead and iron, how would you separate them 222 PRACTICAL CIIRMI8TRY. ■V't-f liili . li I ""'I 13. Explain tho maiiufaoturu of wrought iron and BoHHemer steel ? 14. What arc chief iinpuritius in iron antl how are they removed ? 15. What effect have titanium phosphorus and sulphur on the qual- ity of iron ? 16. Why must " reduced iron " be kept in well-stoppered bottles ? 17. Devise an experiment to ascertain whether iron will conduct electricity. 18. Mention facts which you have yourself observed that go to prove that iron is malleable and ductile. 19. Heat a ferrous salt, say, FeSO^ in a Bunscu .o. What oxide do you obtain ? 20. Heat a ferric salt, say, Fe,(N03)« on charcoal in the reducing flame. What oxide is obtained ? 21. Try to prepare ferrous hydroxide, and ferric hydroxide from ferrous chloride and ferric chloride, respectively. How can these hydroxides be distinguished from each other ? 22. How can it be decided whether FeOl, or FegCl, is the correct formulsB for ferric chloride ? 23. What is rouge ? How is it made ? CHAPTER XLI. § 272.— Manganese. Manganese presents some points of resemblance to iron and aluminum ; and it also resembles members of the chlorine family. It is with this latter family that it is classed in accordance with the periodic law. This metal occurs in nature chiefly in the form of the oxides, of which the dioxide MnOj is the most abundant. Manganese is not used for practical purposes. It is extracted from any of its oxides by reduction with charcoal. Manganese Mn. At, Wt. 55, Melts at a white heat. Manganese is a reddish white metal, very brittle* oxidises readily in air, decomposes warm water, and imparts hardness to iron when mixed with it in small quantities. h". COMPOUNDS. ^23 steel? ovod ? the qual- ittles ? conduct 3 to prove hat oxide reducing xide from can these he correct ance to nbers of Y that it form of he most urposes. ion with heat brittie> ter, lin s and mall § 273.— Compounds. Like iron, it forms two series of salts, the manganous and manganic, which differ from each other in very much the same way as ferrous and ferric compounds do from one another. 1 1 FoRMULiB. Names. MnO Manganous oxide Mn^Oa Manganic " MngU^ Miinganoso-manganic oxide MnOa Manganese dioxide Mn,Oy Manganese heptoxide Ha MnO, Manganic acid HMnO, Permanganic acid KMnO, Pot issium permanganate Remakk.s. A grayish green powder. May be considered as a union of first two oxides. The black oxide. Not eliminated. Of the oxides the first two are basic ; the next two are indifferent ; and the fifth, MnoOy may be considered as the anhydride of permanganic acid. Mii207 + H20 = 2HMn04. The black oxide iyro- lusite, is used extensively in the manufacture o. glass and in preparing chlorine for the manufacture of bleac - ing powder. To what other uses have you seen it ap- plied .'' If adulterated with coal dust or charcoal pow- der it becomes explosive when used for preparing oxygen. Explain how. In the mancranous salts the element is bivalent. These salts are not easily changed to the manganic ; in fact the opposite change is the one that takes place most readily. ; !i H 224 PRACTICAL CHEMISTRY. Experiments. 1. Try to prepare manganoiis sulphate Mn SO4 from manganese dioxide and concentrated sulphuric acid. 2. Try to prepare other manganous salts in a similar way. 3. Try to prepare an insoluble salt of manganese by precipitation. Write the equation and name the salt § 274— Preparation and Properties Both the manganates and permanganates readily lose oxygen ; they are therefore powerful oxidising agents and as such become valuable disinfectants and deo- dorizers. Experiments. 1. Fuse in a crucible, manganese dioxide with one and a half times its weight of caustic potash. Stir the mix- ture with a rod so as to expose all of it to the air. Pot- assic nianganate is obtained. Mix with water and evaporate. 2. The permanganate of potash may be obtained from a clear alkalifte solution of the manganate, by adding to it dilute sulphuric acid until the color changes to purple. Condy's disinfecting fluid is chiefly a solution of the permanganate in water. Both sodic and potassic per- manganates oxidize many chemical and organic sub- stances. The follovving experiments illustrate this : — Experiments. I. Pass sulphur dioxide into a solution of potassic permanganate. Explain the change that takes place by the aid of the equation : — 2KM.1O, + 5S0., + 2H.,0 - K2SO4 + 2M:n SO4 + 2H2 SO4. . 1 TEST. 2. Similary, add ferrous sulphate, sulphuretted hydro- gen, and amnionic sulphide to separate solutiors of potassic permanganale. 3. Bubble air from the lungs through more of the per- manganate solution. Note the chanire of color. 275.-Test. The manganese acids may be distinguished bv the reactions color of their salts, and by yieldin^r the obtained as follows ; — ^ Experiments. I. Make a borax bead on a loop of platinum, add any salt of manganesse and heat in the oxidizing flame Note the aifference in color when the bead is hot and when cold. 2 Repeat, adding soda and a drop of nitric acid to the bead. 3. Add dilute aqua ammonite to a solution of a man- ganese salt. Note the precipitate. It is soluble in excess. QUESTIONS. and the halogenT P^""^^^'^^* P^^^* ^^ resemblance between manganese c^f^i'^i'^ to point out resemblances between the composition of the sulphates and manganates, 3. How would you find out whether it was safe to make oxygen from pure chlorate of potash and adulterated manganese dioxide'^'' How would you remove the impurity supposing it to be carbon ? • \ ^^'}^\ potassic permanganate disinfect the air ? If not, how can air be disinfected ? 5. How could you prepare manganous hydroxide ? Try to do it ex- perimentally. ^ 6 What ischamekon mineral-. Prepare some, aild it to pure water and explain the phenomena that occur. ' III; ! M : ; l! i I If ii ii! 226 PRACTICAL CHEMISTRY. CHAPTER XLII. § 276.— The Periodic Law. We have quite frequently, when discussing the various families of elements, made reference to the periodic law. Mendelejeff, a Russian chemist, was the first to make a systematic attempt to trace the relation between the atomic weights and the chemical and physical properties of the elements. Hence it is often spoken of as Men- delejeff 's law. Muir, in his Ptinciples of Chemistry, thus states it : " We may confidently say that a large proba- bility has been established in favor of the hypothesis that the propertiesof the elements and of the compounds of each element, are periodic functions of the atomic weights of the elements. Lothar Meyer puts the general statement of the periodic law in this form : " if the ele- ments are arranged in order of increasing atomic weights^ the properties of these elements vary from member to member of the series, but return more or less nearly to the same value at certain fixed points in the series." Let the elements be arranged in the order of their atomic weights ; let this list of elements be (broadly) divided into series of sevens ; let the members of the second series be placed under those of the first, those of the third under those of the second, and so on; and let the elements contained in a vertical column be called a groups and those in a horizontal column a series!' In such an arrangement of the elements it will readily be seen that each group comprehends for the most part one or sometimes more of the natural families already des- cribed. Many familiar relationships can now be traced out. These can be seen all the more easily if certain gaps are supposed to exist in the list of eleii.enls. These gaps represent the relative position and probable atomic weights of elements not yet discovered. If the symbol R. stands for any element, its oxides and hy- drides will be denoted by RO and RH respectively. GROUPS. 227 irious c law. lake a ;n the )crties Men- y, thus proba- ^thesis )ounds atomic reneral he ele- eights. iber to irly to >> :s. their oadly) of the ose of nd let lied a ." In ily be irt one y des- traced ertain IT. en is. bable If the d hy- y- > O CD CO o C5 ft| ?o 4' Is II I II 11 3) O fcO c o o O CO 0) O) CO n II O ^ ^Oh 1/5 ;j 8 S g f^ C5 —1 C^ C-J O II u 1 33 « o II -. CO II 3^ II II to O F^ :^ II 1 o I -' II CO !« H « ■*■ .. C-J 1 ^ -f~ 1 ?1- It Q^fA O 00 73 CO f H 1—4 1 o ^'^ C5 00 1—1 CO ?1 II "^ ^1 h- S~i II '^ N 1 CI '=' II P^ CO 1 o — cq ll A-» ■H cq 11 a; C5 C5 00 •O II cc I -< 9) 1 5S 1 o ll m D P^ P3 Pi 11 II CI II II cS 1^ Ci CI II ?5 •o ^ 00 si: a CO CI X4 Z^ il II « o W Pj- CO '^5 II II ^ a Ph I GO fO Or- II w -t1 CI CO CO »o I— ( CO a a CO CJ CI o o c« !30 o o CO c« o "o CI CO C5 -ll 09 I ^ ! CO 't o hardunod without chanye of color ? 5. How would you separate the silver from tlie copper in a silver coin? 6. Account for the (constitution of the following compounds of silver : — A^.O... Ajr,(), Ay,0, and \gCl, Ai,',,Cl, and AgNO.^ 7 Is silver a monad or a diad element ? Bro- langed raphy. silver ppt. irogen § 281.— Copper. Copper occurs native both in large masses, and as disseminated grains and sheets in beds of trap and coiujioiiicrntc. It occurs in nearly every variety of ore. "Three quarters of the world's supply is derived from sulphuretted compounds of copper ami iroii which form a series of ores of definite composition." Copper pyrites contains 34.6 p.ct. of copper. Variegated copper ore contains 60.8 " Copper glance 77.2 •' Copper pyrites is of most frequent occurrence. The chief regions of supply in North America are the Lake Superior region, Arizona and Moutana. Symbol Cu, At. Wt. 63, Melting Ft. 1090°. The first product of roasting is termed coarse vie fa/. Tlie crushed and dressoil ore is roasted to drive off much of the sulphur as sulphur dioxide gis ; then the roasted ore is fused with limestone, coal and sand, when most of tlie iron, etc., is removed as silicates ; the copper as sulphide is found at the bottom of the fui-nace. The second product of roasting is termed Jine metal, when the iron is removed by oxidation. By continued roasting and oxidation, the sulphide is converted into oxide, then into the metal. This process is long and difficult inasmuch as the pure copper is very liable to unite with oxygen and form dif>xide. Experiments. 1. Hold a piece of tliin copper foil in tho outer zone of the lamp flame until a hliu'k coating ia formed ; shake it up in a test-tube with 5 c.o. of water and test with litmus. r:^ wmm 234 PRACTICAL C!HKM ISTKY. 2. Hold a pioc« of copper wire in the inner zone of the tiiniii flume. Notice w hether there ia any tlilTerence in effect l)et\veen tiiis experinieiit unjrhly witii al)out iV of its weifj^lit of jiowdcreil er in an ignition tube through which a slow stream of hydrogen is passing, and holdthedry outer edge of a test-tube contain- inff cold water close to tlie mouth of the ignition tube ; allow tlie tube to cool, then turn off the hydrojfen and examine the contents of the tube. Copper may be separated from a copper solution by the action of clean iron. Devise an experiment to prove this. Copper may also be separated by a galvanic current. Experiment. Allow two platinum slips attached to the terminal wires of a battery to dip into a copper Holut::)!!. Note the deposition of copper on the platinum slip connected with the negative pole of the battery. Copper has a metallic lustre and is red in color ; quite malleable and ductile ; and is a good conductor of heat and electricity, (Jopjjcr, on account of its toughness, is used in the manufacture (.f boilers. As long as the copper vessels are kept clean there is little danger of poison- ing from the copper salts, bat sometimes a greenish acetate is formed —verdhirU — when the metal is exposed to vegetable juices, which is exceedingly dangerou.«. Experiment. Ex]-osc to the air six or seven copper sheets piled up in layers between woollen cloths soaked in vinegar. Examine after a few days and note the verdigris formed. Copper is now used extensively in electro-plating. The alloys — brass, bronze, speculum metal and argentine —-are of great importance. Brass contains about 66 parts copper and 34 parts zinc. Speculum metal " 66 " " 34 " tin. Bronze contains copper, zinc and tin. Ar^jeutine nickel. § 282.— Compounds. Copper Sulphate or Blue Vitriol CuSO^-f-oHjO, is manufactured on a large scale by roasting copper pyrites CuFeSg at a low heat, and digesting the product with water. The copper sulphate along with the ferrous sulphate remains in solution. When all the water of crystalliza- tion is removed, the copper sulphate becomes a yellowish white powder, which becomes blue when moistened with water. Experiment. Heat gently hi a test-tube 1 j,'rm. of coppev turnings and 3 c.c. of strong sulphuric aoid. Allow the solution to «vaporate in a porcelain crucible. TESTS. 235 Copper Nitrate <'u(N()f,)3 is prepared by dissolving oopptr turn- ingH ill dilut(! nitric iV<;id. It is used as "an oxidizing agent in tlio pro- cesses of dying and calico-printing."' Try to prepare it. Cupric Sulphide <^^'uS is j)repared l)y ijassingsulphuretted-hydrogeif through a solution of cupric sulphate, when a brownish olack precipitate is formed. Cuprous Sulphide (^^'u-jS) occurs in nature as the mineral copper- glance, or chalcocite, and may be ol)tained from cupric Buli)hi(le by heating with an excess of sulphur in a hydrogen llamo. § 283.— Tests. The blowpipe tests for copper are generally characteristic. On char- coal, in the reducing thune, small reddish ])articles of copper can bo obtained. To a borax bead copper gives in the oxidizing flame a green color while hot, but bluish when cold. Many copper compounds give a green coloration to the flame. H,jS gives a black precipitate of Cu8 soluble in HNO3. NH4OH gives to copper solutions a blue color. QUESTIONS. 1. Niiiiio as many ores of coi»per as you can, j,'iviii<^ the formula for each. 2. Exi)lain the process of the extraction of the metal from the ore. 3. What is the action of copper on eich of the followinj,' bodies? (a) Sulphuric acid, (b) Anunonia, (c) Nitric acid, (d) Cliilorine. 4. How would you prepare copper oxide? How would 3'ou reduce copper oxide ? f). Ex])lain tlie formation of vcrdii^ris, Scheele's (treen. G. To what is the color of coj)per sulphate due? Prove your statement. 7. How is (topper sulphate obtained conunercially ? What occurs (a) when it is heated, (b) when ammonia is added to a solution of it ? § 284.— Mercury. Mercury occurs native in small quantities, Its chief source is (*inna- bar HgS. The most important cinnabar mines are at Almadcn in Spain, at New Almaden in California, at idria in Austria, in Peru and in Japan. Symbol Hfj ; At. Wt. 200 ; MelliiKj Pt. --40°. For commercial purposes Cinnabar is heated in a furnace where strong currents of air arc continually passing through the ore. The mercury is vaporized, and condens.'d in pipes or chambers which lead the liquid into the reservoir. The same principle is involved in the following ex- perimeuts, . i|! f''! J. i 1 1 lil; 1 f 1, 'y its action oil luoiHtLMied 1)1 iiu litiniiM paper. 3. Scoop a siiiail hole at tilt! iMid of a piece of cliar«'lafe uj)oii she mercury and s. Hf w is pure mercury obtained from Mie ore ? 16. W'.iy should ni;iny:;uiese dioxide be used witii sodium chloride in the prei»ara- tion of corrosive sublimate ? ;ed sulphuric 238 PRACTICAL CHEMISTUY. CHAPTER XLIV. iil^ § 28 7. —Chemical Analysis Chemical analysis is generally recognized to be of tv/o kinds, (1) qualitative, and ^•2) quantitative. In nuikinitate with the gronji reagents, viz., nitric, chloric, acetic, and in very dilute solutions oxalic, boric, nitrous and sulpliurous acids. CHEMICAL ANALYSIS. 239 portions of the substance^u 1 wat'r • ^)T^"i 'u '"'"'''/*^ '^^ ^'^^'-^^ acul; (4) nitro-hydr,)ehloric acid Tf^ f^L Vi'^^'^^ ('^) nitric the substance is probab y <> e of Jhe S' '" ""^^Z ^"^*^ ^'^ t'^^s^. be ested for separately, ^^^Xh^ ^:i^:fiCL:^:^ '^^ *° 7M7J1S:. rd*:^nir[rt! f-.^^^^oodwin. tions contaminL- not mor. fho.. ^„ ' I.^'V ^'^f *'^^ exami -»3^siS£™=-™"- mination of solu- their salts as 11' «f ! b\ 240 Hi.i ' J m H ,Q <^ ce K H W i-H .E- o • l-t C +3 fj^ I— t I— I E- W eg 1-5 0; S3 PRACTICAL CHKMISTHlf. 13 02 ^ S p. ■♦-> ;C is b o p< o ;i?5 1) y. 01 .g-o Q* (^ OS a p. c o b o n ifiO c C3 1 v , **^ O) rt , 1 "O O :^ 5 1 *4i 1 o T 03 ; IV > w s 1 1 tn 1X2 d O s o w p. o ■si s . .T; +a .r-< gS -m y— \ !* Cj o«- 1 :sg:" bo ;- W 3 3 X '-I to c -w Ah 01 3 O m ea p. I ? a 3 O II. o •r * -7 pti bo C — J^ '< 13 5^ lu O cS o "3 _c >i 15 d t- to bo _c °c cj 0> s 3 JB c r— d O ^1* d ^ H —, y: -*-» «-< T o ^ ' a C/J n h^ H-« ^ o -*-» o is it o -e c c m TESTING FOR THE METAL. 241 i I I. •2 01 I S 60 s •c c3 hi 3 a, in p. as o '^ s: o is bt o ■« ? d o < ^. r — 03 -u 5^ .2-3 So Sot - H CI ■♦ « e a o a 6 O H 6^ 5^ •3 >> _ ■0-2 >— ■ -^a a 1 S V -o-« ■w 4) - 3 ■3 T) .22-^ «<1 ■^5 ?» 3 K « CO o^ :j p. o o a. CO a> c be cS .= i= -J a = . en 2 y o » o C "fid lis '/> r, ■/) ft s U s. >. a rt +i OJ rn ■+J ~ 0) u 4-> E "O .'*^ 3 M ^ c rf o o . hr 3 l) ■C C 3 3 3 O .f -S oj 3 5 2 is ^ 2 « -J O, , 3^ - '- ' " .s. ^ a c. S =:5 ^ MM "^ 3 ;3 li O !» a a; o o . ■♦O •- "o -e * "oHcB -- .^ F :i B) •." y; rw •'^ f— 4 • ^« f^ M\ C iC O ►? 41 - P . £ 5; 7> cS 1) •-< r to CQ 1 CZ2 o o 4; ■SSI f. o" O ^ 0_; Ac * o o en ■*J ?4 V O ;^ .'S ? ;< s < TJ aj < 3 1 ^ V 3 o '^ >^ 40 o .TS I ^ >H^^? s J- 3 o 2= ^"--C^ - k -o I fi 4) ^ eq c ** "it^ ^ u o ■73 C d ** T! W) 5^ § W tc rt HO 1) ^ b o o r« tX) o-r 41 J3 *« -fa 2o 246 PRACTICAL CllKMlSTllY. i APPENDIX. ^! ii m m i; II § 288. —Metric System. Thift is a decimal system, lieiice its advantages over tlie English one. It was devised by the French. MEASURES' OP LENGTH. 10 decimetres (dcm.)=:l metre = H9-37 inches. 100 centimetres (cm.) -= " = " lOOOmillimetrea (mm.)= " = 1000 metres (m.)=l kilometre=,Sy370'79 inches. MEASURES OP CAPACITY. The unit is one cubic decimetre, called 1 litre=l-76 Imperial pints, or 61-024 cubic inches. 1000 litres=l kilolitre. MEASURES OF WEIOHT. The unit is tlie weight of 1 cb. Cm. of distilled water at 4''C., called I gram = 15 "432 grains. The commercial unit is 1000 gram8=-l kilogram=2-2043 lbs. Avoir dupois. The subdivision of both the litre, and the . Horon .... B . 11 7. Bromine Br . 80 8. Cadmium C.l . 112 9. ('aisium .... Cs . 133 10. Calcium . . . ' Ca . 40 11. Carbon. . • . ! C . . . 12 12. Cerium . . Cn . 1.38 13. Chlorine CI . i 35 5 14. Chromium Cr . . . ! 62 15. Cobalt .... Co . . . j 58-7 iiglish one. jrial pints, ITJ., called lbs. Avoir- I, hundred- and milli- with their Atomic Wkioht. 27.3 122 75 137 210 11 80 112 133 40 12 138 35 5 52 58-7 LIST OF ELKMENTS. 247 Namb ok Elkment. Symbol. Atomic Wkioht. It). Copper. Ou (Cuprum) 63-5 17. Didymiuni D . 147 18. Erbium . E . 170- 6 H). Fluorine F . 19 20. (^allium. Ua . 70 21. Gluciuum Ul . 9-3 22. (J old . Au (Aurum) 197 23. Hydrogen H . 1 24. Indium . In . 113-4 25. Iodine . I 127 2(). Iridium . Ir 193 27. Iron Fe (Ferrum) 56 28. Lanthanum La . 139 29. Lead . Pb (Plumbum) 207 30. Lithium Li . 7 31. Magnesium Mg . 24 32. Manganese Mn . 55 33. Mercury Hg (Hydrargyrum 200 34. Molyl)denum Mo . 96 35. Nickel . Ni . 58-7 30. Niobium Kb . 94 37. Nitrogen N . 14 38. Osmium Os . 199 3J>. Oxygen . . . 16 40. I'alladium Pd . . . 106 5 41. Phosphorus P 31 42. Platinum Pt . . . 1975 43. Potassium K (Kalium) 391 44. Ehodium Ro . . . 104-5 45. Rubidium Rb . 85.4 40. Ruthenium . Ru . 104 -5 47. Selenion Se . 79-5 48. Silver . Ag (Argentum) 108 49. Silicon. Ni . 28 50. Sodium Na (Natrium) 23 51. tStrontium Sr . . . 87 -5 52. Sulphur S . . . 32 53. Tantalum Ta . 182 54. Tellurium Te . 128 55. Thallium Tl . 203-5 56. Thorium '^^ V 234 57. Tin . Sn (Stannum) 118 58. Titanium Ti . 50 59. Tungsten W (Wolfram) 184 60. Uranium TJ . . 240 61. Vanadium V 51 62. Yttriuui Y . . . 89-6 Q". Zinc Zu . . 1 65 64. Zirconium Zr . 89.0 Some of the more ii ipjrtan t on es are ».int 'd in full-f aofid t^ ri> : UK I ''■ '• 248 PRACTICAL CHEMISTRY. I M The Science Room. The science room in a school should })e one of the largest rooms in the building, well lighted and well ventilated. If used as a chemical, and also as a physical laboratory, the working tables of the students sliould be placed along one or two sitlea of the room. In the centre should be placed desks, or benches with arms, at which pupils may sit when mak- ing observations and writing notes on experiments performed by the science master in jiresence of the class. Tank. — A large barrel will make a good tank for washing water. It should be placed in (tno corner of the room, 5 or 6 feet above the floor, and connected with one or more sinks by rubber, or better still, by iron- tubing. Gas-Chamber. — Tn another corner of the room should be placed the gas-chamber. It may be made of pine boards, and should have sliding glass doors. Convenient dimensions are 3 ft. wide, 2 ft. deep, and 4 ft. high, the top of it being connected with a good ventilating or other chimney. Working Tables. — The following are convenient dimensions : 12 ft. long, H ft. high, and 3^ ft. broad. Such tables are divided length- wise along the middle, and transversely also into compartments 2^ ft. long; conse(j[uently each table affords working room for 10 students, 5 on each side. A cut of one of the tables in use in the Kingston Colle- giate Institute is shown in the frontispiece. The tables, of course, when constructed in this manner are placed along the middle of the room. Apparatus. — Each student's compartment should be supplied with the following apparatus : — 6 test-tubes, about 4 in. long, by ^ in. diam. 2 •' "8 " by about 1 in. diam. 1 rubber cork to fit large test-tubes, and pierced with one hole for bent delivery tube. 1 test-tube rack. 1 swab for washing test-tubes. ) These two articles can easily be 1 glass stirring rod. ) made by the student. 1 piece of platinum wire 3 in. long, with its end fused into a glass tube 2 in. long. 1 piece of platinum foil, or a sheet of mica. 1 blowpipe. 1 pair tongs for test-tubes. I funue' 3 in. in diameter. I package of filter paper to fit funnel. 1 liorence flask, 4 oz. capacity. 2 glass beakers, 1 of 2 oz., and 1 of 4 oz. capacity. (Common tum- blers will do). 1 evaporating dish, 3 or 4 oz. capacity. (A small saucer will do). I Bunsen burner or spirit lamp. 1 ring stand with piece of wire gauze 4 in. x 4 in. I sand bath (a tin saucer with sand in it). 1 piece of charcoal. 1 soup-plate, which does well for a small pneumatic trough. GENERAL Al'l'AKATUS. 249 119 in the ical, and ta should hould be len raak- sd by the ^ater. It the floor, , by irou- )laced the re sliding , and 4 ft. ^ or other sions : 12 ed length- jnt3 2i ft. budents, 5 ton CoUe- of course, die of the plied with le hole for easily be iito a glass mon tum- 'iU do). Each student should provide himself with au apron, a t(»wel and *a box of matches. In schools ia which it has hevAi found inconvenient to i)rovide work- ing tallies for students, and where the teacher alone is expected to per- form allt!X[>eriments, tlu; laboratory should, in addition to the foregoing list, contain the following pieces o{ (jeiwraL apparatus. General Apparatus. A chemical balance with weights running from 50 grs. to brass, and from 500 mgs. to I mg. in platinum. A good chemical thermometer. A pipette of about 5 cc. capacity. A Eudiometer. A decomposition of water apparatus. A spectroscope. Bell jars : I'int and quart fruit jars answer well. Large Beakers, assorted sizes. Funnels, various sizes. Funnel or thistle tubes. Retorts, various sizes. Liebig's condenser. Iron mortiir and pestle. Also a porcelain mortar and pestle. Glass tubing, assorted sizes. Mercury trough. Gas-holder. Kubber corks of various sizes. " tubing •' ** " (Jas-bag. Oxyhydrogen blow-pipe. Crucibles ; porcelain, sand, or plumbago. Test-tubes ; a large stock of assorted sizes. gr. ju h. Students' Reagents. It will be found convenient to supply each student with the following named liquid reagents. None but pure chemicalf^ sliould be used. The bottles for these should have a capacity of about 4 fluid ounces, and rhould have ground glass stoppers —parafriued to prevent them from sticking. Sulphuric acid, H2SO4 ; oil of vitriol. Nitric acid, HNO3 ; ac^ua fortis. Hydrochloric acid, HCl ; spirits of salt. Acetic acid, P^CaHaOo). Ammonia, NH3 (3 volumes water to 1 of strong am.) Aninu)nic chloride, NH4('l, sal ammouiac. Potassium hydroxide, KHO ; caustic potash, (1 to 20 of water). {;*odiuni hydroxide, NaHO ; caustic soda, (1 of salt to 10 of water). rotassiuiu iodide, KI, (I of salt to 20 of water). jl \i 2rtO 1* R ACT I VA h C H K M I HT K Y . * 'Calcium nvtlroxide, r'ii(()il).j ; lime wattu'. A«l«l HO or 40 ])ai'tH of wator and Htir for 15 uiinuteH. Throw away this Bcdiitiou, and add 300 parts of watcsr and Hhaki-, and when Htill milky trun.st'et it to a glass titoi)|>crcd bottle, and allow it to stand until ckar. MagnuHium sidphate, MgSO^ ; Kpsom salts, (1 of salt to 10 of water). Mercuric chh)rid corrotjivc Huhlimatc, (I of suit to l(> of water). Silver nitratt-, AgNO,, ; (I of salt to .10 of water). [itiad acetate, IM)((!y HjjO.,).j ; (I <»f salt to 10 of water). Alc<.h(d, (',jH„(). Methylated spirits for lamp. 'i'urpentine, OjoHio- Tho student's talde shotild also l»e supplied with the following named reagents, in tho solid form. 'I'hey are most conveniently kept in '2 oz. bottles with large mouths. I^eli<|ue8cent solids should be placed in glass stop[)ta'ed bottles : Ferrous sulphate FeS0,-f-7H.^0. ('opper sulphate ('uSO^. Sodium carbonate Na.,(J().,. Potassium chlorate KCIO^. Metallic zinc. ( Copper wire, (^'opper lilings. Manganese dioxide. Sulphur. Besides the reagents already mentioned a nund>er of others are needed by tlie student. These, along with working material, may be left on a side-table, and dealt out as rc(piired. Material and Reagents. The following is a full list of all tho reagents and working material necessary for repeating the ex[)eriments detailed in this book, and no more need be ordered for schools in which the teacher performs all experiments. Where pupils have working-tables ami are expected to try the experiments for themselves, the material should be ordered in nmch larger quantities than herein specilied. Pupils, however, should be taught to use very small quantities of tho nuiterial in performing any experiment. Acid acetic 1 oz. " hv(h()C'hlonc 1 ll>. " nitric lib. " i)li()Hphoric 1 oz. " pyrogallic 1 oz. •' hulphurie lib. " tartaric 1 oz. Alcohol 1 pt. Aliun, potassic 4 oz. Ammonia 2 lb. Aiiiniouic chloride 4 oz. " nitrate . . 1 oz. Arsenic metal J oz. Arsenic white 1 oz. Antimony sulphide 1 oz. Barium peroxide ^ oz. Heeswax 1 oz. Jioric acid 1 oz. Borax 2 oz. Calcium chloride ... 1 oz. " I'arbonate 2 oz. " fluoride 1 oz. Carbon bisulphide 1 oz. (Jhloroforni 1 oz. (Mian^oal, animal 4 oz. " wood 4 oz. m i HOOK8 OK KKFKUUNCK. 201 I (I/, foil uiiil wiri>. . . . 4 1)7.. 1 (•/. 4 «)Z. PhoHphtiniH, re<1 J oz. I'latiiiiiin rof:i«Miiiiii, iiictul i ***• " Jiioail>c?iatc 2 o«. " blclirouiatc 4 oz. " bioiiildo 1 <>z. " clilorurtu 4 oz. " iodido log. nUruto 8 OS. " liydi-uto 2oz. " punii£,u(fuiiatt' .... 1 oz. t,tiil(:kliiiiO 1 11). Siitfjir 2 <)Z. Silicon (liuxldf -uU variuties fSih LT iiiUaU) 1 oz. Starch 1 oz. So/.. " Jiydrato 2 oz. (ihloridu 2 H.. uitnitc 1 H>. Stroiitiuin nitniU- 1 oz. Sulphur (roll uinl flower 1 lb. Tartar ciuotic 1 oz. Turptutiiio 1 oz. Watch -spring Wax (taiidluu utid taponi Zinc, mutal 1 lb. " clippings PhoHiihoruH, wliitr 1 oz. A f( w reag* nts luive l)oen intentionally omitted, but only these ■\vhieli can easily b<' ^nepared in the lat)orat<)ry. ('onl, varioiiH kindH. Copper, niL'tiil " Hid)>hiit)' " (ixiflf Kthor, sulphuri<'. Hold leaf (JaltMia 1 nz. (l.vnsiiin 1 oz. liKli^fo 1 oz Iodine i oz. Iron HIlnjfH 2 oz. " wire 1 oz. " Hcs(piioxide 1 oz. " Hidphatc' 1 111. " Huliihide 1 oz. ' ' p.vricos 1 oz. LitharKf 1 oz. Lead acetate 1 oz. " nitrato . J oz. " red 1 oz. " |»eroN ide 1 oz. LitnniH, pure 1 oz. Man^'ancse dioxide 1 oz. Ma^'neHiuni rilil)()n 4 ft. " Kidphate 2 oz. Metals— specimens of eo 'h ■' —ores of each Mercury, metal 3 oz. " oxide 1 oz. " bichloride 1 oz. Powder, bluailiinyr 2 oz. Books of Reference. Rook' of reference should l)e kept where pupils can consult them at pleaau'e. The following .sliould he in every school lai)oratory . — Tre' tine on Jnorijunic, Clftahirii, 3 \ols., by Uo^-icoe and Scliorlemnier. FriH'n'ineatal Cliitiuiitrii, Parts I. II., b}- I>r. Reynolds. ' be Cheitustry v/tho Fitrin, ])y Warinyfcoii. ntrodiictiun to Chemical Philofophi), by Tildeu. « University of Toronto. I. 1. Chlorine was formerly reoardej as a eonxpouiul of hydi-oehloric aeid gas with oxygen. Describe oxpurimouts, proving tliat this was an incorrect view. 2. 35-5 parts by weight of Chlorine, combine with 23 parts by weight of Sodium, 20 parta by weight of Oalcium, and .3 [)ai't3 by weight of Carbon, respectively. K.vplaiu wliy chemists consider the atomic weights of the three elemeiits as follows : Na = 23. Ca =40. (' = 12. m m PRACTICAL CHEMISTKY. 3. How may the following compounds of Sulphur he prejuired frotu the i'le/ment : Hydrogen Sulphide, Lalphur Dioxide, Sulphur trioxide, Sulphuric acid, Carbon dibulpliide ? 4. Calculate the pressure produced by 1 grm. of hydrogen confined lu a space of lOO*^" at a temperature of 273°C, and calculate what weight of z no would yield 1 grm,. of hydrogen. Z — 65. 5. Express by equations the following reactions : ((f) Chlorine on solution of potassium hydtate. (/>) Ammonium chloride on calcium hydrate, (o) Heat on ammonuiii nitrate. II. 1. 2. 3. What do you consider the objects of Chemistry as a science ? What facts lead us t() the conclusion : (1) That chlorine is an element? (*2) That the atomic weight of chlorine is 3").;") ? (3) That the molecular weight of chlorine is 71 ? What relations have been been found to exist between the volumes and weights of gases ? Calculate the weight of one cubic metre of each of the following gases at U^.C under a pressure of one metre of mercury : Oxygen, Hydrogen Iodide, Hydrogen Sulphide. S=32; 1 = 127; O = Ki. 4. What explanation is given of the existence of the allotropic modi lication of oxygen known as ozone. 1. University College, Toronto. I. Give equations representing the frllowing reactions : (a) Hydrogen on Ferric Oxide. (/') Steam ou Iron. (c) Chlorine ou a cold and ((/) a hot solution of Potash. {e) Nitric Acid on Tin. 2. Describe the preparation of hydrogen sulphid'?. What '-'olume of oxygen is re(juired for the combustion of 1 litre of this gas, what will ))e'the volume o*' tlie products formed 'i 3. How could carbon monoxide be shewn to contain half its volume of oxygen ? 4. CMorinc i.s s 'dtobe monovalent, oxygen divalent, carbon tetra- valcpt, I'^xplain what is meant by thi'se statements and give a method for tlve determination of the valency of one of these elements. "). Descr'bo the al)otropi(r forms of sulphur. What proofs hnve we th;i.t tin -■.<• con? >it of nothing Imt sulphur "/ I ■11 ' UNIVERSITY COLLEGE, TORONTO. 253 6. Give a inethod for tlio preparation of liydrogni chloride, and tix- plaiu fully tlu; data on wliioh the formula IIOI is assigned to it. II. 1. Fownes, 12th ed., p. 252, states (hat " equal volumes of all gases contain an ecjnal nnndx-r of molecules ;" and at ]>. 12(5, "the combina- tions (of the elements) are rep-esented symbolically by the juxtaposition f»f the elenienfeiry atoms .... thus the molecule of Hydrogen Chloride composed of one atom of hydrogen and one atom of chlorine is repre- sented by HCl, the molecule of water by H.^C) . . 8I{('1 denotes ',i molecules of hydrogen chloride, 3H^S0^ 3 molecules of sul[>]iuric acid." In his account of the Chemistry of (Jhlorine, he represents chlorine in various equations by the following symbols : CI, (U^, Cl^, ("L,. State fully what facts are conveyed by these symbols ; which of them do you consider correct, and for what reasons ? For Honors. Show that the s'^^atement that a certain gas contains - ne-fourth its volume of chlorine is im[M)ssible if the present atomic W(ught of chlorine is correct. 2. Show how the formula in use represent the per-centage composi- tion of bodies, and from the following per-centage com];>ostion calculate a formula : s H 5G-637. 42-478. •885. lOO^ 3. Matter is said to be composed of the elements. Give several illus- strations of tlie facts wliich lead to this theory, and explain why you consider a mixture of ecjual volumes of hydrogen and clilorine gases to be a "mixture " before explosion, and to be replaced by a "comp(Hind" after explosion. 4. Calculate the volume oi I '"• — 453-68'-""'- of sulphur vapor at SOOTJ , and at lOOOX', and 7(30°""" Bar. III. lUlphur, 1. Illustrate the laws of cond)ination with comp<>un'""' T>ar. Avill be formed by the deeompositiou of 400 grms. of water 't What would be the volume if tlie molecule of hydrogen contained 3 atoms ? 4. How may oxygen be prepared ? By what })roperties is it charac- terized ? The oxides of the non-metallic elements have been described as reacting with water to form acids. Illustrate this with the oxides of chlorine, sulphur, and nitrogen. 5. State fully the facts indicated by the fonnula S.^ CI,,. Why should it not be S CI ? IV. 1. " the chemist finds himself o])liged to divide substances into two great classes, (1) Compound Substances— -those wliich he is able to split lip into two or more essentially different materials, and (2) Ele- ments or Simple Sul)stances, those which he is uaable thus to split ixp. " — Roscoe's Elementary Chemistrij. Explain fully what is meant by the expression "split up." 2. A furnace uses 1 kilogram of coal an hour. Wliat vohime (in litres) of air at normal pressure and temper ture must pass tlirough the furnace, assuming that oidy half of the oxygen is burnt, and that coa^ ia pure carbon? 3. Describe experiments showing that carbon monoxide contains half its own volume of oxygen. 4. In what forms does silicon chieriy occur in nature ? 1 low is the element obtained in a free state V Describe the characteristic properties of its oxide and fluoride. 5. How do you prove that air i,s a mixture ? 6. How is the element Sodium pre})arod ? Give a short account of the most important compounds, for what reason do we say that it is monovalent 'I 7. CJive a brief account of the Chemistry of Copper. Calcu'ate the percentage composition of cuprous chloride Cu==()3r) 01 = 35-5. V. 1. What a e the facts which convince chemists that matter is com- posed of a limited nundjer of elements '/ 2. The law of reci[)rocal proportion may be stated f bus : if each of two ck'inenta combine with a third, then those (juantities of . the first two elements which combine witli a fixed weight of the thii'd are either those in which the first two elements combine or bear some simple ratio to them. Show that this is true of the elements mercury, chlorine anoriments to show the nature and properties of Sulphur. How much air is neetled to burn ciunpletjly 4 oz. of Sulphur? 5. Fully describe and explain the following experiment : (a) 8(nne strong Siilphnric Acid is ])oured on a piece of zinc, and after the chemical action has ceased, water is earefutly added. {!)) Carbon Dioxide isp;i^sed for sometime through lime water. A portion of the clear solution thus obtained is boiled ; an.»ther portion of it is exposed for an luuir or so to the air ; and, to another portion, lime water is added. (c) Some distilleil water is shaken up in each of the full receivers mentioned in 3 above. (d) Some Chlorine gan is exposed to the air in an open receiver, {e\ One volume of Hydrogen is mixed with one volume and a half of Chlorine, and the mixture exposed to the action of dittused sunlight. G. You are given a j>nwder known to be Carbonate of Ammonia, Phosphate of Soda, Kiti'atc of Tieatl, or < hlorate of I'otash, Describe the simplest mode of determining which it is. m 256 PRACTICAL CHKMISTRY. First Class Teachers. — Grade 0, 1885. Examiner — .John Seatu, B.A. 1. Sttate the principles that govern the relation of gases to pie.ssure and to temperature. One volume of Hydrogen is confined in a fi.^sk at 10°C under the ordinaiy pressure of the atmosphere, athU'd to tliat of a colunni of mer- cury GO //mi hi gli. The flask is to he heated to H()0°C without any in- crease taking })lace in the volume of the gas. How high must the colunm of Mercury then stand, supposing the atmospheric pressure to increase to 900/// m. ? 2. ()..33r)5 Oi' an organic compound, containing only Carbon, Hydro- gen and Oxygen, gave on combustion O.G715 gramme of COo and 0.2745 graniTue of H .,( ), and its vapor density was found to be forty-four times that r.il Hydrogen. Find its empiiical and its molecular formula ; and express tlie latter in tlie graphic notation. 3. Make a list of (a) the impurities of city well-water, (h) the sources of sucliimi)urities, (c) the tests by vvhieli you would detect them, and ((/) the means yon Avould use to purify a given sample of impure water. 4. A powder is given yoii known, to be Carbonate of Soda, Iodiav^ of Potash, Bromide of Potash, Fluor S[)ar or Sulphate of Ijime. Descrd^e a fi'utqile mode of determining which it is. 5. 20 granmies of an aqueous solution of HCl were mixed with an excess of Argentic Nitrate. The precipitate, wlien co lected, washed ami dried, weighed 4.,'j3 grammes. Calculate tlie percentage of HCl in the original solution (Ag= 108). 6. Fully describe and explain the following experiments : — ■ {a) Some white Arsenic is boiled with diluted Kitric Acid, and the gas given off is passed into water. To :i portion of tliis solution is added a solution of Permanganate of Potash, and to an(jther a solution of Iodide of Potash and Starch. (b) A test-tube containing an aqueous solution of Chlorine is ex- posed to the strong rays of the snu. (r) Oxygen which has l)een allowed to bubble through strong Sul- phurous Acid is passed through a tube in which is heated some plati- num si)onge. (//) Some Manganic Dioxide is boiled with an excess of strong HCl. The gas evolved is led into a strong aipieous solution of Potassic Iodide. ((') Some Nitric oxide is mixed Avith an excess of Hj^drogeu and passed over moderately heated platinum sponge. {/) A test-tube containing a ])iec'3 of Phosphorus in an aqueous solution of fi'esk slaked lime is boiled for some time. 7. Explain fully what is meant by the statement that Silicon is an exception to Dulong and Potit's law. SECOND CLASS TEACHERS, 188G. 257 Second Class Teachers, 1886. Examiner — John Seatii, B. A. 1. How wonll you (lciuonstrat(! with KCIO,, tlie differcnco })etween physical change and chemical chaiigo ? 2. With some water containing C'O^ in solution, is shaken up a mix- ture of pure sand and NaCl. (1) How would you separate these four substances ? (2) How would you prove that y )U had separated them? 3. An organic Ixxly which is knoM n to contain on'y (', O, and H, gives an analysis 27-r)8% of O and l(i-oo% of H. Its vai)or density is 58, that of H heing unity. What is its molecular formula? 4. You are given HCl and iVHa (each in the form of a gas), litmus paper, and turmeric paper, and [)ure distilled water. How would you demonstrate the nature and properties of an acid, an alkali, and a salt? 6. A liquid is known to contain HoSO^, HI, HCl, KHO, or NH.jOH- Give a simple mode of determining which it is. 6. How would you demonstrate (1) TliL! reseml)lanoes and differences between H and (.'1? (2) Tilt! effects of heat upon a mixture of 4 vols, of H, one vol. of O, one of 01, and one of N ? 7. The water of a well is supposed to be contaminated by sewage. Describe the means you wouM take to determine the question ? 8. (1) A glass rod moistened with strong H.^^O^ is held very near a mixture of jjowdered KCIO3 and dry loaf sugar, hut so a.s not to touch it. (2) A glass rod moistened with strong H^SO^ is broutjLt into con- tact ivith the same mixture. Describe and explain what happens in each case, and state the general couolusiou you would li.ise on these and similar experiments. IS ex- Second Class. NORMAL SCHOOLS, JUNE, 1887. Examiner — John Seatu, B.A. 1 . What facts would lead you to the conclusion that matter is com- posed of elements ? Explain why, when equal volumes of 71 and CI have been passed into a vessel, you consider the contents of the vessel to be a '•mixture" before explosion and a " compound " after explosion. 2. Describe v^he allotropic forms of Carl ion and Sulphur, and the ex- periments by which you would prove that they are allotrojuc forms. 3. Into separate test-tubes containing dilute HCl are put the follow- ing : — zinc, zinc oxide, marble, common salt, charcoal, gold. Explain, by means of equations, the chemical changeo that take place. 18 pi 258 PRACTICAL CIIKMISTRY. 4. If 112 litres of H weigh ten grammes, wliat is the weiglit in grammes of the same volume of (.'1, and of the same volume of HCl,the, temperature and the pressure being the same ? Explain, in each case, the process by which you reach your con- clusion. First Class Teachers.— Grade C, 1887. .u If! I> (J. A. JMCl-iKLLAN, LL.L), Ext 1. How would you (1) prove and (2) explain Graham's Law of diffu- sion ? What volume of COg will diffuse tlirough a stacco plate in the same time as 2-461 grms of N^O, both being at the standard pressure and temperature ? 2. One gram of a certain metal when dissolved in dilute sulphuric acid liberates 200 c.e. of hydrogen gas. Find the combining weight of the metal. 3. By what experiments would you distinguish marsh gas and olelian fc gas? 5 c.c. of a mixture of marsh gas and olefiaat gas are exploded with 14 c.c. of oxygen : 1) c.c. of gis remain, of which 7 c.c. are absorbed ])y caustic potash. Find the volume of each of the gases in the original mixture. 4. (I) How would you prove that the gas obtained by pouring sul- phuric acid upon ferrous sulphide contains both S and H ? (2) A solid substance contains both a carbonate and an easily dissolved sulphide. How would yo^i prove the presence of these two bodies ? (3) A piece of sodium was completely converted into chloride by uniting with 200 c.c. of CI at the standard temperature and pressure. What was the weight of the sodium? 5. Name the chief oxidizing agents with which you have experimented, and explain the theory of the action of each. 6 A small cage containing a live rabbit is placed in one pan of a delicate balance, and the instrument is then exactly counterpoised. . H the whole be allowed to remain at rest for some time, a distinct diminu- tion is seen of the v/^eight of the cage and its contents. Explain this fact. 7. Name the elements in your course of study which form volatile compounds with hydrogen, arranging tliese elements according to their atomicities and giving the graphic formuhe of the hydrides. 8. Ynn are gi\'en an unknown salt a comm(m acid, and are required to identify the acid radicle. On analysis, the salt is found to be a phosphate, (iive a full account of your analysis. SECOND CLASS TKACHEKS, 1887. 259 eight IP iCl.the, )ur con- of aiffu- te in the pressure uric acid ht of tlie 1 olefian fc )clecl with orbe("i l)y original inng sul- m easily lese two oride by )ressure. iniented, pan of a ised. . If diminu- lain this volatile I to their required I to be a Second Class Teachers, 1887. jiT„„,„ ■„,,.„. f.JOHN Skatii. B.A. Examineis. | j_ ^^ McLellan. LL.B. 1. The gas contained in a transparent receiver is kixwri to consist of N and O. How wouhl you determine whether it is a chemical com- pound or a mechanical mixture ? 2. A glass is given you, containing muddy water impregnated with ammonia, {a) How would you render the water pure ? (b) How would you prove that you had done so ? 3. Some chlorate '' potash and black oxide of manganese are heated in a test-tube till tlic evolution of gas ceases. (a) How would you find the weight of the chloride ? {b) If the chloride weighs 10 grammes, what was the weight of the chlorate ? (c) How would you prove that the chloride is a different substance from the chlorate ? 4. Find the quantities of lime and sal-ammoniac necessary to i)repare 11.2htresof NHa. 5. Two receivers of the same size and shape, containing res])ectively CI and H, are placed, for some time, mouth to mouth in difl'iised sun- light, the gaaes being separated by a thin plate of plaster of Paris. (a) Describe minutely ami explain what takes place in each receiver. (/>) What conclusions would you base on this experiment ? 6. Into a receiver containing perfectly dry chlorine, is introduced some litmus paper. Describe and explain what takes place (1) when the litmus paper is perfectly dry, and (2) when it has been moisteneil with water. 7. You are given an o])aque receiver covered with a ground glass plate and known to contain NoO, NgOa, HI, or CI. How would you determine most .simply which it contains ? 8. 3 litres of H, 2 of N, and 4 of O are measured at ()°C and 760'"»» mercurial pressure, ami an electric spark is passed through the mixture. What is the volume of the gases after combustion, the measure- Hicnt ])eing made at U'^C and TOO""" mercurial pressure ? 1). 100 giammcs of nitre are distilled Avith sulphuric acid. "What weight of i:.muu)nia will be needed to neutralize the tlistillate ? 10. What volumes of the constituent gases would be obtained by de- composing three volumes of e;ich of the following gases : — nitric oxide, ammonia, water vapor and hydrochloi-ic acid? From Williainsou's Chemistry : — I. 1. A litre of oxygen is required of tiie density of 100 at 0°('. Wliat weight of potaK.sic chlorate nuist be used for its preparation and what total pressure must be applied to it t tj^H ^ r s I^B I \ 1 iil' ' 1 200 PHACTIl'AIi CUKMl.STKV. 2. 100 cubic metres of liydrogt'u are supplied .at I2''0. Wanted the weight of a balloon which when lilled with the liydrogun would press upwards with a weight of 20 grainiiies. 3. What volume of air is required for the oxidation of tliat ([uantity of metallic eopi)er which is reduced from its oxide by 10 grammes of liydrogen '! 4. What wtiiijlit of potassic chlorate is recjuired for the evolution of that ([iiantity of oxygen which is needed for the combustion of 10 litres of hydrogen ? 5. An experimeatalist rills with dry and pure atmospheric air at the temperature of 10\J, a bottle of 1 litre capacity at a height in the atmosphere such that a barometer stands at a height of 350 millimetres. What volume of nitrogen will he have at tlu noruial temperature and pressure ? II. 1. What weignt or nitrous oxide is obtained from a milligrammeo f ammonio nitrate ? And what is the volume of the gas at the normal temperature and p/essure ? 2. What will be the dimensions of a flv'ik capable of containing 20 grammes of ammonia gas at 12''G and 7M mtn. pressure ? 3. 3 grammes of nitric oxide are mixed with an excess of hydrogen and passed over moderately heated platinum sponge. What weight of ammonia is formed ? 4. What veiglit of air is nee led for the complete combustion of 1 miL''gramme of carbon V 5. What weight of carbcm is contained in a litre of cai'bonic acid gas? III. 1. What volume of carbonic acid muse be passed over white-hot charcoal for the preparation of 10 litres of carbonic oxide ? 2. What weight of ammonia would be neutralized by a kilogramme of liydric oxalate (H2C2O4) ? 3. What volume of air is needed for the complete combustion of a litre of marsh gas? What will be the volume (calculated at 0°C and 760 mm ) of the steam formed by its combustion ? 4. What volume of hydrojzen would be obtained by passing a litrti of hydrochloric acid gas over hot metallic iron ? 5. W^hat weight of potassic hydrate is njeded for tlie neutralization of 100 grammes of hydrochloric acid gas ? IV. 1. 20 grammes of an aqueous solution of hydrochloric acid were mixed with an excess of argentic nitrate. The precipitate (argentic chloride) was collected, washed, dried. ;.nd weighed. It amounted to 453 grammes. (Calculate the percentage of hydrochloric acid in the original solution. Wanted the A'oultl press lat (luantity granunes of evolution of I of 10 litres ic air at the sight ill tlie millimetres. )erature and ligrammeo f the normal ntaining 20 r)f hydrogen at weight of )ustion of 1 trbonic acid r white-hot vilograinme )iistion of a at (J°0 and ng a litrd of utralization acid were e (argentic loiinted to acid in the EXAMINATION QUKSTIONS. 2r>i 2, Wliat vohime of cldorine can ho made to condiim; dinctly with ^ litre of oletiant gas ? What is the vai)(»ur-V(»hime of the product V 3. What ia the weight of a cubic centimetre of bromine vapor at 15()°(! and 700 min. 'i 4 .What volume of liydriodie and gas could be prepared from 10 grams of iodine ? 5. What volume of oxygen is re(iuired for the combustion of a cubic centimetre of sulijhuretted hyreparod by Mr. Lochlieati : — 1. Give the distinction between metallic ami non-matellic elements What reason are there for disregardfng the terms noii-mttal aiul metal as applied to the elements ? 2. A substance on analysis gave the following percentage composition: Mg = 9-76, vS - 1301. U — 2601 H^O 51-22. Calculate its formula. 3 Explain the chemical action which takes place in the formation of chalybeate spas. 202 FUAflTICAL CIIKMISTHY. 4. Now may the atoiiiic wcjights of olument.H l)o (leturmiuctl 'I 5. Kxplain the formation of stalarfiii's, si/icijud wootl, and c.nal. 6. Required the weight of limestone needed to convert 50 tons of soila cryatala into hicarbonatc. 7. I()l)()308 grains of mercury were obtained from IIS'MD.'W grams of the red oxide. What is the atomic weight (»f mercury 't 8. Explain Mendelejeff 'a classification of the elements. What modi- lications has liothar Aleyer made in his representatitdi of the I'eriodic Law ? 9. What are: — Paris Green, Sfhfiele\'i Green, Turnhuirn Blue., White. Lead, Blue, Vitriol, VerdhjrU and fer rd duit ? Explain their composi- tion. 10. What inprovements did Simens and Dafur make on Hessemer's process ? 11. Describe the chemical action which takes place when mortar nets and lime flakes, also when Pnrin planter seta. 12. Explain the preparation of the super phofiphate of lime, fertilize! from apatde. 13. Describe the chemical jjrocess of producing a photographic print, 14. What is the composition of kaolin, lapis lazuli, emery, elaij, and rubij ? 15. Explain the process of solderintf. What is the use of borax in the operation ? 10. Write a short article on the occurrence of potassium in nature, its uses and the part it takes in the vegetable and mineral kingdoms. 17. When gunpowder explodes, represent the chemical change by au equation. 18. What relation exist between the atomic weights of the aikaK. metals ? 19. Explain a lime-kiln. What change takes place when lime is lef-' exposed to the air ? 20. Why are the zinc plates in galvanic batteries amalgamated ? W^hat objection would there l)c to liaving a little corrosive sublimate mixed with the calomel which is used in medicine ? 21. Name the chief zinc ores. How is the metal extracted from the ores ? 22. How can potassium and sodium be detected when both are present ? 23. GivcTi that calomel vapor has the specific gravity 117'(J. Is it proper to write its formula HgoCl.^ ? What reasons have we in sup- posing that the sp, gr. is much higher ? 24. How would you distinguish between mei-curous and mercuric, ferrous and ferric, stannous and stannic compounds ? 25. How would you prepare mortar as used by masons ? "What use , is made of lime in agriculture ? EXAMINATION QUESTIONH. 263 26. Explain the composition of tho following cemonta : hifilrauUc, Jiunian, Portland. 'll. What Huhstanco is eni])loyo(l ;ih an adulterant of white lend? 28. What in diuli/.^i-i i How is tlui niuthod employed in the separation of arsenic from organic mixtures ? 2!). How would you sliew tliat "the heat capacity of an element is inversely as its etpiivalent ? " Distinguish l)etwoen the <'7»(/ra/rHniml the atomic weight of an element. 30. Enunciate Dulong and Petit'a Law, and state what use it is in the accurate deterininati(m of atomic weights ? 31. What is meant by cfccfnhfxmfirf and cJcctrn-neiiat'n'e elements ? Does this divir^ion corre^<^K>nd to the noii-nictairn' and iintnUh'. division? 32. Distinvjuiah between empirical, rational and (jraphic formuhe. 33. What experimental evidence is there for the existence of metal ammonium ? 34. ICxi»lain the com])naition of the different i//ass('s. What use does silica serve in the l)uilding u[> of plants ? 35. What is the composition <»f talc, vnersrhanm, xcrpnitine and asbi'MoH ? What uses are made of these minerals in the arts ? 3(). Distinguish between Eji.'°(J and 7<>l) mm. ? 39. Caleulo.t;'. the formula of soda f.dspar from the following analysis : SiOa - ()8-4."); A\J\ - lS-71 ; Fc.O., - 0-27; CaO - Or)0;MgO - 0-18;Kj) -0G5; NaaO - 11-24 = lOOOO. 40. What is a mordant ? Name some compounds which are mordants. Tho following sots of (jiiestioiis liavo been selected from those set !it tiie exaiuiiiatioiis of the Science and Arts Department, South Kensington, England : — I. 1 . I add two vohnncs of oxygen to one volume of each of the follow- ing gases : What takes })lace and what effects will be produeeil if an electric S])ark be afterwards passed througli each of the mixtures? Chlorine, hydrogeii, sulphuretted hydrogen, nitrous oxide, nitric oxide, carbonic oxide, carbonic anhydritle. 2. What takes place when carbonic anhydride is passed into — 1st, distilled water ; 2nd, baryta water ; and 3rd, water containing some freshly precipitated calcic carbonate (carbonate of lime) ? 3. Stf^''''s exactly how you Avould separate from each other, and indi- v' iuo-Ujf letect, the following constituents of a solid substance given to IMAGE EVALUATION TEST TARGET (MT-3) / O fc 1.0 I.I 1.25 *- IIM IIIM -' illM i||2l m 2.0 mm 1.4 il.6 d /W'' ^ //, VI ^^ <5i. '/a / m Photographic Sciences Corporation 23 WEST MAIM STREET WEBSTER, NY. 14580 (716) 872-4503 # iV iV \\ 4^, ^>. "9) ^ ^^ c^. c^- ^ r 264 PRACTICAL CHEMISTRY. you for analysis : — Peroxide of mercury, rioda, protoxide of iron, oxide of copper, magnesia, sulphuric acid, and hydrochloric acid. 4. How much hydrogen by weight and by volume (in litres) is re- quired to reduce 25 grammes of ferric oxide (FeaOg) to metallic iron ? 5. You have given to you distilled water, oil of vitriol, nitric acid, copper turnings, iron filings, and metallic lead. State what salts you could prepare from these materials, and describe briefly how you would make them. Give their graphic formula? and explain the chem- ical changes by equations. II. 1. You have given to you some sulphur, water, and nitric acid. De- scribe how you would make sulphuric acid from these materials. 2. Y ou are required to make oxygen from chlorine and water. De- scribe exactly how you will do it, and give a sketch of the apparatus which you ])urpc se to employ. 3. I pais sulphuretted hy: II. 1. DescriUo the chaugea which are produced in lead, zinc, and tin by heating them, and describe the experiments which taugtit you what these changes are. 2. How did you learn that the air had anything to do with these changes ? 3. Did heat have anything to do with the changes ? Suppose you knew that the bits of lead, zinc and tin increased in weight when heated in the air, and that they did not increase in weight when heated BO that the air could not get at them, what would that show ? 4. What familiar facts show that the air has something to do with burning ? HI. 1. How can we get oxygen from the air ? What happens to oxygen when it is much cooled down and compressed ? 2. How does oxygen behave towards other substances at ordi- nary temperature ? How do you know this ? Does oxygen act upon anything at the ordinary temperature ? Give examples. 3. What difference is there between the action, of oxygen at the ordinary temperature and at higher temperatures ? How did you learn this difference ? 4. la burning in the air the same chemical act as burning in oxygen ? How can this be proved? Why do substances nob burn as actively iu the air as they do in oxygen ? 5. What is meant by the kindling temperature ? Explain why it is thii.b a fetiok of Wood burns gradually and not all at once. 6. Explain the connection between the heat and light produced, and the combastiou of a substance. 7. What is meant by the expressions chemical work and chemical energy ? IV. 1 . How are natural laws discovered ? 2. Wliat is a natural law ? 3. What are tho combining weights of the elements ? V. 1. How cax nitrogen bo obtained from the air oy the use of copper I How does it differ from the oxygen ia its conduct towards burunij^ things? Could animals live in it? Why? Suppose faoie were no nitrogen m the air, how would on/ fires differ from the fiics in the air t VI. 1. How can it be shown that water is containea m woo.l ? iu meat ? 2. Is water present in large or small proportion in animal and veg*^- table substances ? 3 What is meant by water of crystallization ? EXAMINATION QUESTIONS. 273 4. What are efflorescent substances ? 5. What are delicjuesoent substances ? VII. 1. Which are the common acids ? What do they all contain ? What takes place when they are treated with metallic elements ? 2. What relation is there between the combining weights of hydrogen and oxygen and the weights of eijual bulks of the two gases ? 3. When we say that the combining weight of hydrogen is 1 and that of oxygen 16, what is meant? What is meant 'when wo say the combining weight of iron is 56 ? 4. If we should call the combining weight of oxygen 100, what would be the combining weight of hydrogen ? VIII. 1. Explain exactly how the experiment with copper ojude teaches us what the composition of water is. 2. What is the character of the water found in mountain streams which flow over sand-stone ? Why ? 3. What is the character of water which flows over lime-stone ? 4. How does water become salt ? 5. What are effervescent waters ? What is sulphur water ? 6. What change takes place in river-water which has been contam- inated with drainage ? 7. Does combination take place more readily between tuose elements which are alike or between those which are unlike ? 8. What change takes place in oxygen when electric sparks are passed through it ? IX. 1 . From what kind of substances is ammonia given off in destructive distillation ? 2. What is one of the products formed when substances containing carbon, hydrogen, and oxygen are heated ? In what experiment which you have already performed is this shown ? 3. Explain why ammonia is formed in gas-works ? 4. What does the process of decay consist in ? 5. What becomes of the nitrogen contained in animal substances when they decay ? 6. What connection is there between saltpetre and nitric acid ' be- tween potassium nitrite and nitrous acid ? 7. How is nitrio acid formed in nature ? 8. What is the appearance of pure nitric acid? What takes place when It IS boded ? when the sun shines directly upon it ? flow does strong nitric acid act ? 9. Why do substances burn in strong nitrio acid? Describo some JLv 274 PRACTICAL CHEMISTRY. ' ! .1 ^i experimentfl which illustrate the jmwer of nitric acid. What does the nitric acid give up to the substances upon which it acts ? 10. What does nitric acid give up when a metallic element acts upon it ? Describe what further takes place, and wliy ? X. 1. What is meant by saying that oxygen belongs to a family of elements ? 2. What two stops are necessary in order to get chlorine out of sodium chloride or common salt? Wliat resemblance is there between the process for making hydrochloric acid from common salt and. that of making nitric acid from sodium nitrate ? What is formed besides hydrochlori(3 acid and nitric acid in each case ? 3. What is the action of chlorine upon water ? 4. What is disinfection? What is "bleaching powder"? What other name has it ? Why is it valuable as a disinfecting agent ? 5. Compare the action of hydrogen on oxygen and on chlorine. What are th(3 products ? What are chlorides ? 6. What diflference is there between the action of a mixture of chlor- ine and hydrogen and a mixture of hydrogen and oxygen ? 7. What happens to hydrochloric acid when it is treated with a metallic elent^nt like zinc ? when treated with an oxide like zinc oxide ? When treated with substances which give up oxygen readily ? XI. 1. How can you determine what is formed when an acid acts upon a base ? What is formed when hydrochloric acid acts upon caustic soda? nitric acid upon caustic soda ? sulphuric acid upon caustic soda ? hydro- chloric acid upon caustic potash ? nitric acid upon caustic potash ? sulphuric acid upon caustic potash ? XII, 1. Wliat takes place when animal and vegetable substances ai'e heated to a high temperature ? Why is this ? What takes place when they are heated in the air ? 2. What is coke, and liow is it obtained? lampblack ? bone-black or animal charcoal ? 3. What are charcoal filters used for ? What are bone-black filters used for ? 4. What is the object of charring piles which are exposed to the action of air and water ? 5. What diflFerent kinds of coal do we distinguish between ? What is lignite ? peat ? 6. How can we form an idea in regard to the reason why one and the same thing can appear in diflPerent forms ? 7. Explain what takes place when a mixture of charcoal and copper oxide is heated ; when a mixture of arsenic and charcoal is heated, la EXAMINATION QUESTIONS. 275 there any resernblauce between the action of hydrogen and of charctwd on heated copjier oxide ? v-ncivvm 8. What iuiportant use m made of charcoal as a reducing agent ? XIII. tor; v^^under' wJ^lV'"'- '^"""J ^'^ ^heBc easily formed in the labora- formed? circumstances are they easily and abundantly w?' y^^u """u*^*" ^'"^^ products of the oxidation of vegetable matter' What 18 the chief product of the reduction of vegetable matter ? ' 3. Of what importance ia the occurrence of marsh-gas in coal-niines ' foita^n trcrbTdSe^ "'"'^"^^^ ''^^"'« ^^^'^^^ ^'^ "«« *« *^-' 5, Why does not carbon dioxide burn ? 6 What is meant by the statement, "Carbon can do chemical work ? " bodf of'watfv"'^ '" ^*'^'^'' * P^"'^ "^ °^"^"^ ^d *" ^le^ate^l 7. What is choke-damp, and what does the name come from ' fuei bItdil'sTotT " *'"' '^*"^^" *'^ '^^^ «^ ^'^-^l^ -1 *he ag^in?'"'^ '^''''' *^° ''^'*^''" ''^ *"'™*^^ """'^ P^^"*^'' S^* ^^ck into the air 10. In wliat way is all life directly dependent upon the sun ? 11. How are carbonates formed ? What is the composition of sodium carbonate? of potassium carbonate? What, then, is the comDositZ 12. Wliat takes place when carbon dioxide acts upon potassium Iiv droxide V upon calcium hydroxide ? Give the equatioAs represTtZ th^ action, and name the products. representing the 13. Why is the use of water-gas sometimes objected to? 14. Why is carbon monoxide a good reducing agent ? 15. Of what importance ia it in the reduction of iron from its ores ' a limpT^'*^ '' ^ ^*"" • ^^"* '' *^" ^^''^''^^ ^«*^««n a candle and is extiS'uTsh" ?' """ "''""''' *^'* "^"^ " ^"^"^"S g- i« -«l-l ^o^ it XIV. 1. Do we know why substances combine according to the law of defi- nite and multiple proportions ? ^ ®" 2. What is an hypothesis ? a theory ? anLTurAf;;:^rtos?'' ^^^-'^^ "' -P"-- the laws of definite 4. Show how this theory accounts for the facts ? i'l 1; In ' ,: i [ Jtil ?!■■■ li 276 PRACTICAL CHEMISTRY. 5. How does Avogaclro's law help us to determine the relative weights of the molecules of gaseous substances ? 6. How are the atomic weights determined from the molecular weights ? 7. What diflPerence is there bi t ween chlorine, oxygen, nitrogen and carbon as shown by the symbols of their compounds wita hydrogen ? What is meant by the valence of an element ? 8. What is meant by a univalent element ? a bivalent element ? a tri- valent element? a quadrivalent element? Barium forms the compound BaCly ; what is the valence of barium ? Sodium forms the compound NaCl ; what is the valence of sodium ? 9. In the formation of potassium nitrate from nitric acid, how is the valence of potassium shown ? When a bivalent element like calcium forms a salt with nitric acid, how does the displacement of hydiogen take place ? Calcium is bivalent. What is the sj-mbol of its salt with sulphuric acid ? Explain this. What is the symbol of the sodium salt of sulphuric acid ? What does this show with regard to the valence of sodium ? 11. If magnesium, Mg, is bivalent, what is the symbol of its sulphate ? of its nitrate ? of its chloride ? What is the basis for the distinction between r,cid-forming and base-forming elements? Give examples of the two classes. What are these classes sometimes called ? 12. What is meant when we speai: '^f a family of elements ? 13. What are the families of acid-forming elements ? XV. 1. How is bromine obtained from sodium bromide? Give the equa- tions representing the steps which 7uust be taken, and explain what is meant by them. 2. What is hydrobroraic acid, and how is it formed V What differ- ence is there in the conduct of common salt and of sodium bromide towards sulphuric acid ? How is this explained ? 3. How is iodine obtained from sodium iodide ? Give the equations representing the action, and explain what they mean. 4. What are the properties of iodine ? Compare chlorine, bromine, and iodine, stating the points of resemblance and difference. 5. How can you easily tell whether a substance is an iodide or not ? 6. What takes place when potassium iodide is treated with sulphuric acid. 7. Whax, analogy is there between chlorine, b I'omine, and iodine, as far as the compounds which they form are concerned ? 8. What relation is there between the atomic weights of chlorine, bromine and iodine ? XVI. 1. Why has sulphur been known for a long time ? Where is it found in nature? What is the chief source of the sulphur of commerce? EXAMINATION QUESTIONS. 277 Name some of the principal compounds in which sulphur occurs in nature ? 2. Describe the process by which sulphur is extracted from its ores. 3. Can sulphur and hydrogen be made to combine directly ? Wliat is formed ? Where is tliis compound found in natare ? Under what conditions is it formed ? 4. How is hydrogen sulphide made in the laboratory ? Explain what takes place when sulphuric acid is used ; when hydrochloric acid is used. How is the substance collected ? How does it behave towards water? towards metals? What takes place when it is passed over heated iron ? Is there any resemblance between this action and that which takes place when steam is passed over heated iron V Express both acts by chemical equations. 5. How can hydrogen sulphide be used for the purpose of learning what things are made of. What is formed when sulphur dioxide takes up more oxygen ? What is the product of the action of sulphur trioxide on water ? What relation is there between sulphurous acid and sul- phuric acid ? 6. Where is sulphur dioxide found in nature ? How is it made in the laboratory ? Explain the reactions, giving the equations. 7. How are sulphites made? What is the composition of sodium sulphite ? What takes place when sodium sulphite is treated with sul- phuric acid? with hydrochloric acid? Compare these reactions with those which take place when sodium carbonate, Na^COg, is treated with sulphuric acid and with hydrochloric acid. 8. How does sulphuric acid act towards water ? What change does it produce in wood? Explain the change. How does it act upon organic substances which contain hydrogen and oxygen ? 9. What is a monobasic acid ? a dibasic acid ? 10. Define acid, normal and neutral salts. XVII. 1. Explain what takes place when phosphorus and iodine are brought in contact with each other. 2. Mention some of the varieties of silicic acid, and point out the relation which exists between them. 3. What important manufactured product contains silicon ? XVIII. 1. What is meant by the name base-forming elements ? What name is given to the elements which are not base-forming elements ? How does the number of base-forming elements compare with the number of acid-forming elements ? 2. What is meant by metallic properties ? 3. What are the chief classes of metal derivatives ? 4. What ar-e mineraL ? 5. What is meant by the term metallurgy ? 278 PllACTICAL CHKMISTRY. XIX. 1. Give an account of the manufacture of gunpowder, and explain its use as an explosive. 2. What is sodium amalgam, and for what is it used ? 3. What is water-glass, and how is it made ? What is it used for ? XX. 1. Mention some of the chief compounds of calcium found in nature. 2. Explain a lime-kiln. Explain the use of lime in making the lime- light. What change takes place when lime is left exposed to the air? What change takes place in it when it is treated with water ? 3. What is bleaching- powder, and how is it made ? Of what value is bleaching-powder ? Under what conditions does it give up its chlorine ? 4. Mention some of the principal varieties of calcium carbonate which are found in nature. What are stalagmites and stalactites ? 5. Explain the diflference between permanent and temporary hard- ness of water. 6. Of what importance is calcium phosphate to plants ? What objec- tion is there to the use of normal calcium phosphate as a fertilizer ? What is superphosphate of lime, how is it made, and what is it used for? 7. How is mortar made ? Why do freshly-plastered rooms remain moist so long? How can the process of drying be hastened ? 8. What is common glass ? What is the diflference between sodium glass and potassium glass ? What is flint-glass, and for what is it used ? Extract from the Curriculum of the University of Toronto Junior Matriculation : CHEMISTRY. Elementary Inorganic Chemistry. — This examination will be limited to the chemistry of the elements — hydrogen, chlorine, bromine, iodine, fluorine, oxygen, sulphur, nitrogen, phosphorus, arsenic, carbon, boron, silicon, and their characteristic compounds, including the laws of combination of the elements, and the meaning and use of the theory of the " Molecular and Atomic Structure of Matter." ADDENDA. The following was received too late for insertion in its proper place : UNIVERSITY OP TORONTO. At the Junior Matriculation Examination, Pass and Honor papers will be set in Chemistry and Biology ; and the following prescription of work replaces that on page 11 of the Arts Curriculum, dated 1885 (this enactment comes into force at the Examinations of 1888) : I.— ELEMENTARY CHEMISTRY. fnPtlnin*'*'''" "f ''^'\^*' *?^.*^^ '^^^''^^' ^eJations of the physical sciences to Biology, and of chemistry to physics. °^i^u^va Chemical change : elementary composition of matter. Av^gldro'l ^^Z^'""^^''"' ""^ *^' ^^^"''''*' '■ ^*''^^" *^"°"y' "molecules, The determination of atomic weight, specific heat, atomic heat— nomenclature, classification. o , f , a,Lumn. neai nf Til! P'ifP^':^*^^"' characteristic properties, and principal compounds of the fo lown,g elements : hydrogen, chlorine, bromine, iodine, oxy- gen, sulphur, nitrogen, phosphorus, carbon, sihcon. ' ^ rr^Ult ^S''''' ^^a^n^ti'^n will include the chemistry of all the ele- mmits suthcient to illustrate tlie classification known as Mendele%ff^ The following is a fuller syllabus : § 1. Definitions of the objects of the science : its relations to physics • Uolo^" ''^'*''" '^ '^' ^^^''''^ ''''''''' (chemistry and phS ?o § 4. Definitions of matter in its three forms-gaseous, liquid and sohd ; a chemist confines his attention to homogenous forms JmatS- importance of mass (weight) as a measure of mitter ' m w 180 ADDENDA. If ; f' ! ; I All matter, without an exception, is subject to chemical change ; such changes may be classified into three divisions. Those in which each resultant form is lighter than (1), heavier than (2), or of equal weijght (3) with tho initial form. By continuing the chemical changes which result in a lighter form of matter, chemists are led to a limited number of forms which can not be made to give any lighter matter. These forms of matter have distinct spectra as gases. From these, in almost all cases, the original matter may be constructed. They are therefore caUed the elements. § S. The names of the elements. The laws of combination of the elements in Dejinite Proportion, Multiple Proportion^ Reciprocal Pio- portion, Dalton's theory, that the elements are composed of atoms, explains these laws. The use and meaning of the term molecule. The use of symbols to denote atoms and molecules, and the use of equations to denote chemical change. § 4. Dalton's theory does not admit of practical application unless we have the means of measuring the number of atoms in a molecule. Dalton assumes that he knew this number, e.g. HO for water. Chemists solve this problem by Avogadro's law that "equal volumes of gases, measured at the same temperature and pressure, contain the same number of molecules, and therefore weigh in the ratio of the weights of these molecules, " deduced from the physical laws of gases, and from their relative densities as compared with their combining weights, and also from the laws of combination by volume. § 5. The study of the combination of the elements hydrogen and chlorine gives proof that the molecule of hydrogen contains two parts. The study of the compound hydrogen chloride convinces chemists that these parts are indivisible and therefore atoms. Hydrogen is therefore represented by the symbol Hg. § 6. Hence that volume of any gas will weigh its molecular weight in any system of units, which weighs two such units when filled with hydrogen gas at the same temperature and pressure. Thus 22.327 litres 0° C. and 760 m.m. Bar of hydrogen weigh 2 grams, and of any other gas its molecular weight in grams. In like manner 377 cubic feet at 60° F. and 30 inches Bar of hydrogen weigh 2 lbs., and therefore this volume of any gas at the same tempera- ture and pressure weigh its molecular weight in lbs. avoirdupois. § i. Chemists have agreed to take the least weight of any element found in such a molecular weight as the weight of the atom. § 8, The law of the specific heat of the elements may be used to de- dermine atomic weight. § 9. Classification of the elements by their atomic weight and by the chemic"' 'character of their compounds. Outlines of Mendelejeff's classi' 1. AUotropic modifications of the elements. Valency. § 10. Acid. Salts. Bases. Nomenclature. ADDENDA. in 181 § 11. The law of isomorphism, its application to the determination of atonuc Weight. Heat as cause and § 12. The conditions of chemical combination result of chemical action. Electricity. Light. § 13. Many of the physical properties of bodies may be traced to the E^ mds individual atoms. Molecular volume of solids and This syllabus is intended to indicate to tlie student the most important theories of chemistry. The following selection of the elements, with their most characteristic compounds, may be studied in illustration of the outlines of MendelejelTs classi- fication of the elements : Hydrogen Sodium Potassium Magnesium Calcium Zinc Strontium Barium Carbon Silicon Tin Lead Nitrogen Phosphorus Arsenic Antimony Bismuth Oxygen Sulphur Fluorine Chlorine Bromine Iodine Boron Aluminium Manganese Iron Gold Platinium The student is reminded that those facts are most necessary to be remembered which are of importance to the whole subject. INDEX. (The numerals refer to pages.) invA U ,if I , ;i..' If ■ « r . ( ' t< << Acids, 90. carbonic,- 136. chloric, 126. chlorous, 126. hydriodic, 129. hydrofluorio, 132. hydrochloric, 114. hypochlorous, 126. metaphosphoric, 141. nitric, 95. nitrons, 100. orthophosphoric, 140. perchloric, 12C. silicic. 144. sulphuric, 87. sulphurous, 86. Acids on metals, 116. Addenda, 279. Air. 14, 27, 148. [1.51,^2. *« composition of, IB, 16, 27, 147, " impurities in, 149, 151, 152. Aff>te, 1^3. Alkali family, 194. Alkalies, 92. Allotropism, 68. Alum, 145. Aluminum, 145, 179. " silicate, 145. Amethyst, 148. Ammonia, 105, 108, 113. AltltiiuiiiUiil, 2u2. " chloride, 109, 110. " hydroxide, 108, 111. Analysis, chemical, 238. " of air, 27. " of water, 30. Analytical tables, 240. Anhydride. 86. Atimoniuretted hydrojfen. 175. Antimony, 174. Apparatus, general, 249. Arsenic poisoning", 174. Artiads, 60. Atom, 46. Atomic weifjhts, 48, 51. Atomicity, 59, 60. Avogadro's Law, 45. B. Barium. 185. Bases. 91. Basicity, 97, 141. Bismuth, 175. Bleaching, 85, 122. " powder, 122. Boracic acid, 178. Borates tests, 179. Borax, 177. Books of reference, 251. Boron, 177. Boyle's Law , 45, 73. Bromine, 130. " compound, 130, 131. Burning, 16. ' ' of air, 17. Calcedony, 143. Calcium, 133. •' hydroxide, 134, " carbonate. 136. " phosphate, 138. " family, 183. Calculations chemical, 74, 75, Carbon, 65. " dioxide, G8, 69. " .dioxide, decomposition of, 6o " forms, iB7. " monoxide, 70, 71. " properties of, 66. Charcoal animal, 67. " wood, 67. Change, chemical, 6. " chemical, cause of, 7. " physical, 6. Charles' Law, 45, 73. Chemistry, definition of, 160- '• and physics, 161. " and biology, 1C2. Chemical action, 16:5. " analysis, 238. Chlorines, 119, 120, 122. Chlorine compounds, 126. Chlorides, 115. Clark's solution, 159. Clav, 144. Coal, 6V. gas, 78. " tar, 79. Combination of elements, -■•" Combustion, 79. Compounds binary, 61. " chemical. 11, 13. " mechanical, 10. •' preparation of, 18i Composition percentt^e, 68. INDEX. 283 of, 60 Daltou's Theory, KM. Decouijwsitlon, 12. Deodorant, 123. Dififusion of ga-ses, 1.03. Disinfectant, 123. Distillation, 2!). Displacement, 49. Dulong & Petit'a Law, 49. Electrioitv, 9,. 04. Electrolysis of watei', 31, Elements, 13. " Classification of, 169. " List of, 246. " symbols, 53. Electro-chemical series, 52. Equations, 62, 63. " molecular, 64. Equivalents, 51. Ethylene, 78. Eudiometer, 32. Examination Papers, 256. " Questions, 261 Facts and theories, 5, 46. Ferric Chloride, 2i9. Ferrous " 219. Sulphate, 220. Sulphide, 220. Filtration, 3. Flame structure, 80. " of candle, 91. " of Bunsen, 81. Finit, 143. Fluorine, 143. Forces, chemical, 7. heat, 7. *' mechanical, 8. light. 8. " electricity, 9, 54. " chemisra, 10. Formulae, 55, 56, 57, 59, 61. Furrum redactum, 11. Q. Gay-Lussae's law, 4n. Glass, 145. Gold, 213. Graham's Law, 154. Gypsum, 137. H. Halogens, 127. Hardness of water, 158. Harmonirum chemical, 36. Honestone, _i3. Hydrocarbons, 76. lydrogen, 34. " Antimoniuretted, 175. " prooerties of, 35, 36, 37, 38, 39. Hydrates, V. ii i< Hydrochloric acids, 114, 117, 118. Hydroxides, 91, 94, 112. Hypothesis, 46. I. Incandescent gas, 80. Iodine. 127. ■' compounds, 128. Iron Family, 215. Isomorphism, Laws of, 229. L. Law of constants, 19, 42. Boyle's 4;'), 73. Gay-Lussac's and Charles, 45, 73. Avogadro's, 45. Dulong & Petit's, 49. " multiple proportion, 71. " Graham's, 154. Lead, 203. " Compounds of, 205. " Fanuly, 203. '• Poisoning, 205. T.eblanc's method, 115. Lime, 112, 133. " quick, 133. stone, 135. phosphate, 137. M. Magnesium Family, 189. Manganese, 222. Matches, 139. Material and Reagents, 250. Matter, indestructible, 18. Matriculation Work, 278. Mercury, 235. Metals, 53. " Compounds of, 182. •' Extraction of, 181. Metric System, 246. Mixture, mechanical, 11. Molecules, 5, 46. " Division of, 166. Mortar, 136. N. Nascent state, 106. Nitrogen, 25, 26. " acids, 105. " dioxide, 101. " family, 172. " monoxide, 103. " trioxides, 99. Nomenclature, 60. Notation, chemicil, 54. O. defiant gas, 77. Opal, 143. Ores aiul Extraction, 215. Oxidizing agents, 96. t 'xygen, 21. " properties of, 24. i i J'' ' > ji 284 Oxygen conipounrls, 24. Ozone, 150. Periodic Law, 226. Perissads, CO. Physical chanpe, 6. " characters of gases, 44, 4.'j. PhosphoruB, 138, 130. " aoidtf, 140, 141. •• Of id hydrogen, 142. Plants influence on air, 152. Platinum Family, 212. Potaa&iuin, 194. " chlorate, 124, 125. " compounds, 195. " ferrocyanide, 220. hydroxide, 197. nitrate, 97, silicate, 110. Pottery, 146. Pressure, standard, 73, 74. INDEX. Quartz, 14;?. Q jartzite, 143. R. Radical, 111 Reagents, Students, 249. " and material, 250. Reference, Books of, 251. S. Salts, 92. " naming of, 93. Sand, 143. Saturation, 3. Science Room, 248. Seleneum, 170. Silicon, 143. Silicates, 144. Silicic acid, 144. Signs, 63. Silver Family, 231. Sodium, 198. «' silicate, 146. Soil, 146. Solution of solids, 1. " of liquids, 3. Solution of gnseSt 4. •' Theory of, ^ " Problems on, 5. Speciflo gravity of gases, 47. " gravity of air, 148. Steam, volume of. 33. Strontium. 187. Students' Rengentfj, 249. Sulphides, 80. Sulphur, 83. " allotropfts 83. " dioxide, 84. •' trioxide, 86. Sulphuretted hydrogen, 89. Sulphurous acid, 85. Sulphuric " 86,87,88. Synthesis of water, 31. T. Tellurium, 171. Temperature, standard, 73, 74. ** of ignition, 82. Tin, 208. " compounds of, 209. Theory, 46. U. Unit of Volume, 168. Union, chemical, 63. University of Toronto, 251. " College, 252. Valence, 59. 60. Ventilation, 15'i. Volume, combination by, 20, 63. " unit of, 168. '• units, 72. W. Water, impurities in, 155, 155, 158. pure, 28, 30, 31. sea, 157. Weights, metric, " combination, 20, 63. Wood-tar, 67. Z. Zinc, 191. ijs'j- THE COPP, CLARK COMPANV, LIMITED, PRINTERS, COLBORNK 8TRBKT, TORONTO.